STUDY-MODE - Practice Exam 7 Flashcards

1
Q

A Jungian “archetype” is best described as:
Select one:

A. an organized constellation of feelings, thoughts, and perceptions.
B. a structural component of the collective unconscious.
C. the “mask” adopted by a person in response to social demands.
D. a significant event that occurred during childhood.

A

According to Carl Jung, the psyche consists of a number of different, but interrelated, elements. For Jung, the major division in the psyche is between the personal unconscious and the collective (transpersonal) unconscious, with each containing unique elements.

a. Incorrect This better describes Jung’s notion of “complexes” (e.g., mother complex), which exist in the personal unconscious.
b. CORRECT An archetype is a universal thought form that is deposited in the mind when an experience has been constantly repeated for many generations. Numerous archetypes reside in the collective unconscious, including the archetypes of birth, death, the earth mother, and the wise old man.
c. Incorrect This best describes the “persona,” which is acquired in response to social demands as well as archetypal needs.
d. Incorrect This is not an accurate description of archetypes.

The correct answer is: a structural component of the collective unconscious.

How well did you know this?
1
Not at all
2
3
4
5
Perfectly
2
Q

When working with Native American/Alaskan Native clients, an effective intervention would involve coordination with traditional healers (if appropriate), incorporation of members of the extended family, and a(n):
Select one:

A. insight-oriented approach.
B. reality therapy or a solution-focused approach.
C. combination of client-centered and behavioral approaches.
D. combination of Adlerian and rational-emotive approaches.

A

Answer C is correct. There is some disagreement among the experts about the optimal therapeutic approach for members of this population; however, it depends on several factors including the client’s level of acculturation. D. W. Sue and D. Sue (Counseling the culturally different, New York, John Wiley & Sons, 1999) recommend a combination of client-centered and behavioral approaches.

The correct answer is: combination of client-centered and behavioral approaches.

How well did you know this?
1
Not at all
2
3
4
5
Perfectly
3
Q

The primary target of “advocacy consultation” is best described as:
Select one:

A. worker well-being.
B. the child’s best interests.
C. interpersonal conflicts.
D. social change.

A

Advocacy consultation, unlike other consultation modes, focuses on social systems rather than individuals or small groups.

a. Incorrect See explanation for response d.
b. Incorrect See explanation for response d.
c. Incorrect See explanation for response d.
d. CORRECT As noted by J. C. Conoley (Consultation in schools: Theory, research, procedures, Academic Press, New York, 1981), a distinctive characteristic of advocacy consultation is that it is based on an “explicit value orientation that targets social change in the direction of power equalization” (p. 162).

The correct answer is: social change.

How well did you know this?
1
Not at all
2
3
4
5
Perfectly
4
Q

A practitioner of nonsexist therapy:
Select one:

A. interprets a client’s behavior in terms of social, political, and cultural forces.
B. stresses the egalitarian nature of the therapeutic relationship.
C. views gender as a key determinant of behavior.
D. uses non-biased techniques designed to promote personal growth.

A

Nonsexist therapy and feminist therapy share several characteristics. For the exam, you want to be familiar with their major similarities and differences, which are described in the Clinical Psychology chapter of the written study materials.

a. Incorrect This is characteristic of feminist therapy but not nonsexist therapy.
b. Incorrect A nonsexist therapist may or may not emphasize an egalitarian relationship depending on his/her theoretical orientation. Therefore, this is not the best answer.
c. Incorrect For nonsexist therapists, gender is only one of many personal characteristics that affect behavior and are relevant to counseling.
d. CORRECT Of the answers given, this is the best one. As its name implies, nonsexist therapy involves using nonsexist (unbiased) techniques. In addition, its focus is on personal growth.

The correct answer is: uses non-biased techniques designed to promote personal growth.

How well did you know this?
1
Not at all
2
3
4
5
Perfectly
5
Q

Among the groups listed below, admission rates for psychiatric hospitalization are highest for which of the following?
Select one:

A. never married men
B. married men
C. divorced men
D. widowers

A

Of the choices given, never married men have the highest rate of psychiatric hospitalization for both men and women. Other factors to consider for admission rates include age (with the highest proportion between age 25 to 44) and race (with whites representing the largest number of psychiatric inpatients).

a. CORRECT This is the correct response. See the Clinical Psychology chapter for additional information on psychiatric hospitalization rates.
b. Incorrect See explanation for response a.
c. Incorrect See explanation for response a.
d. Incorrect See explanation for response a.

The correct answer is: never married men

How well did you know this?
1
Not at all
2
3
4
5
Perfectly
6
Q

A family therapist using Minuchin’s structural approach is most likely to be interested in which of the following?
Select one:

A. communication styles
B. transgenerational processes
C. boundaries
D. exceptions

A

As its name implies, Minuchin’s structural family therapy focuses on the structural characteristics of the family.

a. Incorrect Although communication patterns provide clues about family structure, this is not the best response of those given.
b. Incorrect This sounds more like Bowenian family therapy.
c. CORRECT Minuchin considered maladaptive behavior to be the result of overly rigid or permeable boundaries between family members.
d. Incorrect Exceptions are of interest to solution-focused therapists.

The correct answer is: boundaries

How well did you know this?
1
Not at all
2
3
4
5
Perfectly
7
Q

In structural family therapy, “joining” depends on:
Select one:

A. the motivation of family members to change.
B. the permeability of the boundaries between family members.
C. the therapist’s ability to remain neutral.
D. the therapist’s ability to adapt to the family.

A

As described by Minuchin, joining is both a diagnostic and restructuring technique. It involves relating to family members and the family system by, for example, adopting its communication style.

a. Incorrect See explanation for response d.
b. Incorrect See explanation for response d.
c. Incorrect See explanation for response d.
d. CORRECT In order to join the family, the therapist must be able to adapt or accommodate to its style.

The correct answer is: the therapist’s ability to adapt to the family.

How well did you know this?
1
Not at all
2
3
4
5
Perfectly
8
Q

As defined by Carl Jung, individuation refers to:
Select one:

A. being able to separate one’s intellectual and emotional functioning.
B. differentiating between self and object.
C. experiencing a state of anonymity.
D. integrating parts of the self to create a unique identity.

A

For the exam, you want to be familiar with all of the terms that are referred to in the four answers to this question.

a. Incorrect This answer describes Bowen’s concept of differentiation. According to Bowen, the greater a person’s differentiation, the better able he/she is to resist becoming emotionally fused with other family members.
b. Incorrect This answer refers to Mahler’s separation-individuation phase of early development.
c. Incorrect This describes deindividuation which is used to explain why a person may be more willing to act aggressively or in other uncharacteristic ways when acting as a member of a group than when acting alone.
d. CORRECT As defined by Jung, individuation is the goal of psychological development and involves integrating the conscious and the unconscious elements of the psyche.

The correct answer is: integrating parts of the self to create a unique identity.

How well did you know this?
1
Not at all
2
3
4
5
Perfectly
9
Q
William Cross (1971, 1991) used which of the following terms to describe the transformational process that involves a shift from a non-Afrocentric to an Afrocentric identity?
Select one:

A. Afrocentrism
B. nigrescence
C. integrative awareness
D. racination

A

Knowing the original name of Cross’s Black Racial Identity Development Model - i.e., the Nigrescence Identity Development Model - would have helped you identify the correct answer to this question.

a. Incorrect Afrocentrism refers to a worldview that emphasizes Africa and African people, especially in terms of their influence on culture and history.
b. CORRECT Cross (1971) originally used the French term for “turning black” (i.e., nigrescence) as the name for his identity development model, and he described Black identity development as involving a transformation from a pre-existing non-Afrocentric identity to an Afrocentric identity.
c. Incorrect Integrative awareness is the final stage in Atkinson, Morten, and Sue’s (1998) Racial/Cultural Identity Development Model.
d. Incorrect The term racination is sometimes used to describe the reverse of deracination, which refers to removing or displacing someone or something from its natural environment.

The correct answer is: nigrescence

How well did you know this?
1
Not at all
2
3
4
5
Perfectly
10
Q

The most common informal caregivers for older adults with chronic health conditions are family members. More specifically, the most common primary caregiver is:
Select one:

A. the spouse or adult daughter.
B. the adult daughter or son.
C. a same-gender sibling.
D. another relative.

A

The most common informal caregiver depends on the availability of family members and the gender of the individual requiring caregiving. However, some generalizations can be drawn from the research.

a. CORRECT The research has found that the spouse is the most common primary caregiver, with the children adopting a secondary role. However, when there is no spouse, a daughter is most likely to assume the role of primary caregiver. See, e.g., S. H. Zarit and D. J. Eggebeen, Parent-child relationships in adulthood and later years, in M. H. Bornstein (Ed.), Handbook of parenting: Children and parenting (Vol. 1, pp. 135-164), Hillsdale, NJ, Lawrence Erlbaum, 2002.
b. Incorrect See explanation for response a.
c. Incorrect See explanation for response a.
d. Incorrect See explanation for response a.

The correct answer is: the spouse or adult daughter.

How well did you know this?
1
Not at all
2
3
4
5
Perfectly
11
Q

Which of the following would probably be LEAST useful when a therapist is working with an Asian or Asian American client?
Select one:

A. identifying therapy goals during the initial session
B. establishing an egalitarian relationship with the client early in therapy
C. making use of “relationship questions” that elicit information about the client’s interactions with others
D. asking the client to try to identify exceptions to the presenting problem

A

It is “dangerous” to make generalizations about the best treatment approach for any group of clients, but the licensing exam may include a question that requires you to do so.

a. Incorrect A direct approach that focuses on specific goals is considered useful for Asian American clients.
b. CORRECT An egalitarian relationship is usually contraindicated because of “the Asian norms of deference to authority and modesty in the presence of superiors” (H. Kitano and M. Maki, Continuity, change, and diversity, in P. B. Pedersen, et al., Counseling across cultures, Thousand Oaks, Sage Publ., 1995).
c. Incorrect Focusing on relationships is a good approach.
d. Incorrect Identifying strengths is also useful.

The correct answer is: establishing an egalitarian relationship with the client early in therapy

How well did you know this?
1
Not at all
2
3
4
5
Perfectly
12
Q

First-and second-grade children who have been identified as being at risk for academic underachievement are provided with a special after-school program. This is an example of:
Select one:

A. primary prevention.
B. secondary prevention.
C. tertiary prevention.
D. crisis intervention.

A

Interventions aimed at identified individuals that have as their goal the reduction of the severity and/or prevalence of a disorder are categorized as secondary preventions.

a. Incorrect As defined by many authorities, primary preventions are aimed at populations rather than at specific individuals who have been identified as being at-risk.
b. CORRECT Since the school program is being given to identified individuals (children who have been identified as being at risk for academic problems), this qualifies as a form of secondary prevention.
c. Incorrect Tertiary preventions are more rehabilitative in nature.
d. Incorrect Clearly, this program is not a form of crisis intervention.

The correct answer is: secondary prevention.

How well did you know this?
1
Not at all
2
3
4
5
Perfectly
13
Q

Polly Pathos is seeing Dr. Genial for her depressive symptoms. As a practitioner of Interpersonal Therapy (IPT), Dr. Genial will most likely:
Select one:

A. help Polly gain insight into how interpersonal experiences during childhood are contributing to Polly’s problems in the present.
B. focus on clarifying how Polly’s current relationships and her expectations about them are contributing to her symptoms.
C. have Polly participate in a family therapy group that targets the dysfunctional interaction patterns that contribute to symptoms.
D. help Polly identify and understand how interpersonal “schemas” are inhibiting her ability to actualize her full potential.

A

Interpersonal Therapy is a brief form of therapy that was originally developed as a treatment for depression but has since been applied to other disorders.

a. Incorrect See explanation for response b.
b. CORRECT The focus of interpersonal therapy is on the link between current interpersonal problems and depressive symptoms. Interpersonal therapy targets one or more of the following areas: grief, interpersonal role disputes, role transitions, interpersonal deficits.
c. Incorrect See explanation for response b.
d. Incorrect See explanation for response b.

The correct answer is: focus on clarifying how Polly’s current relationships and her expectations about them are contributing to her symptoms.

How well did you know this?
1
Not at all
2
3
4
5
Perfectly
14
Q

Research on the use of hypnosis to recover repressed memories suggests that:
Select one:

A. memories recalled under hypnosis are both more accurate and more detailed than other memories.
B. memories recalled under hypnosis are about as reliable as other memories.
C. memories recalled under hypnosis are less reliable than other memories and may be more susceptible to distortion.
D. memories recalled under hypnosis are almost always confabulated.

A

It has been difficult to check the accuracy of “repressed memories” that have been retrieved under hypnosis, and there is not a great deal of empirical research on this topic. However, the research on memory suggests that memories are often constructed (or reconstructed) to some degree rather than simply recalled and that hypnosis can exacerbate this process.

a. Incorrect See explanation for response c.
b. Incorrect See explanation for response c.
c. CORRECT This is the best conclusion that can be drawn about repressed memories retrieved under hypnosis. While some memories may be accurate, care must be taken in accepting memories retrieved under hypnosis since hypnosis seems to make people particularly susceptible to suggestion and to believing that false memories are, in fact, true.
d. Incorrect See explanation for response c.

The correct answer is: memories recalled under hypnosis are less reliable than other memories and may be more susceptible to distortion.

How well did you know this?
1
Not at all
2
3
4
5
Perfectly
15
Q

From the perspective of Gestalt therapy, an optimal “contact boundary” is semipermeable and coterminous with the actual self. Introjection is one type of boundary disturbance. It occurs when:
Select one:

A. the contact boundary is located far in the environment.
B. the contact boundary is located deep within the self.
C. the contact boundary is turned back against the self.
D. the contact boundary is extremely rigid.

A

Introjection occurs when a person incorporates aspects of another person into him/herself.

a. Incorrect This is projection.
b. CORRECT The boundary is located inside one, thereby allowing others to be incorporated into the self.
c. Incorrect This is retroflection.
d. Incorrect This is deflection.

The correct answer is: the contact boundary is located deep within the self.

How well did you know this?
1
Not at all
2
3
4
5
Perfectly
16
Q

According to Helms’s White Identity Development Model, the reintegration stage is characterized by:
Select one:

A. a realization that whites have a responsibility for racism.
B. adoption of a “culture-blind” perspective.
C. embracing white identity while rejecting racist views of minorities.
D. adopting a belief in white superiority and minority inferiority.

A

Helms’s model of White identity development includes six statuses:

CDRPIA

  1. contact
  2. disintegration
  3. reintegration (Adopting a belief in white superiority and minority inferiority.)
  4. pseudo-independence
  5. immersion-emersion (A realization that whites have a responsibility for racism.) (Embracing white identity while rejecting racist views of minorities.)
  6. autonomy.

a. Incorrect This is more characteristic of immersion-emersion and autonomy statuses.
b. Incorrect This is not characteristic of any of the stages described by Helms.
c. Incorrect This is characteristic of immersion-emersion status.
d. CORRECT The disintegration phase, which is marked by considerable conflict, may be resolved by adopting traditional racist views, which in turn characterizes the reintegration phase.

The correct answer is: adopting a belief in white superiority and minority inferiority.

How well did you know this?
1
Not at all
2
3
4
5
Perfectly
17
Q

According to Margaret Mahler’s object relations theory, the development of a sense of self is the result of a separation-individuation process that begins at about _____ months of age.
Select one:

A. 2 to 3
B. 4 to 5
C. 8 to 10
D. 12 to 14

A

Mahler distinguishes between two phases that contribute to the development of a unique sense of self: symbiosis and separation-individuation.

a. Incorrect An infant is in the symbiosis phase at this age.
b. CORRECT Separation-individuation is triggered by the child’s ability to separate from his/her primary caregiver. The process begins at about 4 to 5 months of age when a child who is being held by his/her caregiver is able to lean away to scan the environment.
c. Incorrect This age is very important for the separation-individuation process because the child is now able to crawl away from his/her caregiver. However, the process begins prior to 8 to 10 months of age.
d. Incorrect See explanation above.

The correct answer is: 4 to 5

How well did you know this?
1
Not at all
2
3
4
5
Perfectly
18
Q

When working with older adults, it is important to remember that:
Select one:

A. psychotherapy, especially insight-oriented therapy, is generally ineffective.
B. there is greater variability among older people than younger people on a range of characteristics.
C. a therapeutic alliance is particularly difficult to establish, especially when the therapist is young.
D. all of the above.

A

When working with older adults, it is important to remember that there is greater variability among older people than younger people on a range of characteristics (physical health, cognitive skills, income, etc.).

a. Incorrect See explanation above.
b. CORRECT See explanation above. Additional information about providing therapy to older adults is provided in the Clinical Psychology chapter of the written study materials.
c. Incorrect See explanation above.
d. Incorrect See explanation above.

The correct answer is: there is greater variability among older people than younger people on a range of characteristics.

How well did you know this?
1
Not at all
2
3
4
5
Perfectly
19
Q

George Kelly is associated with which of the following?
Select one:

A. Reality Therapy
B. Personal Construct Therapy
C. Existential Therapy
D. Solution Focused Therapy

A

George Kelly developed Personal Construct Therapy (PCT) in the 1950s as both a theory of personality and approach to therapy.

a. Incorrect See explanation for response b.
b. CORRECT Personal Construct Therapy (also known as “constructive alternativism”) is based on the premise that people construe (construct) their own experiences. His approach was very influential in the development of narrative-constructivist approaches to therapy. Additional information about Personal Construct Therapy is provided in the Clinical Psychology chapter of the written study materials.
c. Incorrect See explanation for response b.
d. Incorrect See explanation for response b.

The correct answer is: Personal Construct Therapy

How well did you know this?
1
Not at all
2
3
4
5
Perfectly
20
Q

Research comparing the use of mental health services by heterosexual adults and gay/lesbian adults has generally found that:
Select one:

A. gay men are more likely than heterosexual men to use mental health services, but lesbian women are less likely than heterosexual women to do so.
B. lesbian women are more likely than heterosexual women to use mental health services, but gay men are less likely than heterosexual men to do so.
C. gay men and lesbian women are more likely than their heterosexual counterparts to use mental health services.
D. gay men and lesbian women are less likely than their heterosexual counterparts to use mental health services.

A

A number of studies have confirmed a disparity in mental health service use by heterosexual adults and gay/lesbian adults.

a. Incorrect See explanation for response c.
b. Incorrect See explanation for response c.
c. CORRECT A frequently cited investigation of mental health service use by gay and lesbian adults was conducted by S. D. Cochran, J. G. Sullivan, and V. M. Mays [Presence of mental disorders, psychological distress, and mental health service use among lesbian, gay, and bisexual adults in the United States, Journal of Consulting and Clinical Psychology, 71(1), 53-64, 2003]. Their results (which have been subsequently replicated by several other researchers) indicated that gay and lesbian adults have higher levels of perceived need for mental health services and are more likely to use mental health services.
d. Incorrect See explanation for response c.

The correct answer is: gay men and lesbian women are more likely than their heterosexual counterparts to use mental health services.

How well did you know this?
1
Not at all
2
3
4
5
Perfectly
21
Q

When working with an African American client exhibiting “healthy cultural paranoia,” an Anglo therapist would be best advised to:
Select one:

A. refer the client to an African American therapist.
B. use a culturally sensitive approach that ameliorates the client’s paranoia.
C. help the client bring feelings of suspiciousness, frustration, and antipathy toward whites into conscious awareness.
D. help the client understand that his/her behavior is manifestation of resistance.

A

C. R. Ridley presents a “typology of black client self-disclosure,” which relates the willingness to self-disclose to four levels of “paranoia”: intercultural nonparanoic disclosure; functional paranoiac; healthy cultural paranoiac; and confluent paranoiac (Clinical assessment of the nondisclosure of the black client, American Psychologist, 39(11), 1234-1244, 1984).

a. Incorrect This is probably the best course of action, according to Ridley, for the confluent paranoiac but isn’t necessary for the individual experiencing healthy cultural paranoia.
b. Incorrect This isn’t the best response since healthy cultural paranoia may not need to be “ameliorated.”
c. CORRECT Ridley recommends that therapists confront the meaning of the client’s cultural paranoia by bringing his/her feelings into conscious awareness and then help the client clarify when it is appropriate or inappropriate to self-disclose.
d. Incorrect This is not the course of action recommended by Ridley.

The correct answer is: help the client bring feelings of suspiciousness, frustration, and antipathy toward whites into conscious awareness.

How well did you know this?
1
Not at all
2
3
4
5
Perfectly
22
Q

A psychotherapist is most likely to say that which of the following is the greatest source of stress in his or her work?
Select one:

A. isolation
B. overwork
C. the general passive nature of his/her work
D. the lack of therapeutic success

A

A survey of psychotherapists conducted in the early 1980s indicated that therapists expect their work to be stressful but also expect that their efforts will be appreciated and have positive results.

a. Incorrect In their survey of therapists, B. A. Farber and L. J. Helfetz (The process and dimensions of burnout in psychotherapists, Professional Psychology, 13(2), 293-301, 1982) found that isolation was cited by only 11.1% of their respondents as a major source of stress.
b. Incorrect Only 22.2% of respondents said that overwork was a major problem.
c. Incorrect About 13% of respondents cited this as a source of stress.
d. CORRECT The overwhelming majority of respondents (73.7%) said the major source of stress in their work was the lack of therapeutic success.

The correct answer is: the lack of therapeutic success

How well did you know this?
1
Not at all
2
3
4
5
Perfectly
23
Q

Freud (1984) introduced the concept of “defense” in his description of hysteria and attributed hysteria to which of the following defense mechanisms?
Select one:

A. projection
B. reaction formation
C. sublimation
D. repression

A

Freud’s initial explanation for hysteria was that it represents an attempt to eliminate memories related to early psychic trauma from consciousness.

a. Incorrect See explanation for response d.
b. Incorrect See explanation for response d.
c. Incorrect See explanation for response d.
d. CORRECT Repressions is considered the most basic defense mechanism because it underlies all others. It was the initial defense mechanism identified by Freud, and he used it to explain the etiology of hysteria.

The correct answer is: repression

How well did you know this?
1
Not at all
2
3
4
5
Perfectly
24
Q

In family therapy, the adolescent daughter complains that she doesn’t think she can change her attitudes toward school and feels that doing what the therapist has requested is pointless. The therapist responds by saying, “Well, in that case, I think you shouldn’t even try.” The therapist’s response is an example of which of the following?
Select one:

A. reactance
B. restraining
C. positioning
D. prescription

A

In the situation described in the question, the therapist is using a paradoxical technique.

a. Incorrect Reactance is not a paradoxical technique but refers to the tendency of people to do the opposite of what they are asked to do, especially when they feel their personal freedom is being threatened.
b. CORRECT This technique (telling a client not to change) is referred to as restraining.
c. Incorrect Positioning involves exaggerating the severity of the symptom.
d. Incorrect Prescription involves instructing the client to deliberately engage in the target behavior, usually in an exaggerated form (e.g., “nag your spouse for at least three hours each evening”).

The correct answer is: restraining

How well did you know this?
1
Not at all
2
3
4
5
Perfectly
25
Q

Freud and the object relations theorists share in common an emphasis on the role of _________________ in personality development.
Select one:

A. the superego
B. innate sexual and aggressive drives
C. early childhood experiences
D. separation and individuation

A

Object relations theory shares a number of assumptions in common with Freudian theory but differs in its emphasis on the need for interpersonal relationships (versus needs related to sex and aggression).

a. Incorrect This is true about Freudian theory but not about object relations theory.
b. Incorrect This characterizes Freudian theory.
c. CORRECT Object relations theorists, like Freud, believe that early childhood experiences are critical to personality development. They also posit a stage theory of development, but they view interpersonal relations as the primary contributor to personality.
d. Incorrect This applies only to object relations theory.

The correct answer is: early childhood experiences

How well did you know this?
1
Not at all
2
3
4
5
Perfectly
26
Q

A couple comes to therapy complaining that their conversations always end up as arguments. In talking with the couple, you realize that a common pattern is for one partner to make an offensive comment to the other and for the other partner to respond with an even more offensive remark. This pattern of communication is referred to as:
Select one:

A. symmetrical.
B. complementary.
C. mystification.
D. pseudohostility.

A

In this situation, the husband and wife are mirroring each other’s behavior in what is sometimes referred to as a “one-upsmanship” game.

a. CORRECT Symmetrical communication is characterized by equality between the partners. It can lead to conflict and competition (symmetrical escalation) when each partner tries to “one-up” the other.
b. Incorrect Complementary interactions are based on inequality with one partner assuming a dominant role and the other a subordinate position.
c. Incorrect Mystification refers to masking what is really going on through denial in order to maintain the status quo.
d. Incorrect Pseudohostility occurs when real conflicts are denied and obscured by superficial bickering.

The correct answer is: symmetrical.

How well did you know this?
1
Not at all
2
3
4
5
Perfectly
27
Q

A person who experiences cataplexy as part of his narcoleptic attacks will most likely try to avoid:
Select one:

A. afternoon naps.
B. expressing strong emotions.
C. fluctuations in salt intake.
D. eye strain.

A

Cataplexy involves bilateral loss of muscle tone.

a. Incorrect See explanation for response b.
b. CORRECT Cataplexy is usually triggered by intense emotions. Consequently, people with cataplexy often attempt to control their emotional expressiveness.
c. Incorrect See explanation for response b.
d. Incorrect See explanation for response b.

The correct answer is: expressing strong emotions.

How well did you know this?
1
Not at all
2
3
4
5
Perfectly
28
Q

PET scans of people with _______________ show that these individuals often have increased activity levels in the orbitofrontal cortex, cingulate cortex, and caudate nucleus.
Select one:

A. Posttraumatic Stress Disorder
B. Narcolepsy
C. Obsessive-Compulsive Disorder
D. Schizophrenia

A

Knowing that the caudate nucleus is one component of the basal ganglia and that the basal ganglia are involved in voluntary movement and that the orbitofrontal cortex and cingulate cortex mediate emotional reactions may have helped you identify the correct response to this question.

a. Incorrect See explanation for response c.
b. Incorrect See explanation for response c.
c. CORRECT Activity in these areas of the brain is higher in individuals with OCD, especially during provocation of symptoms. In contrast, drug and behavioral treatments for OCD reduce activity in these areas.
d. Incorrect See explanation for response c.

The correct answer is: Obsessive-Compulsive Disorder

How well did you know this?
1
Not at all
2
3
4
5
Perfectly
29
Q

The mean age of onset of motor tics in Tourette’s Disorder is:
Select one:

A. 1 to 3 years.
B. 5 to 7 years.
C. 9 to 11 years.
D. 12 to 14 years.

A

For individuals with Tourette’s Disorder, the onset of motor tics ordinarily precedes the onset of vocal tics.

a. Incorrect See explanation for response b.
b. CORRECT The reported mean age of onset of MOTOR tics varies, but a commonly reported range is between 5 and 7 years. The onset of VOCAL tics is usually later, with a mean age of 11 years. See, e.g., A. Carroll and A. Robertson, Tourette Syndrome: A practical guide for teachers, parents, and carers, New York, David Fulton Publishers, 2003.
c. Incorrect See explanation for response b.

d. Incorrect See explanation for response b.
The correct answer is: 5 to 7 years.

How well did you know this?
1
Not at all
2
3
4
5
Perfectly
30
Q

The onset of bulimia often follows a period of moderate dieting, which has been linked to low blood levels of tryptophan. Low levels of tryptophan, in turn, are associated with:
Select one:

A. lower-than-normal levels of brain serotonin.
B. higher-than-normal levels of brain serotonin.
C. lower-than-normal levels of brain dopamine.
D. higher-than-normal levels of brain dopamine.

A

Even if you are unfamiliar with the link between tryptophan and serotonin, you would have been able to choose the correct response if you know that the SSRIs have been found effective for treating bulimia.

a. CORRECT Tryptophan is a precursor of serotonin, and a rapid drop in plasma levels of tryptophan is known to precede a decrease in brain levels of tryptophan and serotonin.
b. Incorrect See explanation for response a.
c. Incorrect See explanation for response a.
d. Incorrect See explanation for response a.

The correct answer is: lower-than-normal levels of brain serotonin.

How well did you know this?
1
Not at all
2
3
4
5
Perfectly
31
Q

The World Health Organization’s International Pilot Study of Schizophrenia (IPSS) and Determinants of Outcome Study have compared patients with Schizophrenia from non-Western developing countries to those from Western industrialized countries. This research has found that patients from developing and industrialized countries tend to differ in terms of:
Select one:

A. age and gender ratios.
B. course and outcomes.
C. symptom profiles.
D. gender and prognosis.

A

The World Health Organization studies have been reviewed by a number of investigators including K. Lin and A. Kleinman (Psychopathology and clinical course of Schizophrenia: A cross-cultural perspective, Schizophrenia Bulletin, 14(4), 555-567, 1988). The outcomes of these studies are consistent with research conducted by other investigators.

a. Incorrect See explanation for response b.
b. CORRECT The IPSS found course and outcome differences in patients from developing and industrialized countries; patients from developing countries were much more likely to exhibit an acute onset of symptoms, a shorter clinical course, and a complete remission of symptoms. Age, gender, and symptom profile were not found to differ in consistent ways.
c. Incorrect See explanation for response b.

d. Incorrect See explanation for response b.
The correct answer is: course and outcomes.

How well did you know this?
1
Not at all
2
3
4
5
Perfectly
32
Q

A 48-year-old woman is admitted to the emergency room of a hospital, complaining of chest pain and shortness of breath. While being interviewed, she is very restless and often gets up to pace back and forth. She says she has had similar “attacks” in the past, but that this one is the worst. She also states that the attacks sometimes occur when she’s concerned about getting bad news; but, most of the time, they don’t seem to be related to anything and they make her feel like she’s “going to die.” Assuming that there is no medical explanation for the woman’s symptoms, the most likely DSM-5 diagnosis is which of the following?
Select one:

A. Illness Anxiety Disorder
B. Specific Phobia
C. Panic Disorder
D. Panic Disorder without Agoraphobia

A

Answer C is correct: A DSM-5 diagnosis of Panic Disorder requires recurrent unexpected panic attacks with at least one attack being followed by one month of persistent concern about having additional attacks or about their consequences and/or involving a significant maladaptive change in behavior related to the attack.

Answer A: Illness Anxiety Disorder involves a preoccupation with having a serious illness, an absence of somatic symptoms or the presence of mild somatic symptoms, a high level of anxiety about one’s health, and performance of excessive health-related behaviors or a maladaptive avoidance of doctors, hospitals, etc.

Answer B: Specific Phobia is characterized by intense fear of or anxiety about a specific object or situation, with the individual either avoiding the object or situation or enduring it with marked distress.

Answer D: Panic Disorder without Agoraphobia is not a DSM-5 diagnosis.

The correct answer is: Panic Disorder

How well did you know this?
1
Not at all
2
3
4
5
Perfectly
33
Q

In terms of long-term effects, the most successful treatment for a DSM diagnosis of Enuresis is:
Select one:

A. fluid restriction and night lifting.
B. the night alarm.
C. a synthetic antidiuretic drug.
D. an antidepressant drug.

A

All of the treatments listed have been found useful for Enuresis, but only one has good long-term effects.

a. Incorrect While these interventions reduce symptoms initially, the benefits are usually short-term.
b. CORRECT The night alarm, which is the current version of the bell-and-pad, is successful in about 80% of cases, and, of the treatments listed, it has the best long-term effects.
c. Incorrect Although synthetic antidiuretics have good short-term effects, relapse rates are high once the drug is discontinued.
d. Incorrect A high relapse rate is also associated with antidepressants, which is why they are rarely used alone in the treatment of Enuresis.

The correct answer is: the night alarm.

How well did you know this?
1
Not at all
2
3
4
5
Perfectly
34
Q

Which of the following is most likely to be an effective treatment for PTSD?
Select one:

A. cognitive processing therapy
B. multisystemic therapy
C. critical incident stress debriefing
D. mentalization-based therapy

A

Answer A is correct: Cognitive processing therapy (CPT) has been found to be an effective treatment for PTSD. It integrates psychoeducation, exposure, and cognitive restructuring.

Answer B: Multisystemic therapy (MT) is also known as multisystemic treatment and is a comprehensive intervention that targets individual, family, school, peer group, and community factors that are maintaining conduct problems. It is considered especially useful for youth in mid- to late-adolescence who have exhibited the more serious symptoms of Conduct Disorder and are at-risk for out-of-home placement.

Answer C: Critical incident stress debriefing (CISB) is a type of psychological debriefing that involves providing treatment to the victims of a trauma within 72 hours of the event whether or not they have exhibited symptoms, and it is usually administered during a single lengthy session. Research on its effects suggests that it may actually worsen symptoms of PTSD by interfering with the normal recovery process.

Answer D: Mentalization-based therapy (MBT) has been found to be an effective treatment for Borderline Personality Disorder.

The correct answer is: cognitive processing therapy

How well did you know this?
1
Not at all
2
3
4
5
Perfectly
35
Q

Among older adults, major surgery, congestive heart failure, and pneumonia are all associated with an increased risk for which of the following?
Select one:

A. dyskinesia
B. dementia
C. delirium
D. dystonia

A

The risk for delirium is affected by age, with children and older adults being at highest risk.

a. Incorrect See explanation for response c.
b. Incorrect See explanation for response c.
c. CORRECT The conditions listed in the question are among those that are associated with an increased risk for delirium for older adults. (Note that one of the most common causes of delirium for members of this population is intoxication with prescription drugs.)
d. Incorrect See explanation for response c.

The correct answer is: delirium

How well did you know this?
1
Not at all
2
3
4
5
Perfectly
36
Q

A new client joins a clinician’s grief group and tells everyone that his wife died and that he “just can’t get over it.” He says they were married 25 years and were planning on traveling around the world when she became ill. The man complains of not sleeping, eating very little, and not wanting to spend time with friends. The clinician is more likely to assign a DSM-5 diagnosis of Uncomplicated Bereavement to the man than a diagnosis of Major Depressive Disorder if:
Select one:

A. the man’s feeling of loss is accompanied by bouts of anxiety, a sense of emptiness, and intrusive thoughts of his wife and their relationship.
B. the man’s symptoms have dissipated in intensity over time and occur primarily when he thinks about his wife and their relationship.
C. the man has had persistent symptoms for less than two months.
D. the man’s wife died less than six months ago and he considers his reaction to her death to be normal.

A

Answer B is correct: Uncomplicated Bereavement is not a mental disorder but is included in the DSM-5 with Other Conditions That May Be a Focus of Clinical Attention. It involves a feeling of emptiness or loss that tends to decrease in intensity over days to weeks and is associated with thoughts or reminders of the deceased.

Answer A: See explanation for answer B.

Answer C: The DSM-5 does not specify a particular duration of symptoms as a requirement for a diagnosis of Uncomplicated Bereavement.

Answer D: Although the DSM-5 identifies viewing one’s reaction to the death as normal as an indicator of Uncomplicated Bereavement, it does not specify a particular interval of time since the death occurred as a requirement for its diagnosis.

The correct answer is: the man’s symptoms have dissipated in intensity over time and occur primarily when he thinks about his wife and their relationship.

How well did you know this?
1
Not at all
2
3
4
5
Perfectly
37
Q

Kegel exercises are used to:
Select one:

A. decrease performance anxiety.
B. treat insomnia.
C. establish a state of relaxation.
D. increase muscle tone.

A

Kegel exercises are used to restore muscle tone in the perineal area.

a. Incorrect See explanation for response d.
b. Incorrect See explanation for response d.
c. Incorrect See explanation for response d.
d. CORRECT Arnold Kegel developed the Kegel exercise to help women regain control over urination after childbirth. The exercise also helps increase sensitivity during intercourse.

The correct answer is: increase muscle tone.

How well did you know this?
1
Not at all
2
3
4
5
Perfectly
38
Q

Jacob, age 14, is being seen by a school clinician for repeated angry outbursts at school. He is persistently irritable, constantly argues with adults, has a history of behavioral referrals, and recently changed schools after being expelled for threatening a teacher. Jacob’s parents also report outbursts at home. He is failing in school, and his parents are thinking of sending him to a wilderness camp where he can “get some sense knocked into him.” The most likely DSM-5 diagnosis for Jacob is which of the following?
Select one:

A. Intermittent Explosive Disorder
B. Oppositional Defiant Disorder
C. Bipolar II Disorder
D. Disruptive Mood Dysregulation Disorder

A

Answer D is correct: Disruptive Mood Dysregulation Disorder involves severe, recurrent temper outbursts with a persistent irritable or angry mood between outbursts on most days.

Answer A: A persistent irritable mood between temper outbursts is not characteristic of Intermittent Explosive Disorder.

Answer B: Oppositional Defiant Disorder differs from Disruptive Mood Dysregulation Disorder primarily in terms of the severity, frequency, and chronicity of the temper outbursts, with Disruptive Mood Dysregulation Disorder having more severe, frequent, and chronic symptoms. When the criteria for both disorders are met, a diagnosis of Disruptive Mood Dysregulation Disorder is assigned.

Answer C: The essential feature of Bipolar II Disorder is hypomania (a distinct period of abnormally elevated, expansive, or irritable mood accompanied by increased activity or energy). Although people with Disruptive Mood Dysregulation Disorder are irritable, they do not exhibit the increased energy that is characteristic of Bipolar II Disorder. Also, the mood disturbance associated with Bipolar Disorder is episodic in nature, whereas the mood disturbance associated with Disruptive Mood Dysregulation Disorder is persistent.

The correct answer is: Disruptive Mood Dysregulation Disorder

How well did you know this?
1
Not at all
2
3
4
5
Perfectly
39
Q

Cataplexy is:
Select one:

A. a movement disorder in which involuntary muscle contractions produce repetitive twisting movements.
B. a sudden loss of muscle tone that can produce muscle weakness, loss of voluntary muscle control, or postural collapse.
C. a condition involving postural rigidity and a lack of response to external stimuli.
D. a loss of the ability to perform skilled, coordinated movements in the absence of motor or sensory deficits.

A

Cataplexy is associated with narcolepsy.

a. Incorrect This describes dystonia.
b. CORRECT Cataplexy is often precipitated by strong emotion and causes flaccid paralysis (loss of muscle tone).
c. Incorrect This describes catalepsy, which is a symptom of Schizophrenia and epilepsy and can be produced by hypnosis.
d. Incorrect This describes apraxia.

The correct answer is: a sudden loss of muscle tone that can produce muscle weakness, loss of voluntary muscle control, or postural collapse.

How well did you know this?
1
Not at all
2
3
4
5
Perfectly
40
Q

Tom, a 35-year-old television repairman, becomes extremely upset whenever his boss and co-workers say something unfavorable about his work and is very uncomfortable whenever he feels he is being watched at work. He reports having only one close friend who he says he is very afraid of losing. Tom rarely leaves the house except to go to work, and he tells you that one of the reasons is because he is not a very good driver and he fears he’ll get into a car accident. Based on this information, the best diagnosis for Tom is:
Select one:

A. Schizoid Personality Disorder.
B. Schizotypal Personality Disorder.
C. Avoidant Personality Disorder.
D. Dependent Personality Disorder.

A

Tom’s behavior is characterized by social discomfort and extreme sensitivity to negative evaluations.

a. Incorrect Although some of Tom’s symptoms are also suggestive of Schizoid Personality Disorder, this is not the best answer since Tom seems to desire close relationships (i.e., he says he is worried about losing his best friend). Also, people with Schizoid Personality Disorder appear indifferent to criticism.
b. Incorrect Schizotypal Personality Disorder entails behaviors that are similar to those associated with Schizophrenia but are less severe and transient.
c. CORRECT Based on the information given, the best diagnosis is Avoidant Personality Disorder, which entails sensitivity to criticism, avoidance of social activities, embarrassment, and distress at the inability to form close personal relationships.
d. Incorrect Dependent Personality Disorder is characterized by a pattern of dependent and submissive behaviors.

The correct answer is: Avoidant Personality Disorder.

How well did you know this?
1
Not at all
2
3
4
5
Perfectly
41
Q

The most common precursor of Bulimia Nervosa during adolescence is:
Select one:

A. academic failure.
B. dieting.
C. sexual abuse.
D. peer rejection.

A

A number of factors have been linked to Bulimia, and, unfortunately, the literature is not entirely consistent with regard to these factors.

a. Incorrect Although academic (and interpersonal) problems are associated with this disorder, they are less consistent predictors than dieting.
b. CORRECT L. K. G. Hsu notes that Bulimia and other eating disorders almost always begin with dieting (Eating disorders, New York, Guilford, 1993). He proposes that “dieting provides the entree into an eating disorder” (p. 76), especially when dieting is combined with other high-risk factors such as depression or social anxiety, low self-concept, and poor identity formation.
c. Incorrect See explanation above.

d. Incorrect See explanation above.
The correct answer is: dieting.

How well did you know this?
1
Not at all
2
3
4
5
Perfectly
42
Q

Epidemiological studies on rates of various mental disorders in urban and rural areas in the United States have found that:
Select one:

A. for most mental disorders, the rates are significantly higher in urban areas.
B. for most mental disorders, the rates are significantly higher in rural areas.
C. some disorders are more common in rural or in urban areas, but the differences in rates are usually not statistically significant.
D. some disorders are more common in rural or in urban areas, and the differences in rates are usually statistically significant.

A

This is a difficult question to answer since the reported rates of various mental disorders in rural and urban areas vary somewhat from study to study.

a. Incorrect See explanation for response c.
b. Incorrect See explanation for response c.
c. CORRECT Overall, the best conclusion that can be drawn is that there are few consistent (or statistically significant) differences in the rates of mental disorders in urban and rural areas. For example, a recent Healthcare for Communities (HCC) survey of 9,585 individuals living in rural and urban areas found no significant differences in the prevalence of the major types of mental illness (L. Tang et al., Report on the Survey Method for the Household Survey of Healthcare for Communities, 1997-1998, Los Angeles, Health Sciences Research Center, UCLA, 2001). This lack of significant differences between rates in rural and urban areas is consistent with the findings of the Epidemiologic Catchment Area Study and the National Comorbidity Survey.
d. Incorrect See explanation for response c.

The correct answer is: some disorders are more common in rural or in urban areas, but the differences in rates are usually not statistically significant.

How well did you know this?
1
Not at all
2
3
4
5
Perfectly
43
Q

In contrast to most organic forms of amnesia, Dissociative Amnesia usually:
Select one:

A. affects only memory for events that occurred before the trauma that precipitated the amnesia.
B. affects memory for events that occurred both before and after the trauma that precipitated the amnesia.
C. affects the ability to acquire new information from the time of the trauma that precipitated the amnesia to the present.
D. affects memory for events that occurred at the time of the trauma that precipitated the amnesia and for a circumscribed period following the trauma.

A

Dissociative Amnesia is a type of functional (psychogenic) amnesia that is often associated with a traumatic event and is characterized by an inability to remember important personal information with gaps in memory being related to the traumatic event.

a. Incorrect See explanation for response d.
b. Incorrect See explanation for response d.
c. Incorrect In most cases of Dissociative Amnesia, the individual does not have trouble acquiring new information once the trauma that precipitated the memory loss is over. In other words, the anterograde amnesia is usually related to the traumatic event and for events that occurred during a circumscribed period following the trauma.
d. CORRECT Memory loss in Dissociative Amnesia is most often anterograde - i.e., it typically involves the trauma and, in some cases, other events that occurred during a circumscribed period following the trauma. Retrograde amnesia (loss of memory for events prior to the trauma) is rare in Dissociative Amnesia; and, when it occurs, the loss of past memories is often reversed through hypnosis.

The correct answer is: affects memory for events that occurred at the time of the trauma that precipitated the amnesia and for a circumscribed period following the trauma.

How well did you know this?
1
Not at all
2
3
4
5
Perfectly
44
Q
Ella E. age 10, often says she doesn't want to go to school because she's afraid some of the children in her class will be "mean" to her. She seems overly concerned about her ability to perform well at school and in social situations and has trouble finishing her homework and other tasks because she keeps re-doing them because she wants them to be "perfect." Ella frequently says she has a headache and has trouble falling asleep at night. Her mother tries to keep Ella from watching the news because she's unwilling to go out of the house whenever there's a story about a local murder or accident. Based on these symptoms, the best diagnosis for Ella is:
Select one:

A. Separation Anxiety Disorder.
B. Agoraphobia.
C. Generalized Anxiety Disorder.
D. Specific Phobia.

A

This child is expressing excessive worry about multiple situations.

a. Incorrect A diagnosis of Separation Anxiety Disorer requires evidence of excessive anxiety concerning separation from those to whom the child is attached. This evidence is not provided in the description given in this question.
b. Incorrect The question does not indicate that the child has a fear of being in places where escape might be difficult or embarassing or where help might not be available if panic-like symptoms develop, which are the essential features of Agoraphobia.
c. CORRECT Generalized Anxiety Disorder involves excessive worry or anxiety about a number of events or activities that the person finds difficult to control. Children with this disorder are often perfectionistic, filled with self-doubt, and typically exhibit physical signs of anxiety (e.g., headaches, stomachaches).
d. Incorrect Specific Phobia is characterized by a marked and persistent fear of a specific object or situation. The child’s fears in this case are broader than this and, therefore, Generalized Anxiety Disorder is the better diagnosis.

The correct answer is: Generalized Anxiety Disorder.

How well did you know this?
1
Not at all
2
3
4
5
Perfectly
45
Q

Marlatt and Gordon’s (1985) relapse prevention model focuses on:
Select one:

A. situations antecedent to relapse.
B. the perceived consequences of relapse.
C. controlled drinking.
D. contingency management.

A

Marlatt and Gordon’s approach to the definition and treatment of addictions is classified as a social learning approach that combines behavioral and cognitive principles.

a. CORRECT According to Marlatt and Gordon, relapse is a failure to maintain a behavior change after treatment and is best avoided by identifying and dealing with its antecedents.
b. Incorrect See explanation for response a.
c. Incorrect See explanation for response a.
d. Incorrect See explanation for response a.

The correct answer is: situations antecedent to relapse.

How well did you know this?
1
Not at all
2
3
4
5
Perfectly
46
Q

For a person with Bipolar Disorder, the risk for suicide is greatest:
Select one:

A. when depressive symptoms are most severe.
B. when depressive symptoms have become less severe.
C. shortly after the onset of a depressive episode.
D. shortly after the onset of a manic episode.

A

There is some evidence that the risk for suicide is related to the severity of symptoms.

a. Incorrect See explanation for response b.
b. CORRECT According to some experts, the risk for suicide is greatest when depressive symptoms begin to lift. Apparently, this is because, at this time, suicidal ideation has not subsided and the individual has the increased energy and planning ability required to carry out a suicide plan. See, e.g., H. N. Holmes (ed.), Professional guide to diseases, Springhouse, PA, Springhouse, 2001.
c. Incorrect See explanation for response b.
d. Incorrect See explanation for response b.

The correct answer is: when depressive symptoms have become less sever

How well did you know this?
1
Not at all
2
3
4
5
Perfectly
47
Q

Which of the following is categorized as a “negative symptom” of Schizophrenia?
Select one:

A. avolition
B. loosening of associations
C. catatonia
D. perceptual disturbances

A

The symptoms of Schizophrenia are classified as positive or negative. Additional information about these symptoms is provided in the Abnormal Psychology chapter of the written study materials.

a. CORRECT Avolition is an inability to initiate and persist in action and is classified as a negative symptom.
b. Incorrect Loosening of associations is classified as a positive symptom.
c. Incorrect Catatonia is a type of disorganized behavior and is classified as a positive symptom.
d. Incorrect Perceptual disturbances (hallucinations and delusions) are positive symptoms.

The correct answer is: avolition

How well did you know this?
1
Not at all
2
3
4
5
Perfectly
48
Q

Longitudinal research conducted by Strauss and Carpenter (1991) has found that, for patients with Schizophrenia, prognosis with regard to future employment is best predicted by:
Select one:

A. frequency of social contacts.
B. symptom severity.
C. employment history.
D. duration of hospitalization.

A

Answer C is correct: This issue was investigated by W. Strauss and J. Carpenter in their longitudinal study of individuals with Schizophrenia [The prediction of outcome in Schizophrenia IV: Eleven-year follow-up of the Washington IPSS cohort, The Journal of Nervous and Mental Disease, 179(9), 517-525, 1991]. A key finding of their study was that the various domains of functioning are relatively independent - previous employment is the best predictor of current employment, past symptom severity is the best predictor of current symptom severity, etc.

The correct answer is: employment history.

How well did you know this?
1
Not at all
2
3
4
5
Perfectly
49
Q

During her first session, Jane, age 43, tells the clinician that she needs to work on anxiety and relationship issues. She says she divorced her second husband 11 months ago and has been dating a man she met at church for a month but thinks he’s going to “dump” her because of her physical problems. Jane has Type 2 diabetes, which is well-managed with medication. However, she complains of frequent stomach problems, severe headaches, and back pain and says she spends a lot of time researching diseases on the Internet to try to figure out what’s wrong with her besides diabetes. Jane has been evaluated by a physician who told her that, other than diabetes, she does not have any medical problems. What DSM-5 diagnosis should the clinician consider first?
Select one:

A. Somatic Symptom Disorder
B. Factitious Disorder
C. Illness Anxiety Disorder
D. Psychological Factors Affecting Other Medical Condition.

A

Answer A is correct: Jane’s symptoms are consistent with Somatic Symptom Disorder, which is included in the DSM-5 with Somatic Symptom and Related Disorders. The essential feature of this disorder is the presence of multiple somatic symptoms that cause distress or disruption in one’s daily life.

Answer B: Factitious Disorder is characterized by falsification of physical or psychological symptoms or induction of an illness or injury that is associated with an identified deception. No information given in the question suggests that Jane is faking or causing her symptoms.

Answer C: Illness Anxiety Disorder is characterized by excessive worry about being or becoming ill and usually does not include somatic symptoms or includes only symptoms of mild intensity.

Answer D: The diagnosis of Psychological Factors Affecting Other Medical Conditions is assigned when a person’s psychological problems affect the course of a medical condition or interfere with its treatment. Jane’s symptoms do not seem to be interfering with her management of her diabetes.

The correct answer is: Somatic Symptom Disorder

How well did you know this?
1
Not at all
2
3
4
5
Perfectly
50
Q

John is a 35-year-old graphic designer who is seeking help for his fear of public speaking. Due to a recent promotion, he is now required to make regular “pitches” to prospective clients. Previously, his work was mostly “behind the scenes,” and he’s afraid that he will “blow it” when making presentations and look incompetent and silly to his co-workers and the clients. John is considering asking to be demoted so that he won’t have to make presentations. He states that public speaking has been a problem for him since high school, and that he feels like he’s having a panic attack whenever he has to speak in front of a group. The most likely DSM-5 diagnosis for John is:
Select one:

A. Social Anxiety Disorder
B. Panic Disorder
C. Specific Phobia
D. Generalized Anxiety Disorder

A

Answer A is correct: John’s symptoms meet the diagnostic criteria for Social Anxiety Disorder, which is diagnosed when the individual experiences marked fear or anxiety of fear of scrutiny by others in one or more social situations that is out of proportion to the actual threat posed by the situation. Since John’s symptoms occur only when he speaks in public, the specifier “performance only” would be added to his diagnosis.

Answer B: Panic Disorder is characterized by sudden, unexpected panic attacks. John’s panicky feelings are clearly linked to a specific stressor (public speaking) and, therefore, are not unexpected.

Answer C: Specific Phobia is characterized by marked fear or anxiety about a specific object or situation; whereas, the fear or anxiety associated with Social Anxiety Disorder is related to a fear of scrutiny by others.

Answer D: Generalized Anxiety Disorder involves excessive anxiety and worry about multiple events or activities and the anxiety and worry are not necessarily due to a fear of scrutiny by others. John’s anxiety and worry are related to a single activity (public speaking) and to a fear of scrutiny by others, so Social Anxiety Disorder is the appropriate diagnosis.

The correct answer is: Social Anxiety Disorder

How well did you know this?
1
Not at all
2
3
4
5
Perfectly
51
Q

Although no single structural abnormality of the brain explains the majority of cases of Schizophrenia, _____________ has been most frequently found in patients with this disorder.
Select one:

A. enlargement of the frontal lobes
B. enlargement of the ventricles
C. atrophy of the cerebellum
D. atrophy of the right hemisphere

A

Several structural brain abnormalities have been linked to Schizophrenia, although no single abnormality has been found to underlie the majority of cases.

a. Incorrect See explanation for response b.
b. CORRECT A minority of patients (15 to 30%) exhibit enlargement of the lateral and third ventricles, which is the largest percent found to have any of the structural abnormalities identified so far. Other abnormalities found in an even smaller percent of patients include smaller-than-normal hippocampus and amygdala.
c. Incorrect See explanation for response b.
d. Incorrect See explanation for response b.

The correct answer is: enlargement of the ventricles

How well did you know this?
1
Not at all
2
3
4
5
Perfectly
52
Q

Approximately 80% of children with ADHD continue to have symptoms of the disorder as adolescents. However, in adolescence, symptoms of ____________ typically decrease.
Select one:

A. hyperactivity
B. inattention
C. hyperactivity and inattention
D. inattention and impulsivity

A

The studies have found that the majority of children with ADHD continue to have symptoms in adolescence but that the symptoms change somewhat.

a. CORRECT Hyperactivity typically decreases in adolescence, while impulsivity (especially verbal and emotional impulsivity) and inattention continue. In addition, during adolescence, ADHD is often masked by other problems such as oppositionality, learning difficulties, low self-esteem, and deficits in social skills. See, e.g., G. Kewley, Attention deficit hyperactivity disorder: What can teachers do?, New York, Routledge, 2011.
b. Incorrect See explanation for response a.
c. Incorrect See explanation for response a.
d. Incorrect See explanation for response a.

The correct answer is: hyperactivity

How well did you know this?
1
Not at all
2
3
4
5
Perfectly
53
Q

A white middle-aged man is at highest risk for suicide if he:
Select one:

A. has been divorced for 10 years, overtly expresses aggression and hostility, and is unwilling to talk about his suicidal feelings.
B. has been divorced for 6 years, exhibits a constriction of affect, and has a peptic ulcer.
C. has been divorced for 3 years, overtly expresses aggression and hostility, and is unwilling to talk about his suicidal feelings.
D. has been divorced for 1 year, exhibits a constriction of affect, and has a peptic ulcer.

A

Knowing that divorce is related to increased risk for suicide, with the risk being greatest during the first year, would have helped you answer this question correctly.

a. Incorrect Unwillingness to talk about suicidal feelings is more characteristic of older adults than middle-aged adults.
b. Incorrect See explanation for response d.
c. Incorrect See explanation for response d.
d. CORRECT The more recent the divorce, the greater the risk for suicide. A higher risk in adulthood is also associated with constriction of affect (versus overt expression) and the presence of certain illnesses including a peptic ulcer. See, for example, S. J. Blumenthal and D. J. Kupfer, Suicide over the life cycle: Risk factors, assessment, and treatment of suicidal patients, American Psychiatric Press, Washington, DC, 1990.

The correct answer is: has been divorced for 1 year, exhibits a constriction of affect, and has a peptic ulcer.

How well did you know this?
1
Not at all
2
3
4
5
Perfectly
54
Q

Which of the following is not an empirically supported treatment for primary insomnia?
Select one:

A. stimulus control therapy
B. sleep restriction
C. cognitive therapy
D. habit reversal training

A

A number of non-pharmacological treatments have been identified as empirically validated treatments for primary insomnia. For a review of the research, see C. M. Morin et al., Psychological and behavioral treatment of insomnia: Update of the recent evidence (1998-2004), Sleep, 29(11), 1398-1414, 2006.

a. Incorrect Stimulus control therapy is a commonly used treatment for insomnia and has been confirmed by the research to be an effective approach. It incorporates several strategies that help make the individual’s bed and bedroom serve as cues for sleep.
b. Incorrect Sleep restriction has also been found to be an effective treatment for insomnia. When using this technique, the amount of time allotted for sleep is initially restricted to create a “sleep debt” and the amount of time spent sleeping is then gradually lengthened.
c. Incorrect Cognitive therapy for insomnia involves identifying and replacing dysfunctional thoughts and attitudes that underlie the person’s sleep problems.
d. CORRECT Habit reversal training is used to eliminate habits and other undesirable repetitive behaviors including tics, trichotillomania, and stuttering.

The correct answer is: habit reversal training

How well did you know this?
1
Not at all
2
3
4
5
Perfectly
55
Q

Recent research investigating treatments for Generalized Anxiety Disorder suggests that the most effective intervention is:
Select one:

A. EMDR.
B. systematic desensitization.
C. stress inoculation training.
D. cognitive-behavioral therapy.

A

A number of well-designed studies on interventions for Generalized Anxiety Disorder confirm that the best treatment is cognitive or cognitive-behavioral in nature.

a. Incorrect See explanation for response d.
b. Incorrect See explanation for response d.
c. Incorrect See explanation for response d.
d. CORRECT The studies have consistently found that a multicomponent cognitive or cognitive-behavioral approach is most effective for Generalized Anxiety Disorder. See, e.g., D. H. Barlow, J. Esler, and A. Vitali, Psychosocial treatments for panic disorders, phobias, and generalized anxiety disorder, in P. E. Nathan and J. M. Gorman (eds.), A guide to treatments that work, New York, Oxford University Press, 1998.

The correct answer is: cognitive-behavioral therapy.

How well did you know this?
1
Not at all
2
3
4
5
Perfectly
56
Q

In contrast to Korsakoff’s Syndrome, Alzheimer’s disease is associated with:
Select one:

A. more severe anterograde amnesia.
B. greater confabulation.
C. less severe working memory impairment.
D. a wider range of cognitive deficits.

A

Korsakoff’s syndrome and Alzheimer’s disease share several symptoms.

a. Incorrect This response is imprecise and not the best one – e.g., how is anterograde amnesia measured and what stage of Alzheimer’s is being compared to Korsakoff’s syndrome? Also, an essential feature of Korsakoff’s syndrome is severe anterograde amnesia.
b. Incorrect Confabulation is an essential feature of Korsakoff’s syndrome.
c. Incorrect Alzheimer’s disease is actually associated with more severe working memory deficits than Korsakoff’s syndrome is.
d. CORRECT Korsakoff’s syndrome is characterized by disproportionate memory deficits relative to other types of cognitive impairment [M. D. Kopelman, The Korsakoff Syndrome, British Journal of Psychiatry, 166(2), 154-173, 1995]. In contrast, Alzheimer’s disease involves memory impairment plus severe impairments in other aspects of cognitive functioning.

The correct answer is: a wider range of cognitive deficits.

How well did you know this?
1
Not at all
2
3
4
5
Perfectly
57
Q

Dr. A., a psychologist, provides office space and secretarial services to Dr. B., a newly-licensed psychologist, in exchange for 25% of the fee that Dr. B. collects from his clients. Dr. A. begins referring clients to Dr. B. and charges him 35% for the referred clients. This is:
Select one:

A. unethical because the arrangement violates APA’s prohibition against “fee splitting.”
B. unethical because the percentage that Dr. A. is charging Dr. B. is exploitative.
C. ethical only if the larger fee for referred clients reflects increased costs to Dr. A. for making referrals to Dr. B.
D. ethical because the arrangement does not violate the provisions of the Ethics Code regarding referral fees.

A

Although referral fees are not entirely prohibited by the Ethics Code, there are some restrictions.

a. Incorrect See explanation for response c.
b. Incorrect See explanation for response c.
c. CORRECT It is ethical to pay a fee for a referral when the fee reflects the person’s expenses for making the referral. However, charging more for referred patients than non-referred patients when there are no additional costs would be unacceptable.
d. Incorrect See explanation for response c.

The correct answer is: ethical only if the larger fee for referred clients reflects increased costs to Dr. A. for making referrals to Dr. B.

How well did you know this?
1
Not at all
2
3
4
5
Perfectly
58
Q

Dr. Weiner, a psychologist, is conducting a research study and obtains subjects for his study by placing advertisements in a local newspaper. Dr. Weiner informs potential subjects about all aspects of the study and tells them that they should not agree to participate unless they are willing to stay for its duration (six sessions over a six-week period), since once they sign up, they will not be permitted to withdraw from the study. According to APA’s Ethics Code, the proposed arrangement is:
Select one:

A. clearly ethical.
B. ethical as long as the withdrawal restriction is included in the informed consents that all subjects must sign.
C. ethical as long as the withdrawal restriction has been determined to be crucial to the study.
D. unethical.

A

A researcher must respect the participants’ right to withdraw at any time for any reason in the course of a study.

a. Incorrect A withdrawal restriction such as the one described in the question is unethical.
b. Incorrect Under no conditions is such a withdrawal restriction ethical.
c. Incorrect Under no conditions is such a withdrawal restriction ethical.
d. CORRECT This answer is in line with Standard 8.02 of the APA’s Ethics Code.

The correct answer is: unethical.

How well did you know this?
1
Not at all
2
3
4
5
Perfectly
59
Q

A research psychologist designs a study that involves hospitalized psychiatric patients who have legal guardians. Before beginning the study, he obtains signed consents from the legal guardians. The psychologist has acted:
Select one:

A. unethically since he has breached the patients’ confidentiality.
B. ethically as long as the patients are allowed to withdraw from the study at any time.
C. ethically as long as he has also obtained assent from the patients.
D. ethically, since all a psychologist must obtain in this situation is the informed consent of legal guardians.

A

This issue is addressed in Standard 3.10 of the APA’s Ethics Code.

a. Incorrect Ethical guidelines require that informed consent be obtained from the patients’ legal guardians in this situation.
b. Incorrect Standard 8.02(a) dictates that participants have the right to withdraw from research at any time. However, this is not the best answer since it does not address the question’s underlying ethical issue; i.e., the need for psychologists to obtain assent from participants who are legally incapable of giving informed consent.
c. CORRECT This answer is consistent with the requirements of the Ethics Code – i.e., when research participants are not legally capable of giving informed consent (either because they are mentally incompetent or minors), consent must be obtained from their legal guardians, and the participants should be provided with understandable information and their assent to participate should be obtained.
d. Incorrect See explanation for response c.

The correct answer is: ethically as long as he has also obtained assent from the patients.

How well did you know this?
1
Not at all
2
3
4
5
Perfectly
60
Q

You are working in a correctional facility and are asked to evaluate a prisoner to determine his eligibility for parole. In this situation, you should:
Select one:

A. conduct the evaluation as requested since it is part of your job.
B. conduct the evaluation only if you believe it will serve a useful dispositional function.
C. conduct the evaluation after reminding the prisoner that anything he says can be shared with prison authorities.
D. refuse to conduct the evaluation unless it has been court-ordered.

A

Relatively little has been written about ethical considerations in correctional facilities, so few guidelines are available. However, APA published recommendations in a 1978 issue of the American Psychologist that apply to this situation (Report of the task force on the role of the psychologist in the criminal justice system, American Psychologist, 33, 1099-1133, 1978).

a. Incorrect See explanation for response b.
b. CORRECT Recommendation 3 of the above-cited document states that “Other than for legitimate research purposes, psychological assessments of offenders should be performed only when the psychologist has a reasonable expectation that such assessments will serve a useful therapeutic or dispositional function.”
c. Incorrect APA recommends that the “ideal level of confidentiality of therapeutic services in criminal justice settings should be the same as the level of confidentiality that exists in voluntary non-institutional settings.” Although there certainly are exceptions to confidentiality in prison settings that would not apply elsewhere, it’s not true that a prisoner has no right to confidentiality, which this response implies.
d. Incorrect See explanation above.

The correct answer is: conduct the evaluation only if you believe it will serve a useful dispositional function.

How well did you know this?
1
Not at all
2
3
4
5
Perfectly
61
Q

You receive a letter from the attorney of a former client requesting that you send test information to her from the client’s file. The requested information will be used in a court case and includes the test questions, the client’s responses to those questions, and the client’s test scores. The attorney’s letter is accompanied by a signed release from the client. To be consistent with ethical requirements, you should:
Select one:

A. release all test information as requested.
B. release only a summary of the client’s test information.
C. release the clients responses and test scores but not the test questions.
D. refuse to release any information to anyone other than a mental health professional.

A

Answer C is correct: Standards 9.04 and 9.11 of the APA’s Ethics Code distinguish between test data and test material and state that test data may be provided to an attorney or other person listed in a client’s release but that test material should not be released because doing so may constitute a breach of test integrity and security. As defined in Standard 9.11, test material includes “manuals, instruments, protocols, and test questions or stimuli.” Of the answers given, this one is most consistent with the provisions of the APA’s Ethics Code and with Paragraph IV.11 of the Canadian Code of Ethics for Psychologists.

The correct answer is: release the clients responses and test scores but not the test questions.

How well did you know this?
1
Not at all
2
3
4
5
Perfectly
62
Q

Dr. K. has just started working at a community mental health center. Several of his new clients belong to an ethnic minority group that he has not had prior experience working with. Dr. K. should:
Select one:

A. refuse to see the clients because he has not had adequate experience.
B. see the clients only if he has completed appropriate coursework on cross-cultural counseling.
C. ask the clients if they desire a referral and, if so, refer them to another therapist.
D. see the clients but obtain appropriate supervision or consultation.

A

During the course of his/her career, a therapist will ordinarily want to acquire new skills and will want to expand his/her practice to include new kinds of clients. This is particularly true for Dr. K. since he is working at a clinic where some clients are apparently members of minority groups.

a. Incorrect See explanation for response d.
b. Incorrect See explanation for response d.
c. Incorrect See explanation for response d.
d. CORRECT The ethical guidelines recommend that therapists seek appropriate supervision or consultation when working with clients who belong to an unfamiliar group or population. In some situations, a referral would be an appropriate course of action, but supervision or consultation seems best in this case.

The correct answer is: see the clients but obtain appropriate supervision or consultation.

How well did you know this?
1
Not at all
2
3
4
5
Perfectly
63
Q

During the third therapy session, a client confides to her psychologist that she was once a sex worker. The psychologist is offended by this type of behavior, feels incapable of dealing with the client objectively, and terminates therapy immediately. His action is:
Select one:

A. unethical because it represents abandonment of the client.
B. ethical because he could not deal with the client objectively.
C. unethical because a therapist should not allow personal opinions to interfere with professional activities.
D. ethical since he has seen the client in therapy for only a few sessions.

A

During the course of therapy, a therapist may occasionally feel incapable of providing a client with the services the client needs. When such situations arise the therapist should handle them in an appropriate, professional manner. If the therapist feels that he/she must terminate therapy, he/she should do so with extreme care. The therapist should be careful not to “blame” the client, and an appropriate referral should be made.

a. CORRECT This situation clearly represents abandonment of the client. Although a therapist might feel it necessary to terminate therapy due to personal beliefs, termination must be handled in a professional, sensitive manner. A therapist must evaluate his/her feelings carefully, and arrange for a referral if necessary. Abrupt termination is not appropriate.
b. Incorrect Although termination might have been appropriate in this situation, the therapist did not handle the termination properly.
c. Incorrect Although it is important that therapists remain objective, a therapist cannot always prevent his/her personal feelings from affecting therapy. When these feelings interfere with the quality of the therapy, the therapist needs to take action necessary to protect the interests of the client.
d. Incorrect The situation described was not handled appropriately and clearly constitutes an ethical violation.

The correct answer is: unethical because it represents abandonment of the client.

How well did you know this?
1
Not at all
2
3
4
5
Perfectly
64
Q

A psychologist in private practice who has been sued once in the past by a client for malpractice and who fears being sued again decides to limit his practice to clients whose problems are not highly associated with higher rates of malpractice claims. This is:
Select one:

A. unethical if he is competent to treat these clients.
B. ethical only if he is willing to see these clients in emergencies and other unusual circumstances.
C. ethical as long as he makes appropriate referrals.
D. ethical as long as he makes his policy clear at the outset of treatment.

A

Although you may feel the therapist’s decision is “not quite right,” therapists are not required to see patients they do not want to see.

a. Incorrect See explanation for response c.
b. Incorrect See explanation for response c.
c. CORRECT This issue is addressed by R. I. Simon in Clinical psychiatry and the law (American Psychiatric Press, Washington, DC, 1992). Although Simon is discussing the obligations of psychiatrists, the same principles apply to psychologists: Self-employed mental health professionals do not have a legal or ethical duty to enter into professional relationships. While the psychologist’s decision may seem unfair, he is free to serve the clients he chooses. He should, however, make appropriate referrals. (Note that the Ethics Code does require psychologists not to discriminate on the basis of race, gender, or any other basis proscribed by law. Litigious clients do not fall into this category, however.)
d. Incorrect See explanation for response c.

The correct answer is: ethical as long as he makes appropriate referrals.

How well did you know this?
1
Not at all
2
3
4
5
Perfectly
65
Q

Dr. Tartuffe is conducting a survey to obtain the information needed to compare therapists who have and have not had a sexual relationship with a client. The results of his survey will most likely indicate that therapists who say they have been sexually involved with a client:
Select one:

A. are also more likely to have been sexually involved with their own supervisor.
B. are also more likely to have had non-sexual dual relationships with their clients.
C. are also more likely to make use of therapeutic touch in therapy.
D. are not distinguishable from those who have not been sexually involved with a client in terms of sex with their own supervisor, non-sexual dual relationships, or the use of therapeutic touch.

A

The characteristics of therapists who have and have not had sexual relationships with their clients were assessed by a survey conducted by D. H. Lamb and S. J. Catanzaro (Sexual and nonsexual boundary violations involving psychologists, clients, supervisees, and students: Implications for professional practice, Professional Psychology: Research and Practice, 29(5), 498-503, 1998).

a. Incorrect See explanation for response b.
b. CORRECT Lamb and Catanzaro found that psychologists who had sex with their clients were no more likely to have had sex with their supervisors than psychologists who did not have sex with their clients. However, psychologists reporting sexual relationships with their clients also reported more nonsexual dual relationships. (The issue of therapeutic touch was not examined in the survey, and there is no evidence to support response c.)
c. Incorrect See explanation for response b.
d. Incorrect See explanation for response b.

The correct answer is: are also more likely to have had non-sexual dual relationships with their clients.

How well did you know this?
1
Not at all
2
3
4
5
Perfectly
66
Q

You have been seeing a couple in therapy for six months. The wife calls you and says she wants to start seeing you for individual sessions also. You should:
Select one:

A. bring up this possibility in the next couples session.
B. tell her you cannot see her individually until couples therapy is over.
C. refer her to another therapist for individual therapy.
D. make an appointment with her to determine if her problem warrants individual therapy.

A

Standard 10.02 of the Ethics Code makes it clear that, in couples therapy, psychologists must avoid potentially conflicting roles. One way to ensure that conflicts do not occur is to clarify one’s policies and responsibilities. It may or may not be the psychologist’s policy to provide individual therapy while also providing conjoint therapy, and this should have been clarified at the onset of therapy. Also, a referral may be appropriate, but the issue should first be discussed in conjoint therapy.

a. CORRECT Bringing up the issue in the next couples therapy session would be the best initial course of action.
b. Incorrect See explanation above.
c. Incorrect See explanation above.
d. Incorrect See explanation above.

The correct answer is: bring up this possibility in the next couples session.

How well did you know this?
1
Not at all
2
3
4
5
Perfectly
67
Q

Dr. Raj provided treatment to a client in State X where Dr. Raj resides and works. Treatment ended six months ago and the client has since moved to State Y. Dr. Raj receives a subpoena from the client’s attorney who also resides and works in State Y. The subpoena requires Dr. Raj to provide testimony at a trial involving the former client. Dr. Raj:
Select one:

A. is not required to appear since the subpoena is from another state.
B. must appear and testify as requested as long as the client has signed a release of information.
C. must appear as long as the subpoena was issued by the court.
D. should contact an attorney to determine if he must appear as requested.

A

This is really a question of law and depends on applicable laws regarding jurisdiction. So, the best thing to do would be to consult a lawyer to determine whether it is necessary to comply with the subpoena.

a. Incorrect See explanation for response d.
b. Incorrect See explanation for response d.
c. Incorrect See explanation for response d.
d. CORRECT In his book on legal matters, R. Woody concludes that the appropriate course of action in this type of situation would be to consult an attorney to determine the legal authority of the subpoena (Legally safe mental health practice: Psycholegal questions and answers, Madison, CT, Psychosocial Press, 1997).

The correct answer is: should contact an attorney to determine if he must appear as requested.

How well did you know this?
1
Not at all
2
3
4
5
Perfectly
68
Q

Dr. Lane, a psychologist, is conducting a research study on peer pressure, and the study’s participants will be 120 fifth-grade students. The study will involve a 30-minute interview with each student and observation of the students during their lunch hour. Before beginning the study, Dr. Lane should:
Select one:

A. obtain an informed consent from each student.
B. obtain an informed consent from each student’s parent.
C. obtain an informed consent from appropriate school personnel.
D. obtain the assent of each student and an informed consent from each student’s parent.

A

Standard 3.10(b) of the APA’s Ethics Code is relevant to this situation.

a. Incorrect See explanation for response d.
b. Incorrect See explanation for response d.
c. Incorrect See explanation for response d.
d. CORRECT This answer is consistent with Standard 3.10(b), which states: “For persons who are legally incapable of giving informed consent, psychologists nevertheless (1) provide an appropriate explanation, (2) seek the individual’s assent, (3) consider such persons’ preferences and best interests, and (4) obtain appropriate permission from a legally authorized person, if such substitute consent is permitted or required by law.” This requirement applies to minors and to adults who have been declared legally incompetent.

The correct answer is: obtain the assent of each student and an informed consent from each student’s parent.

How well did you know this?
1
Not at all
2
3
4
5
Perfectly
69
Q

An undergraduate student is asked by a professor to assist in a new research project. The student will help with the literature review and data analysis. According to the Ethics Code, when the research is published, the student should, at a minimum:
Select one:

A. be listed as a co-author.
B. be mentioned in an introductory statement or a footnote.
C. be mentioned in the abstract.
D. receive a letter of recommendation from the professor.

A

The Ethics Code mandates that contributors to research be given appropriate credit.

a. Incorrect The researcher has the option to list the student as a co-author but this is not required.
b. CORRECT At a minimum, the student should be acknowledged for his/her assistance in a footnote or introductory statement.
c. Incorrect The abstract is not an appropriate section to recognize research assistants. An abstract provides the reader with a brief summary of the research and its results.
d. Incorrect Although it is possible that the student may request a letter of recommendation, the teacher is not ethically bound to comply with the students’ wishes because of his/her assistance in the research project.

The correct answer is: be mentioned in an introductory statement or a footnote.

How well did you know this?
1
Not at all
2
3
4
5
Perfectly
70
Q

An employee is receiving counseling from a psychologist through his company’s employee assistance program. In this situation, the psychologist can let the employee’s supervisor know that the employee is receiving treatment:
Select one:

A. under no circumstance.
B. only if the employee was referred to the program by the supervisor.
C. as long as the employee has signed an authorization for release of confidential information.
D. as long as no other information about the treatment is given to the supervisor.

A

Confidentiality is a critical issue in employee assistance programs (EAPs). Under normal circumstances, information about an employee’s condition or treatment is not revealed without the employee’s consent.

a. Incorrect See explanation for response c.
b. Incorrect See explanation for response c.
c. CORRECT Confidentiality may be breached in the context of EAPs in those situations that it can be breached in other contexts (e.g., when a client has signed an authorization or is believed to be a danger to him/herself or to others).
d. Incorrect See explanation for response c.

The correct answer is: as long as the employee has signed an authorization for release of confidential information.

How well did you know this?
1
Not at all
2
3
4
5
Perfectly
71
Q

The only time a client is able to see you is during her lunch hour, which requires her session to be limited to 30 minutes because of the distance she must travel to get to your office. When you bill her insurance company, however, you bill for the entire hour. This is:
Select one:

A. ethical since the shorter sessions are being provided at the client’s request.
B. ethical since your policy is to always bill clients for an entire hour.
C. unethical.
D. a legal, but not an ethical, issue.

A

The action described in this question represents insurance fraud which is both illegal and unethical.

a. Incorrect See explanation for response c.
b. Incorrect See explanation for response c.
c. CORRECT This is the best answer of those given. Providing any inaccurate information to the insurance company would represent insurance fraud.
d. Incorrect See explanation for response c.

The correct answer is: unethical.

How well did you know this?
1
Not at all
2
3
4
5
Perfectly
72
Q

Dr. Greene, a psychologist in private practice, sets his clients’ fees according to their ability to pay as determined by their yearly income. As a result, some of his clients are paying a very low fee or no fee, while others are paying a fee that is considerably above-average for the location of his practice. Dr. Greene’s fee policy is:
Select one:

A. illegal in most states and clearly unethical.
B. legal in most states but unethical.
C. both legal and ethical.
D. ethical but illegal in most states.

A

Sliding scale fees are legally and ethically acceptable as long as the fee arrangement is made clear from the onset of therapy.

a. Incorrect Sliding scale fees are not unethical and illegal.
b. Incorrect Sliding scale fees are not unethical.
c. CORRECT Sliding scale fees are a common practice and are both legal and ethical.
d. Incorrect Sliding scale fees are not illegal.

The correct answer is: both legal and ethical.

How well did you know this?
1
Not at all
2
3
4
5
Perfectly
73
Q

As defined in the APA’s Ethics Code, uninvited in-person solicitation of business by a psychologist is:
Select one:

A. acceptable as long as the solicitation is not coercive.
B. acceptable when it involves providing disaster or community outreach services.
C. acceptable as long as the person’s “best interests” are of paramount concern.
D. never acceptable.

A

The uninvited in-person solicitation of business is addressed in Standard 5.06 of the APA’s Ethics Code.

a. Incorrect See explanation for response b.
b. CORRECT Standard 5.06 generally prohibits psychologists from engaging in uninvited in-person solicitation of business. However, it also notes that the prohibition does NOT preclude psychologists from “providing disaster or community outreach services.”
c. Incorrect See explanation for response b.
d. Incorrect See explanation for response b.

The correct answer is: acceptable when it involves providing disaster or community outreach services.

How well did you know this?
1
Not at all
2
3
4
5
Perfectly
74
Q

A psychologist is hired by a defense attorney to evaluate a defendant to determine the appropriateness of the insanity defense. During the first meeting with the defendant, the psychologist explains the purpose of the evaluation. The defendant says he doesn’t want to cooperate with the evaluation because of the limits on confidentiality. The psychologist should:
Select one:

A. obtain a court order before evaluating the defendant.
B. obtain as much information from the defendant as possible and explain to the court why the evaluation is not complete.
C. explain to the defendant that privilege is waived in this situation and that the evaluation is in his best interests.
D. postpone the evaluation and notify the defense attorney of the defendant’s concerns.

A

This issue is addressed in the Specialty Guidelines for Forensic Psychology.

a. Incorrect See explanation for response d.
b. Incorrect See explanation for response d.
c. Incorrect See explanation for response d.
d. CORRECT This response is most consistent with “common sense” and with the provisions of Paragraph 6.03.01 of the Specialty Guidelines.

The correct answer is: postpone the evaluation and notify the defense attorney of the defendant’s concerns.

How well did you know this?
1
Not at all
2
3
4
5
Perfectly
75
Q

Dr. Bernard is designing a longitudinal study on drug use in high schools. Since the study is longitudinal, it will be necessary to maintain records containing participants’ identities. A colleague of Dr. Bernard’s reminds him of a similar situation in which the investigator was subpoenaed to appear in court to testify against a participant who had been involved in illegal activities. The investigator refused to testify in order to maintain the participant’s confidentiality and was imprisoned. What should Dr. Bernard do in order to avoid a similar dilemma:
Select one:

A. if subpoenaed by the court, testify, since legal requirements override ethical considerations.
B. if subpoenaed by the court, testify only after obtaining the consent of the participant.
C. warn participants prior to participation in the study about the possibility of a breach of confidentiality and obtain informed consents.
D. design the study so that the names of participants are maintained in a separate file by a third party.

A

Standard 8.02(a) of APA’s Ethics Code states that psychologists “… psychologists inform participants about … reasonably foreseeable factors that may be expected to influence their willingness to participate such as potential risks, discomfort, or adverse effects … limits of confidentiality.”

a. Incorrect To some degree, this answer correctly describes a psychologist’s legal obligations in this situation. After being subpoenaed by the court, a psychologist usually asserts privilege. If privilege is not recognized, the psychologist usually must testify in order to avoid being held in contempt of court. However, this is not the best answer in that it does not deal with how the psychologist can “avoid this dilemma.” If the psychologist testifies, he has fulfilled his legal obligation without having dealt with his ethical obligation regarding confidentiality.
b. Incorrect Legally, a psychologist who has been subpoenaed by the court must testify (if the court does not recognize privilege) whether or not he has the consent of his client. Thus, this course of action would not resolve the dilemma between the psychologist’s legal and ethical obligations.
c. CORRECT This is most in accord with the above-cited Standard, and it represents a balance between the psychologist’s legal and ethical obligations. If he informs participants of the limits of confidentiality, they will give their consent to participate with full awareness of the possibility that the psychologist or his files might be subpoenaed.
d. Incorrect This course of action does not reduce the possibility that either the records or the psychologist will be subpoenaed.

The correct answer is: warn participants prior to participation in the study about the possibility of a breach of confidentiality and obtain informed consents.

How well did you know this?
1
Not at all
2
3
4
5
Perfectly
76
Q

The Publication Manual of the American Psychological Association states that some abbreviations do not require explanation in text. These include all of the following except:
Select one:

A. IQ.
B. HIV.
C. APA.
D. ESP.

A

The Publication Manual states that abbreviations that appear as word entries in the Webster’s Collegiate Dictionary (i.e., those without the label “abbr”) do not require explanation in text.

a. Incorrect See explanation for response c.
b. Incorrect See explanation for response c.
c. CORRECT You may have been able to recognize this as the correct answer as long as you recalled that “APA” can stand for several organizations including the American Psychological Association and the American Psychiatric Association. Therefore, you would need an explanation the first time that it’s used. The abbreviations listed in the other answers are provided in the Publication Manual as examples of abbreviations that do not require an explanation.
d. Incorrect See explanation for response c.

The correct answer is: APA.

How well did you know this?
1
Not at all
2
3
4
5
Perfectly
77
Q

A new client says she wants you to give her an IQ test so that she can use the results to join MENSA. You administer the test, but she obtains an IQ score of only 105. The client asks you to give her the test results so that she can destroy them, and she informs you that she is going to be retested by another psychologist. You should:
Select one:

A. give the original record of the test results to the client as requested.
B. destroy the original record of the test results yourself.
C. tell the client you will forward the original record to the other psychologist.
D. give the client a summary of the test results and retain the original record in your files.

A

This issue is addressed by R. Woody in Legally safe mental health practice: Psycholegal questions and answers, Madison, CT, Psychosocial Press, 1997. Woody notes that “No records, be they test results or otherwise, should ever be destroyed simply because one person, be it client or therapist, does not like the content” (p. 68).

a. Incorrect See explanation for response d.
b. Incorrect See explanation for response d.
c. Incorrect See explanation for response d.
d. CORRECT The client has the right to information about her test results, but the record itself belongs to the therapist, who has a “limited property right” in the document. In addition, records are to protect both the client and the therapist, and destroying a record at the request of the client would not necessarily be in the best interests of either the client or therapist.

The correct answer is: give the client a summary of the test results and retain the original record in your files.

How well did you know this?
1
Not at all
2
3
4
5
Perfectly
78
Q

A psychologist has been seeing a couple in therapy for six months and they have recently decided to get a divorce. The psychologist is asked by the husband to testify on his behalf in a court custody hearing. The psychologist should:
Select one:

A. refuse to testify.
B. agree to testify as long as she does not reveal confidential information about the wife.
C. agree to testify only if the wife gives consent for her to do so.
D. agree to testify only if she will be testifying for both parties.

A

This is fairly straightforward. When working with a couple, the couple is considered the client and the welfare of the client must always come first.

a. CORRECT As a general rule, a therapist should not provide services to one member of a couple when the couple has been the client. Even if the wife consents, it is not possible to foresee a situation that might put her welfare in jeopardy if the therapist testifies on behalf of the husband (response c). The action described in response d could similarly put the therapist in a very awkward position and jeopardize the welfare of one or both of the parties.
b. Incorrect See explanation for response a.
c. Incorrect See explanation for response a.
d. Incorrect See explanation for response a.

The correct answer is: refuse to testify.

How well did you know this?
1
Not at all
2
3
4
5
Perfectly
79
Q

Which of the following would a managed care organization consider most useful for reducing its risk for legal liability?
Select one:

A. capitation
B. behavioral health “carve outs”
C. credentialing of providers
D. utilization review

A

Of the procedures listed in the responses, only one would be likely to reduce a managed health care organization’s risk for legal liability.

a. Incorrect Capitation refers to contracting for a set of services on the basis of a predetermined fee. It reduces the organization’s costs but does not protect it from legal liability.
b. Incorrect Behavioral health “carve outs” involve contracting behavioral health care services that are separate from the services provided for other health care. It is not associated with risk management.
c. CORRECT Credentialing providers helps ensure that providers are competent to provide services and thereby reduces the risk for legal liability (i.e., it contributes to risk management).
d. Incorrect Utilization review is another cost containment technique and may involve denying payment for services that are not deemed necessary by the managed health care organization.

The correct answer is: credentialing of providers

How well did you know this?
1
Not at all
2
3
4
5
Perfectly
80
Q

The attorney of a former client of yours requests that you provide her with test data and other information included in the client’s file. You should:
Select one:

A. provide the information since the request comes from the client’s attorney.
B. provide the information as requested only if you feel it is in the best interests of the client.
C. provide the information as requested only if the client has signed a release.
D. refuse to provide the information unless the client has signed a release and you believe the attorney is qualified to interpret the information.

A

Release of client information to a third party is addressed in Ethics Code Standard 9.04.

a. Incorrect See explanation for response c.
b. Incorrect The best interests of the client are certainly of concern, but this is not the best answer because it overlooks the client’s confidentiality.
c. CORRECT Standard 9.04 states that psychologist may withhold test data “to protect a client/patient or others from substantial harm” but that does not seem relevant to this situation. Therefore, the psychologist should release the test data as requested once the client has signed a release.
d. Incorrect See explanation for response c.

The correct answer is: provide the information as requested only if the client has signed a release.

How well did you know this?
1
Not at all
2
3
4
5
Perfectly
81
Q

The reviewer for a managed health care organization decides that additional services for one of your clients are not medically necessary. You strongly disagree with her decision because you believe your client’s condition is likely to worsen without additional therapy sessions. The client cannot afford to pay for therapy sessions himself. To act ethically and to minimize your legal liability in this situation, you should:
Select one:

A. carefully explain to the client the reason for termination and have him sign a waiver of responsibility.
B. provide the client with the names of several therapists who you know provide services at a low fee.
C. see the client at no fee for several sessions before terminating therapy.
D. file an appeal with the managed health care organization.

A

An important aspect of managed health care are the ethical and legal dilemmas that it can create.

a. Incorrect This action would not be ethical and basically constitutes abandonment of the client.
b. Incorrect Although a referral might be the action that is eventually taken, in most cases, it would not be the best first course of action from either an ethical or legal perspective.
c. Incorrect If it becomes necessary to end the therapeutic relationship, providing the client with free sessions during the termination period would be an ethical course of action. However, this is not the best answer of those given.
d. CORRECT In his discussion of “Legal Liability and Managed Care,” P. S. Appelbaum concludes that a psychologist’s ethical and legal obligation in cases such as this one is to appeal the reviewer’s decision (American Psychologist, 48(3), 251-7, 1993).

The correct answer is: file an appeal with the managed health care organization.

How well did you know this?
1
Not at all
2
3
4
5
Perfectly
82
Q

Dr. Able, a psychologist, has been seeing a client for five months and feels that the client may not be benefitting from treatment. When she mentions this to him, the client says he is quite satisfied with the progress of therapy. Based on this information, Dr. Able’s best course of action would be to:
Select one:

A. continue seeing the client in therapy since the client is satisfied.
B. tell the client that she is ethically required to refer him to another therapist.
C. obtain supervision to ensure that her treatment is effective.
D. consult with another psychologist to discuss the situation.

A

When psychologists believe that a client is not benefitting from treatment, special consideration must be given to the situation.

a. Incorrect See explanation for response d.
b. Incorrect See explanation for response d.
c. Incorrect See explanation for response d.
d. CORRECT Although referral (response b) might be acceptable, it is not necessary as a first course of action in this case given the circumstances. Instead, consultation would be a better action since it would help the therapist determine if his evaluation of the situation is correct.

The correct answer is: consult with another psychologist to discuss the situation.

How well did you know this?
1
Not at all
2
3
4
5
Perfectly
83
Q

To be consistent with ethical requirements, when a psychologist realizes that a dual (multiple) relationship is unavoidable, the psychologist should:
Select one:

A. discuss the situation with the affected person “as early as is feasible.”
B. consult with a member of the Ethics Committee to determine the best course of action.
C. take reasonable steps to resolve the issue in a way that protects the best interests of the affected person.
D. file a complaint against him/herself with the Ethics Committee.

A

This issue is addressed by Standard 3.05(b) of the APA’s Ethics Code.

a. Incorrect See explanation for response c.
b. Incorrect See explanation for response c.
c. CORRECT Standard 3.05(b) states that: “If a psychologist finds that, due to unforeseen factors, a potentially harmful multiple relationship has arisen, the psychologist takes reasonable steps to resolve it with due regard for the best interests of the affected person and maximal compliance with the Ethics Code.” This answer is most consistent with the actual language of the Code and, therefore, is the correct response.
d. Incorrect See explanation for response c.

The correct answer is: take reasonable steps to resolve the issue in a way that protects the best interests of the affected person.

How well did you know this?
1
Not at all
2
3
4
5
Perfectly
84
Q

A legal guardian has been appointed by the court for a 9-year-old child after his biological parents are found guilty of child abuse. The legal guardian brings the child to you for treatment. You should:
Select one:

A. obtain permission from both of the child’s biological parents before providing treatment.
B. obtain permission from one of the child’s biological relatives before providing treatment.
C. obtain permission from the court before providing treatment.
D. provide the treatment to the child as requested by the guardian.

A

In most cases, the biological parents are the legal guardians of a minor child; in this case, the court has appointed a legal guardian.

a. Incorrect See explanation for response d.
b. Incorrect See explanation for response d.
c. Incorrect See explanation for response d.
d. CORRECT The child’s legal representative in this case, the court-appointed guardian, has the right to request treatment for the child. Approval from the court of the child’s biological parents or relatives is not necessary.

The correct answer is: provide the treatment to the child as requested by the guardian.

How well did you know this?
1
Not at all
2
3
4
5
Perfectly
85
Q

Dr. Billings, a psychologist, asks a graduate student to help him with a research project. The student does most of the initial library research, administers the tests and interviews to subjects, and writes some of the article that Dr. Billings will submit to a professional journal. When the article is published, Dr. Billings should make sure that the graduate student is:
Select one:

A. listed as the first author.
B. listed as a co-author.
C. acknowledged in a footnote.
D. acknowledged in an introductory statement.

A

Standard 8.12(b) of 2002 the Ethics Code states that publication credits “accurately reflect the relative scientific or professional contributions of the individuals involved, regardless of their relative status.”

a. Incorrect First authorship is reserved for the principal contributor who, in this case, seems to be the psychologist.
b. CORRECT The graduate student has written some of the article and, therefore, should be listed as a co-author.
c. Incorrect Minor contributions are often acknowledged in a footnote or introductory statement. However, the student made a substantial contribution to the article and the journal article.
d. Incorrect See explanation for response c.

The correct answer is: listed as a co-author.

How well did you know this?
1
Not at all
2
3
4
5
Perfectly
86
Q

The purpose of a __________ is to guide the development of a program so that its final version maximizes the likelihood that the program will achieve its goals.
Select one:

A. summative evaluation
B. formative evaluation
C. needs assessment
D. functional assessment

A

For the exam, you want to be familiar with the difference between formative and summative evaluations so that you can answer questions like this one.

a. Incorrect A summative evaluation is conducted after a training program has been developed and administered to trainees in order to assess the program’s outcomes.
b. CORRECT Formative evaluations are conducted while a training program is being developed (“formed”) to obtain the information needed to determine what modifications are required.
c. Incorrect A needs assessment is conducted to determine if performance problems can be alleviated by providing training and, if so, to identify the content of a training program.
d. Incorrect A functional assessment (also known as a functional behavioral assessment) is conducted to identify the functions that a behavior serves in order to identify ways of modifying that behavior.

The correct answer is: formative evaluation

How well did you know this?
1
Not at all
2
3
4
5
Perfectly
87
Q

In contrast to Maslow’s need hierarchy theory, Alderfer’s ERG theory:
Select one:

A. places motivation within a satisfaction/dissatisfaction framework.
B. proposes that needs emerge in a predictable order.
C. assumes that needs may become more (rather than less) important as they are fulfilled.
D. has received little support as an explanation for motivation in the workplace.

A

ERG theory is a modification of Maslow’s need hierarchy theory. Among other things, it reduces Maslow’s five needs to three: existence, relatedness, and growth.

a. Incorrect This sounds like Herzberg’s two-factor theory.
b. Incorrect This is a characteristic of Maslow’s (not Alderfer’s) theory.
c. CORRECT This is an assumption of ERG theory and the opposite of what Maslow’s theory predicts.
d. Incorrect Alderfer’s theory has actually received more support than Maslow’s.

The correct answer is: assumes that needs may become more (rather than less) important as they are fulfilled.

How well did you know this?
1
Not at all
2
3
4
5
Perfectly
88
Q

Abraham Maslow derived his need hierarchy theory from:
Select one:

A. his experiences as a clinical psychologist.
B. his experience as an organizational consultant.
C. a review of the literature on motivation.
D. the principles of psychodynamic theory.

A

During his work as a clinical psychologist, Maslow noticed that people are motivated by different needs.

a. CORRECT Maslow derived his theory about the need hierarchy and self-actualization from his observations of therapy clients, work with college students, and study of people he believed to be highly productive (e.g., Abraham Lincoln, Thomas Jefferson, Eleanor Roosevelt).
b. Incorrect See explanation for response a.
c. Incorrect See explanation for response a.
d. Incorrect See explanation for response a.

The correct answer is: his experiences as a clinical psychologist.

How well did you know this?
1
Not at all
2
3
4
5
Perfectly
89
Q

Bazerman, Tenbrunsel, and Wade-Benzoni’s (1998) distinction between the “should self” and the “want self” is most useful for understanding which of the following?
Select one:

A. ethical decision-making
B. affiliation preferences
C. employee satisfaction
D. adolescent egocentrism

A

Brazerman et al. distinguish between two competing selves - the “should self,” which is rational and cool-headed, and the “want self,” which is emotional and hot-headed. [Bazerman, M. H., Tenbrunsel, A. E., & K. A. Wade-Benzoni, Negotiating with yourself and losing: Understanding and managing conflicting internal preferences, Academy of Management Review, 23, 225-241, 1998.]

a. CORRECT The distinction between the “should self” and “want self” has been applied to ethical, business, and other types of decision-making. With regard to ethical decision-making, the should self refers to our ethical intentions and the belief that we should act in ways that are consistent with our ethical principles. In contrast, the want self applies to our actual behavior and reflects self-interest and a relative disregard for ethical concerns.
b. Incorrect See explanation for response a.
c. Incorrect See explanation for response a.
d. Incorrect See explanation for response a.

The correct answer is: ethical decision-making

How well did you know this?
1
Not at all
2
3
4
5
Perfectly
90
Q

An advantage of the “paired comparison” technique is that it:
Select one:

A. helps reduce certain rater/observer biases.
B. provides helpful information for employee feedback.
C. is less time-consuming to complete than other techniques.
D. is more acceptable to employees and supervisors than other methods.

A

The paired comparison technique is a relative method for assessing performance that entails comparing an employee to other employees on predefined dimensions of job performance.

a. CORRECT One advantage of relative measures is that they help reduce rater biases.
b. Incorrect This is the opposite of what is true.
c. Incorrect This is the opposite of what is true.
d. Incorrect Employees tend to dislike relative measures because they do not want to be compared to other employees.

The correct answer is: helps reduce certain rater/observer biases.

How well did you know this?
1
Not at all
2
3
4
5
Perfectly
91
Q

One area of interest to industrial psychologists is the effects of rest breaks on productivity. For example, Bhatia and Murrell (1969) evaluated daily output figures after the introduction of rest breaks that reduced the total work time from eight to seven hours per day. Generally speaking, studies like the one conducted by Bhatia and Murrell have indicated that the introduction of rest breaks results in:
Select one:

A. a rise in daily output despite the fact that employees actually spend less time on the job.
B. a decrease in daily output due to reduced time on the job.
C. no significant change in productivity rates.
D. a decrease in productivity but an increase in job satisfaction.

A

Even if you are unfamiliar with the research in this area, it makes sense that the less the fatigue, the greater the productivity.

a. CORRECT Studies on the effects of rest breaks have indicated that such breaks actually increase productivity despite reduced time on the job, presumably due to decreased fatigue.
b. Incorrect There is no evidence that rest breaks cause a decrease in output.
c. Incorrect Rest breaks do appear to cause an increase in productivity.
d. Incorrect Rest breaks do not decrease productivity.

The correct answer is: a rise in daily output despite the fact that employees actually spend less time on the job.

92
Q

A problem with establishing “comparable worth” is that:
Select one:

A. job evaluation techniques are not as useful for very complex jobs.
B. males and females may use different strategies to reach the same decision or solution.
C. the job evaluation techniques themselves may be gender-biased.
D. it is difficult to compare achievement across different domains.

A

The doctrine of comparable worth extends “equal content” as a determinant of wages to “comparable worth.” In others words, even if two jobs are different in terms of content, they should have the same salary as long as they involve similar processes and demands. Comparable worth became an issue because of the gender differences in wages.

a. Incorrect See explanation for response c.
b. Incorrect See explanation for response c.
c. CORRECT A difficulty in establishing comparable worth, at least according to some experts, is that the job evaluation techniques used to establish it may themselves be gender-biased. This issue continues to be debated in the literature.
d. Incorrect See explanation for response c.

The correct answer is: the job evaluation techniques themselves may be gender-biased.

93
Q

According to Fiedler’s (1984) contingency theory of leadership, task-oriented leaders are most effective when the work situation is _______________ with regard to the leader’s sense of certainty, predictability, and control over his/her employees.
Select one:

A. moderate in favorableness
B. very high in favorableness
C. very low in favorableness
D. very high or very low in favorableness

A

Fiedler’s theory distinguishes between low-LPC (task-oriented) and high-LPC (person-oriented) leaders.

a. Incorrect See explanation for response d.
b. Incorrect See explanation for response d.
c. Incorrect See explanation for response d.
d. CORRECT Fiedler concluded that low-LPC leaders are most effective in extreme situations, i.e., in situations that are very high or very low in favorableness. In contrast, high-LPC leaders are most effective in moderately favorable situations.

The correct answer is: very high or very low in favorableness

94
Q

Chin and Benne (1976) propose that which of the following techniques would be part of a normative-reeducative strategy for overcoming resistance to change in an organization?
Select one:

A. providing information to employees about the need for change
B. having groups of employees discuss ways to accomplish change
C. having a strong leader describe ways to achieve change
D. using rewards to encourage employees to comply with plans for change

A

Chin and Benne (1976) identified three strategies for overcoming resistance to change in organizations: rational-empirical, normative-reeducative, and power-coercive.

a. Incorrect Providing employees with information about the change when resistance to change is due to a lack of knowledge is characteristic of the rational-empirical strategy.
b. CORRECT Use of the normative-reeducative strategy is based on the premise that peer pressure and sociocultural norms are effective methods for promoting change. Therefore, having employees work together to accomplish change is likely to be part of this strategy.
c. Incorrect Relying on a strong leader to gain cooperation is characteristic of the power-coercive strategy.
d. Incorrect Using rewards is also characteristic of the power-coercive strategy.

The correct answer is: having groups of employees discuss ways to accomplish change

95
Q

According to Herzberg’s “two-factor theory”:
Select one:

A. job motivation increases as job responsibility and autonomy increase.
B. job performance is determined by the interaction of two factors – job content and job context factors.
C. job motivation is a direct result of job outcomes.
D. the factors that increase an employee’s motivation depend on the employee’s level of growth need.

A

Herzberg’s two-factor theory is a theory of both satisfaction and motivation. As its name implies, it proposes that satisfaction and motivation are affected by two different types of job-related factors.

a. CORRECT According to Herzberg, job-related factors can be divided into two types: hygiene (job context) factors and motivator (job content) factors. Only the second can affect satisfaction and motivation; i.e., providing workers with jobs that are high in responsibility, challenge, and autonomy will result in satisfied and motivated workers. (The presence of hygiene factors only ensures that workers are not DISSATISFIED.)
b. Incorrect See explanation for response a.
c. Incorrect Although this is not wrong, it is too general and, therefore, not the best answer of those given.
d. Incorrect Although this is true, it is not part of Herzberg’s theory.

The correct answer is: job motivation increases as job responsibility and autonomy increase.

96
Q

The primary goal of a “realistic job preview” is to:
Select one:

A. increase productivity.
B. reduce turnover.
C. increase achievement motivation.
D. reduce sabatoge.

A

A realistic job preview provides job applicants with realistic information about the job.

a. Incorrect See explanation for response b.
b. CORRECT An assumption underlying the use of realistic job previews is that disillusionment with the job is the primary cause of turnover. Realistic job previews attempt to reduce disillusionment and resulting turnover by providing accurate information about the job.
c. Incorrect See explanation for response b.
d. Incorrect See explanation for response b.

The correct answer is: reduce turnover.

97
Q

Rational decision-making in organizations most often fails because:
Select one:

A. decisions are affected by “social desirability” and other decision biases.
B. decisions are typically guided by economic concerns only.
C. too many people are involved in the decision-making process.
D. there is insufficient information, time, and money.

A

Decision-making strategies have been categorized in terms of several basic types. One strategy is referred to as rational decision-making; as its name implies, it involves examining alternatives in a rational way to identify the best one.

a. Incorrect Social desirability is a type of response style that affects how an individual responds to test items and is not relevant to decision-making strategies.
b. Incorrect Although this is true in some cases, it is not an accurate generalization.
c. Incorrect This has not been identified by the research on rational decision-making as a cause of bad decisions.
d. CORRECT Herbert Simon has proposed that rational decision-making often fails because the decision-maker often “satisfices” (rather than “maximizes”); i.e., he/she chooses the first satisfactory alternative rather than continue to search for the optimal alternative due primarily to inadequate information about the problem and alternative solutions, to a lack of time and money, and to limits on the decision-maker’s own intelligence.

The correct answer is: there is insufficient information, time, and money.

98
Q

The Position Analysis Questionnaire (PAQ) is useful for:
Select one:

A. conducting a job analysis.
B. evaluating an employee’s job performance.
C. reducing criterion contamination.
D. reducing adverse impact.

A

The Position Analysis Questionnaire provides information on six dimensions of worker activity (e.g., mental processes, relationships).

a. CORRECT The PAQ is a quantitative, worker-oriented method of collecting data for the purpose of job analysis.
b. Incorrect See explanation for response a.
c. Incorrect See explanation for response a.
d. Incorrect See explanation for response a.

The correct answer is: conducting a job analysis.

99
Q

The results of research conducted at Ohio State University in the 1950s indicated that the behavior of leaders can be described in terms of which of the following dimensions?
Select one:

A. initiating structure and consideration
B. Theory X and Theory Y
C. participative and autocratic
D. transformational and transactional

A

All of the terms listed in the answers have been used to describe leader behaviors or leader styles. However, this question is asking about the research conducted in the 1950s at Ohio State University, so only one of the answers is correct.

a. CORRECT A number of approaches to leadership distinguish between leaders who are task-oriented and leaders who are person-oriented. This distinction is traceable to studies conducted at Ohio State University, which found that leader behavior can be described in terms of two dimensions: initiating structure (task-orientation) and consideration (person-orientation).
b. Incorrect See explanation for response a.
c. Incorrect See explanation for response a.
d. Incorrect See explanation for response a.

The correct answer is: initiating structure and consideration

100
Q

The rotating shift is associated with a number of adverse consequences including fatigue, concentration, and performance errors. The negative effects can be reduced, however, when:
Select one:

A. the rotation is clockwise and weekly.
B. the rotation is clockwise and either rapid (every 3 days) or slow (every 3 weeks).
C. the rotation is counterclockwise and weekly.
D. the rotation is counterclockwise and either rapid (every 3 days) or slow (every 3 weeks).

A

Several factors contribute to the impact of the rotating shift on worker health and performance.

a. Incorrect See explanation for response b.
b. CORRECT There is evidence that the rotating shift has fewer negative consequences when the rotation is clockwise (morning, afternoon, night) and is either rapid or slow. Weekly rotations are generally most detrimental because they do not allow resynchronization of circadian rhythms (which slow rotations do) and are long enough to allow for the build up of a sleep deficit (which fast rotations do not do). See, e.g., O. Van Reeth, Sleep and circadian disturbances in shift work: Strategies for their management, Hormone Research, 49, 158-162, 1998.
c. Incorrect See explanation for response b.
d. Incorrect See explanation for response b.

The correct answer is: the rotation is clockwise and either rapid (every 3 days) or slow (every 3 weeks).

101
Q

Tiedeman and O’Hara’s (1963) theory of career development was most influenced by the work of:
Select one:

A. Erikson.
B. Bandura.
C. Baumrind.
D. Maslow.

A

Knowing that Tiedeman and O’Hara emphasize the role of the ego in career development would have helped you identify the correct answer to this question.

a. CORRECT Tiedeman and O’Hara’s theory is classified as a cognitive-developmental theory that is consistent with the work of Erikson, Bruner, Piaget, and Allport. It views career development as being an aspect of ego identity development and as a process that continues throughout the lifespan.
b. Incorrect See explanation for response a.
c. Incorrect See explanation for response a.
d. Incorrect See explanation for response a.

The correct answer is: Erikson.

102
Q

Maslow’s theory of motivation predicts that needs are activated in a specific order, with lower level needs being aroused first. From lowest to highest level, Maslow’s five needs are:
Select one:

A. safety, physiological, social, esteem, self-actualization.
B. existence, social, growth, esteem, self-actualization.
C. physiological, social, achievement, esteem, self-actualization.
D. physiological, safety, social, esteem, self-actualization.

A

Maslow’s theory distinguishes between five needs that emerge in a hierarchical fashion from lowest to highest.

a. Incorrect See explanation for response d.
b. Incorrect See explanation for response d.
c. Incorrect See explanation for response d.
d. CORRECT These are Maslow’s five needs, and they are arranged in the correct order from lowest to highest.

The correct answer is: physiological, safety, social, esteem, self-actualization.

103
Q

The belief that job-related behaviors are related to the similarity between basic personality and characteristics of the work environment underlies which of the following?
Select one:

A. Roe’s fields and levels theory
B. Holland’s occupational themes
C. Alderfer’s ERG theory
D. Super’s stages of vocational development

A

Holland’s six occupational themes represent basic personality types.

a. Incorrect Roe’s theory focuses on the relationship between childhood experience and occupational choice.
b. CORRECT Holland proposed that job satisfaction, performance, and so on are directly related to the degree of similarity between the person’s basic personality and the nature of the environment. A conventional person, for instance, is happiest in a conventional environment.
c. Incorrect ERG theory is a theory of motivation that posits three basic needs (existence, relatedness, and growth).
d. Incorrect Super’s theory emphasizes the role of self-concept in vocational development and choice.

The correct answer is: Holland’s occupational themes

104
Q

From the perspective of the situational leadership model, when an employee is both competent and confident, the best leadership style is which of the following?
Select one:

A. selling
B. telling
C. participating
D. delegating

A

Hersey and Blanchard’s (1982) situational leadership model is based on the premise that a leader’s style should match the “maturity level” of the employee, which is related to both ability and motivation.

a. Incorrect A selling style is appropriate for employees who are moderate in maturity (are unable but willing or confident).
b. Incorrect A telling leadership style is best for employees who are low in maturity (are both unable and unwilling or insecure).
c. Incorrect A participative leadership style is most effective for employees who are moderate to high in maturity (are able but unwilling or insecure).
d. CORRECT A delegating leadership style is appropriate for highly mature employees. When adopting this style, the leader is low in both relationship and task behavior.

The correct answer is: delegating

105
Q

Research on goal-setting theory suggests that monetary incentives:
Select one:

A. are irrelevant to goal achievement as long as the goals have been accepted by employees
B. are irrelevant to goal achievement as long as the goals have been self-set by employees
C. have a positive effect only if employees participate in determining the difficulty of the goals
D. have a positive effect whether or not employees have been involved in goal-setting

A

This is one of those questions you probably could have answered correctly without being familiar with the research.

a. Incorrect See explanation for response “d.”
b. Incorrect See explanation for response “d.”
c. Incorrect See explanation for response “d.”
d. CORRECT Monetary incentives seem to have a positive effect on goal achievement (especially when they are tied specifically to achievement) regardless of the circumstances. (Although some research suggests that monetary incentives can actually have a detrimental effect when goals are self-set, this isn’t addressed by any of the answers, so this response is the best one. See M. Erez, et al., Effects of goal difficulty, self-set goals, and monetary rewards on dual task performance, Organizational Behavior and Human Decision Processing, 47, 247-269, 1990.)

The correct answer is: have a positive effect whether or not employees have been involved in goal-setting

106
Q

Which of the following is NOT an example of a centralized communication network?
Select one:

A. wheel
B. chain
C. circle
D. Y

A

In a centralized communication network, all communication passes through a central person.

a. Incorrect See explanation for response c.
b. Incorrect See explanation for response c.
c. CORRECT The wheel, chain, and Y are examples of a centralized communication network, while the circle is an example of a decentralized network in which information can flow freely from one individual to another without having to go through a central person.
d. Incorrect See explanation for response c.

The correct answer is: circle

107
Q

Which of the following would be most effective for reducing a rater’s leniency bias?
Select one:

A. BARS
B. peer ratings
C. forced-choice technique
D. BIB

A

A leniency bias occurs when a rater rates all ratees very positively regardless of their actual level of performance. Likert-type scales are most susceptible to leniency and other rater biases.

a. Incorrect BARS is a type of rating scale. Although it was developed to help increase rater accuracy, the research has not been particularly favorable.
b. Incorrect Peer ratings would not overcome the problem of leniency.
c. CORRECT A forced choice scale presents items that have been identified as being of equal social desirability in dyads or triads and requires the rater to pick the item that best describes the ratee. Because of its format, it eliminates the leniency bias.
d. Incorrect A BIB - biographical information blank - is not a rating scale.

The correct answer is: forced-choice technique

108
Q

Level of organizational commitment is most predictive of which of the following?
Select one:

A. turnover
B. job satisfaction
C. quantity and quality of work
D. life satisfaction

A

Organizational commitment refers to the strength of a person’s identification with and involvement in a particular organization.

a. CORRECT Like job satisfaction, commitment appears to have the highest correlation with turnover. It is also related to the individual’s willingness to exert effort on behalf of the company.
b. Incorrect See explanation above.
c. Incorrect See explanation above.
d. Incorrect See explanation above.

The correct answer is: turnover

109
Q

Which of the following theories of leadership provides a leader with a decision tree to help him or her decide whether an autocratic, consultative, or group decision-making style is optimal given the characteristics of the situation?
Select one:

A. Hersey and Blanchard’s situational leadership model
B. Graen’s leader-member exchange model
C. House’s path-goal theory
D. Vroom, Yetton, and Jago’s leader-participation model

A

Although three of the theories listed in the responses propose that the best leader style depends on the characteristics of the situation, only one includes a decision tree to help leaders pick the best style.

a. Incorrect See explanation for response d.
b. Incorrect See explanation for response d.
c. Incorrect See explanation for response d.
d. CORRECT The Vroom-Yetton-Jago model distinguishes between autocratic, consultative, and group decision-making leader styles and proposes that the best one depends on certain characteristics of the situation including time constraints, the importance of subordinate commitment to the decision, and the structure of the task. The model includes a decision-tree to help leaders choose the optimal style for their situation.

The correct answer is: Vroom, Yetton, and Jago’s leader-participation model

110
Q

Pavlov found that requiring dogs to make difficult discriminations between stimuli during the course of conditioning trials led to which of the following?
Select one:

A. superstitious behavior
B. experimental neurosis
C. overshadowing
D. blocking

A

For the exam, you want to be familiar with all of the phenomena listed in the answers to this question. These are described in the Learning Theory chapter of the written study materials.

a. Incorrect See explanation for response b.
b. CORRECT In one series of studies, Pavlov found that dogs that were conditioned to respond with salivation to a circle but not to respond with salivation to an ellipse subsequently exhibited unusual behaviors when presented with a more circular ellipse. He attributed this behavior to a conflict between cortical excitation and inhibition and referred to it as experimental neurosis.
c. Incorrect See explanation for response b.
d. Incorrect See explanation for response b.

The correct answer is: experimental neurosis

111
Q

__________ inhibition occurs when the ability to remember new information is impaired by previously acquired information.
Select one:

A. Proactive
B. Retroactive
C. Anterograde
D. Retrograde

A

For the exam, you want to be familiar with the difference between proactive and retroactive inhibition (interference) and between anterograde and retrograde amnesia.

a. CORRECT Proactive inhibition occurs when old (previous) learning interferes with the learning or recall of new (subsequent) information.
b. Incorrect Retroactive inhibition occurs when recently learned information interferes with the ability to recall previously learned information.
c. Incorrect Anterograde amnesia refers to the inability to recall events and experiences that occur after the event that produced the amnesia.
d. Incorrect Retrograde amnesia refers to the inability to recall events and experiences that occurred before the event that produced the amnesia.

The correct answer is: Proactive

112
Q

The primary goal of self-instructional training (SIT) for children exhibiting hyperactivity and excessive impulsivity is to:
Select one:

A. eliminate obsessive ruminations.
B. re-align intention and action.
C. clarify response contingencies.
D. insert thought between stimulus and response.

A

SIT involves training the individual to guide his/her actions with internally-originated verbal instructions. Additional information about SIT is provided in the Learning Theory chapter of the written study materials.

a. Incorrect See explanation for response d.
b. Incorrect This answer is close but not as good as response d.
c. Incorrect Although self-instruction includes self-reinforcement, this is not the primary goal of SIT.
d. CORRECT Hyperactive and impulsive children often respond to events automatically. The goal of SIT is to reduce automatic behaviors by inserting thoughts between an event and the response to that event.

The correct answer is: insert thought between stimulus and response.

113
Q

The “FI scallop” refers to:
Select one:

A. the pause followed by accelerated responding that occurs within each fixed interval.
B. the increase in responding that occurs after reinforcement at the end of each fixed interval.
C. the gradual pause followed by an essential instantaneous high rate of responding that occurs between each fixed interval.
D. the pause in responding that signals satiation after exposure to an extended fixed interval schedule.

A

The term “scallop” is ordinarily applied to the response pattern that is observed when an organism is being reinforced on a fixed interval schedule.

a. CORRECT When using an FI schedule, the delivery of reinforcement is typically followed first by a pause in responding and then an increasing rate of responding until the next reinforcement is delivered. On a cumulative record, this produces a “scallop.”
b. Incorrect See explanation for response a.
c. Incorrect See explanation for response a.
d. Incorrect See explanation for response a.

The correct answer is: the pause followed by accelerated responding that occurs within each fixed interval.

114
Q

As originally described by Wolpe (1958), reciprocal inhibition involved replacing:
Select one:

A. an automatic response with an intentional response.
B. a dysfunctional thought with a more functional one.
C. an anxiety response with a relaxation response.
D. an external attribution with an internal attribution.

A

Reciprocal inhibition involves using classical conditioning to replace an undesirable response with an incompatible response. The term reciprocal inhibition is often used interchangeably with counterconditioning.

a. Incorrect See explanation for response c.
b. Incorrect See explanation for response c.
c. CORRECT Wolpe originally used reciprocal inhibition to reduce anxiety. This involved pairing, in imagination, anxiety-arousing events with relaxation, which is incompatible with anxiety.
d. Incorrect See explanation for response c.

The correct answer is: an anxiety response with a relaxation response.

115
Q

Implosive therapy would be most useful as a treatment for which of the following?
Select one:

A. Specific Phobia
B. Major Depressive Disorder
C. Hypoactive Sexual Desire Disorder
D. Reactive Attachment Disorder

A

Implosive therapy combines classical extinction with psychodynamic interpretation and is used to treat anxiety reactions.

a. CORRECT Knowing that implosive therapy is used to extinguish a phobic or other anxiety response would have enabled you to identify the correct answer to this question.
b. Incorrect See explanation above.
c. Incorrect See explanation above.
d. Incorrect See explanation above.

The correct answer is: Specific Phobia

116
Q

The effectiveness of the procedure known as time-out is maximized when:
Select one:

A. the length of the time-out is directly related to the severity of the misbehavior.
B. the alternative behavior is truly incompatible with the problem behavior.
C. the time-out is alternated with the technique known as response cost.
D. the time-out begins with a brief explanation of why it is being applied.

A

Time-out is used to reduce an undesirable behavior and involves removing the individual from all sources of positive reinforcement following the behavior for a specific period of time.

a. Incorrect Relatively short time-outs (1-5 minutes) have been found to be as effective as longer time-outs, and there is no evidence linking the effectiveness of this technique to matching its length to the severity of the misbehavior.
b. Incorrect When using time-out, there is no incompatible behavior. (Of course, whenever one behavior is being eliminated, it’s always a good idea to reinforce an alternative behavior at the same time, but this is not part of the time-out procedure itself.)
c. Incorrect There’s no data supporting this combination of techniques.
d. CORRECT To maximize its effectiveness, the time-out should begin with either a signal or a brief explanation of why it is being applied (e.g., “You hit your brother, so you’ll have to sit in the time-out chair for five minutes”).

The correct answer is: the time-out begins with a brief explanation of why it is being applied.

117
Q

In the context of operant conditioning, “fading” refers to:
Select one:

A. a reduction in positive reinforcement.
B. the gradual removal of prompts.
C. extinguishing a previously reinforced response.
D. eliminating stimulus generalization.

A

Fading is often confused with thinning. For the exam, you want to be sure you know the difference between the two terms.

a. Incorrect A reduction in positive reinforcement (e.g., switching from a continuous to an intermittent schedule) is referred to as thinning.
b. CORRECT Nonverbal and verbal prompts are used to help initiate a response. Because the ultimate goal is for the response to occur independently, prompts are gradually removed once the response is well-established. This procedure is referred to as fading.
c. Incorrect See explanation for response b.
d. Incorrect See explanation for response b.

The correct answer is: the gradual removal of prompts.

118
Q

A mental health facility establishes a new policy that prohibits the use of all forms of positive punishment to control the behavior of its inpatients. Which of the following techniques would be unacceptable under this policy?
Select one:

A. verbal reprimands (e.g., yelling “stop” after an undesirable behavior)
B. response cost (e.g., removing TV privileges after an undesirable behavior)
C. Premack Principle (e.g., making TV privileges contingent on the performance of a desirable behavior)
D. time-out (e.g., placing the individual in an empty room for ten minutes after an undesirable behavior).

A

Positive punishment involves applying a stimulus following a behavior to decrease the frequency of that behavior.

a. CORRECT Verbal reprimands are a type of positive punishment because they involve applying a stimulus after a behavior in order to reduce or eliminate that behavior.
b. Incorrect Response cost is a type of negative punishment and involves removing a stimulus following a behavior in order to decrease the frequency of that behavior.
c. Incorrect The Premack Principle is a type of positive reinforcement.
d. Incorrect Time-out is classified as a type of negative punishment because it involves removing all sources of reinforcement following a behavior to decrease or eliminate that behavior.

The correct answer is: verbal reprimands (e.g., yelling “stop” after an undesirable behavior)

119
Q

When using ________ as a memory aid, a phrase or rhyme is constructed from the first letter of each word in the list of words that is to be recalled.
Select one:

A. the chunking strategy
B. the keyword method
C. an acronym
D. an acrostic

A

For the exam, you want to be familiar with the four memory devices listed in the answers to this question.

a. Incorrect Chunking involves grouping related items of information. To remember a string of 21 numbers, for instance, you might chunk the numbers into groups of three, resulting in seven distinct units of information. Chunking is useful for maintaining information in working memory.
b. Incorrect The keyword method is an imagery mnemonic that is useful for paired associate tasks in which two words must be linked. It involves creating an image for each word and visually joining the two images. As an example, to remember that the French word for church is eglise, you could form an image of a church with a giant egg outside the church doors.
c. Incorrect An acronym is a word that is formed using the first letter of each item in a list. For example, the acronym HOMES is used to remember the Great Lakes (Huron, Ontario, Michigan, Erie, and Superior).
d. CORRECT An acrostic is made up of words beginning with the first letter of each word or phrase that is to be remembered. For example, the acrostic “my very educated mother just sent us nachos” is used to memorize the names of the eight planets (Mercury, Venus, Earth, Mars, Jupiter, Saturn, Uranus, and Neptune).

The correct answer is: an acrostic

120
Q

In Albert Ellis’s (1985) A-B-C model, B refers to:
Select one:

A. the environmental barriers to rational thought.
B. the biological bases for human behavior.
C. a person’s belief about an activating event.
D. a person’s behavioral reaction to an activating event.

A

Ellis’s A-B-C model is central to the theory and practice of his Rational-Emotive Behavior Therapy.

a. Incorrect See explanation for response c.
b. Incorrect See explanation for response c.
c. CORRECT Ellis described emotions and behaviors in terms of a chain of events – an external activating event (A), the person’s belief about the activating event (B), and the emotional or behavioral consequence of that belief (C).
d. Incorrect See explanation for response c.

The correct answer is: a person’s belief about an activating event.

121
Q

A behaviorist would most likely argue that the use of spanking as a behavioral management technique is not effective because:
Select one:

A. it leads to physical child abuse.
B. it fosters a sense of learned helplessness in the child.
C. changes in behavior aren’t likely to persist over time.
D. changes in behavior won’t occur at all.

A

From a behavioral perspective, punishments such as spanking are problematic in that they suppress but do not eliminate behavior.

a. Incorrect Though this may be true in some cases, it has nothing to do with the effectiveness of spanking as a behavioral management technique (i.e., its effectiveness in reducing a particular behavior).
b. Incorrect This again has little, if anything, to do with the effectiveness of spanking as a behavioral management technique.
c. CORRECT Over time, punishment alone is not likely to be effective, since it only results in a temporary suppression of behavior. In fact, as soon as the punishment is not present, the behavior is likely to increase.
d. Incorrect Punishment usually does result in a decrease in the targeted behavior, though, as already noted, this increase is likely to be temporary.

The correct answer is: changes in behavior aren’t likely to persist over time.

122
Q

From the perspective of Rational-Emotive Behavior Therapy:
Select one:

A. irrational beliefs are acquired primarily through social learning processes.
B. irrational beliefs are acquired largely through the process of selective reinforcement.
C. people are biologically prone to the acquisition of irrational beliefs.
D. people adopt irrational beliefs as the result of early traumatic events that are still unresolved.

A

This question is asking about one of the lesser known facts about REBT.

a. Incorrect See explanation for response c.
b. Incorrect See explanation for response c.
c. CORRECT Ellis believed that the majority of behavior has a biological basis. With regard to irrational thoughts, he proposed that they are attributable to natural tendencies toward, for example, moodiness, negativism, and excitement-seeking.
d. Incorrect See explanation for response c.

The correct answer is: people are biologically prone to the acquisition of irrational beliefs.

123
Q

Research on the serial position effect has provided evidence for which of the following?
Select one:

A. the presence of hemispheric specialization.
B. the distinction between short- and long-term memory.
C. the effects of proactive and retroactive interference.
D. the effects of latent learning.

A

The serial position effect refers to the tendency of people to recall words in the beginning and end of a list better than words in the middle of the list when asked to recall the words immediately after reading the list.

a. Incorrect See explanation for response b.
b. CORRECT The ability to recall words from the beginning of the list is referred to as the primacy effect and is attributed to the storage of those words in long-term memory. In contrast, the ability to recall words from the end of the list is referred to as the recency effect and is attributed to the presence of those words in short-term memory.
c. Incorrect See explanation for response b.
d. Incorrect See explanation for response b.
The correct answer is: the distinction between short- and long-term memory.

124
Q

In his research, a cognitive psychologist uses paired-associate and serial learning tasks. Apparently this psychologist is investigating:
Select one:

A. verbal learning.
B. selective attention.
C. procedural memory.
D. iconic memory.

A

A paired associate task requires the individual to respond with one member of a pair when presented with the other member. A serial learning task requires the individual to learn and recall a list of words in a particular order.

a. CORRECT These tasks are frequently used in the study of verbal learning.
b. Incorrect See explanation above.
c. Incorrect See explanation above.
d. Incorrect See explanation above.

The correct answer is: verbal learning.

125
Q

Biologically-based phobias are to socially-based phobias as:
Select one:

A. secondary reinforcers are to primary reinforcers.
B. response generalization is to stimulus generalization.
C. stimulus discrimination is to stimulus generalization.
D. unconditioned stimuli are to conditioned stimuli.

A

Biologically-based phobias are generally viewed as unconditioned responses, while socially-based phobias are considered to be conditioned (learned) responses.

a. Incorrect See explanation for response d.
b. Incorrect See explanation for response d.
c. Incorrect See explanation for response d.
d. CORRECT Of the responses given, this one is best because it parallels the nature of the relationship between biologically- and socially-based phobias.

The correct answer is: unconditioned stimuli are to conditioned stimuli.

126
Q

Which of the following is an example of higher-order conditioning?
Select one:

A. A CS is paired with an US until a CR is established; subsequently, the CR occurs in the presence of stimuli similar to the US.
B. A CS is paired with an US until a CR is established; subsequently, the CS elicits a variety of responses including the CR.
C. A CS is paired with an US until a CR is established; subsequently, the CS is paired with a second US until a second CR is established.
D. A CS is paired with an US until a CR is established; subsequently, a second CS is paired with the first CS until it also elicits a CR.

A

Higher-order conditioning involves using a CS as an US in order to establish a CR for a second CS.

a. Incorrect This describes stimulus generalization.
b. Incorrect This could be interpreted as response generalization, although the description is too vague to know exactly what is being described.
c. Incorrect This simply describes classical conditioning in which the CS is paired with two different US’s.
d. CORRECT This describes the procedure known as higher-order conditioning.

The correct answer is: A CS is paired with an US until a CR is established; subsequently, a second CS is paired with the first CS until it also elicits a CR.

127
Q

A practitioner of Beck’s cognitive behavioral therapy works together with a therapy client to identify and test faulty interpretations. This process is referred to as:
Select one:

A. cognitive experimentation.
B. collaborative empiricism.
C. cognitive disputation.
D. collaborative analysis.

A

A key feature of Beck’s cognitive behavioral therapy (CBT) is its collaborative approach.

a. Incorrect See explanation for response b.
b. CORRECT In CBT, the therapist and client work collaboratively to reality test the client’s cognitions. This approach is referred to as collaborative empiricism.
c. Incorrect See explanation for response b.
d. Incorrect See explanation for response b.

The correct answer is: collaborative empiricism.

128
Q

Research (e.g., Davidson and Parker, 2001) evaluating the mechanisms that contribute to the effectiveness of EMDR (eye movement desensitization and reprocessing) has generally found that its beneficial effects are attributable to:
Select one:

A. rapid eye movements.
B. exposure to the feared stimulus.
C. a placebo effect.
D. counterconditioning.

A

P. R. Davidson and K. C. H. Parker summarize the research in their article, Eye movement desensitization and reprocessing (EMDR): A meta-analysis [Journal of Consulting and Clinical Psychology, 69, 305-316, 2001].

a. Incorrect See explanation for response b.
b. CORRECT Based on the results of their meta-analysis, these investigators conclude that “eye movements are unnecessary and that EMDR may be viewed as an imaginal exposure technique” (p. 2).
c. Incorrect See explanation for response b.
d. Incorrect See explanation for response b.

The correct answer is: exposure to the feared stimulus.

129
Q

A research participant is given a list of unrelated words to remember. The list is taken away and the participant is engaged in a distracting task for 10 seconds before she is asked to recall the words in any order. The participant’s recall will be best for:
Select one:

A. words that were at the end of the list.
B. words that were in the middle of the list.
C. words that were in the beginning of the list.
D. words that were in the beginning and middle of the list.

A

This question is asking about the serial position effect.

a. Incorrect See explanation for response c.
b. Incorrect See explanation for response c.
c. CORRECT When there is no delay between exposure to the list and recall, words at the beginning and end of a list are recalled the best and to about the same degree. However, when there is a brief delay (as in this question), words at the beginning of the list are recalled best.
d. Incorrect See explanation for response c.

The correct answer is: words that were in the beginning of the list.

130
Q

A therapy client says, “I feel useless and incompetent and, therefore, I must be a worthless person.” From the perspective of cognitive therapy, this is an example of:
Select one:

A. all-or-none thinking.
B. overgeneralization.
C. personalization.
D. emotional reasoning.

A

In this situation, the client is using her emotions as evidence for the way things really are.

a. Incorrect All-or-nothing thinking is the tendency to evaluate performance or personal qualities in terms of extremist, black-and-white categories.
b. Incorrect Overgeneralization refers to arbitrarily drawing the conclusion that a single negative event will happen over and over again.
c. Incorrect Personalization involves erroneously attributing external events to oneself.
d. CORRECT The logic underlying emotional reasoning is “I feel, therefore I am.”

The correct answer is: emotional reasoning.

131
Q

Whenever a mother yells at her child when he is whining, the boy stops whining for a short period. Over time, the mother notices that she’s having to yell at her son more and more often to keep the boy from whining. The boy is controlling his mother’s behavior through:
Select one:

A. positive reinforcement.
B. negative reinforcement.
C. positive punishment.
D. negative punishment.

A

Note that this question is asking about the mother’s behavior (yelling), not the boy’s behavior. To determine what is occurring, you have to determine (1) whether the mother’s yelling is increasing or decreasing and (2) what occurs after the mother yells: The mother’s yelling is increasing, which indicates that it is being reinforced. And the consequence following her yelling is removed (i.e., the boy temporarily stops whining). This indicates that the reinforcement is negative.

a. Incorrect Positive reinforcement involves the application of a stimulus following a behavior with a resultant increase in the behavior. In the situation described in this question, the stimulus is removed following the behavior, not applied.
b. CORRECT As described above, this situation is an example of negative reinforcement. The mother’s behavior is increasing because a stimulus is removed after the behavior occurs.
c. Incorrect Punishment results in a decrease in the behavior, with positive punishment occurring when a behavior decreases because a stimulus is applied following the behavior.
d. Incorrect Negative punishment is occurring when a behavior decreases because a stimulus is withdrawn following the behavior.

The correct answer is: negative reinforcement.

132
Q

The symptoms of fetal alcohol syndrome (FAS):
Select one:

A. gradually decline during early childhood with the appropriate environmental stimulation.
B. are largely reversible with the proper diet during the first several months of life.
C. are limited primarily to physical defects (facial abnormalities and retarded growth).
D. are largely irreversible and include physical, behavioral, and cognitive impairments.

A

Fetal alcohol syndrome is the most severe form of fetal alcohol spectrum disorder (FASD) and is characterized by a pattern of physical, behavioral, and cognitive impairments that are associated with alcohol consumption by pregnant women.

a. Incorrect Although some of the facial deformities become less obvious with increasing age, most deficits are irreversible.
b. Incorrect Proper diet has not been found to have a significant effect on impairments due to fetal alcohol syndrome.
c. Incorrect fetal alcohol syndrome also involves substantial cognitive and behavioral problems.
d. CORRECT The severity and nature of the symptoms of fetal alcohol syndrome are related to the amount of alcohol consumed by the mother during pregnancy. For most children, however, the symptoms are serious and are, to a large degree, irreversible.

The correct answer is: are largely irreversible and include physical, behavioral, and cognitive impairments.

133
Q

It is not until about _____ years of age that most children express more favorable attitudes toward members of their own ethnic or racial group than toward members of other groups.
Select one:

A. 4
B. 6
C. 8
D. 10

A

Early studies on in-group preferences found that children typically demonstrate preferences for members of their own ethnic/racial group and gender by about age 4 and 5, respectively.

a. CORRECT A. Clark, D. Hocevar, and M. H. Dembo, for instance, found that, by 4 years of age, children typically express more favorable attitudes toward members of their own ethnic group than members of other ethnic groups [The role of cognitive development in children’s explanations and preferences for skin color, Developmental Psychology, 16(4), 332-339, 1980].
b. Incorrect See explanation above.
c. Incorrect See explanation above.
d. Incorrect See explanation above.

The correct answer is: 4

134
Q

Kohlberg’s cognitive-developmental theory distinguishes between three stages in gender identity development. The first stage is ________, which involves recognizing one’s own gender and is ordinarily first apparent between 2 and 3 years of age.
Select one:

A. gender constancy
B. gender conservation
C. gender identity
D. gender stability

A

According to Kohlberg’s cognitive-developmental theory, the development of gender identity involves a sequence of three stages - gender identity, gender stability, and gender constancy.

a. Incorrect Gender constancy does not develop until 6 to 7 years of age.
b. Incorrect Gender conservation is an alternative term for gender constancy.
c. CORRECT Children begin to identify themselves (and others) as either a boy or girl at 2 to 3 years of age. This is referred to as gender identity.
d. Incorrect Gender stability is usually evident by 4 to 6 years of age.

The correct answer is: gender identity

135
Q

A woman is having trouble with her six-month old son who often “fusses” at bedtime and wakes up at least once or twice during the night. She does an Internet search on the problem and finds several articles about Dr. Richard Ferber. She learns that his method for getting babies to sleep (“Ferberizing”) is supported by many experts and involves:
Select one:

A. a “progressive-waiting” approach.
B. therapeutic touch.
C. a “cold turkey” approach.
D. co-sleeping.

A

Ferber’s method is similar to that promoted by a number of other experts.

a. CORRECT When using Ferber’s method, the parent puts the child to bed. If he cries, the parent returns and reassures the child and pats him on the back but does not pick him up. If the child cries again later, the parent waits a little longer before returning. On subsequent nights, the wait period is gradually increased, which is why the method is known as the progressive waiting method. (R. Ferber, Solve Your Child’s Sleep Problems, Fireside, New York, 2006.)
b. Incorrect See explanation above.
c. Incorrect See explanation above.
d. Incorrect See explanation above.

The correct answer is: a “progressive-waiting” approach.

136
Q

Children begin to express jealousy, embarrassment, and other self-conscious emotions between the ages of:
Select one:

A. 6 to 12 months.
B. 18 to 24 months.
C. 30 to 36 months.
D. 42 to 48 months.

A

Emotional development follows a predictable sequence of stages. One milestone in this sequence is the emergence of self-conscious emotions. Note, however, that the research is somewhat inconsistent with regard to the specific age at which different self-conscious emotions emerge.

a. Incorrect See explanation for response b.
b. CORRECT Children begin to exhibit self-conscious emotions at the time the sense of self emerges, which is between 18 and 24 months of age. By 24 months, jealousy, embarassment, and empathy are usually evident; and these emotions are followed by shame, guilt, and pride at 30 to 36 months.
c. Incorrect See explanation for response b.
d. Incorrect See explanation for response b.

The correct answer is: 18 to 24 months.

137
Q

Research using elicited imitation tasks (e.g., imitating a sequence of events such as removing a mitten from a puppet, shaking the mitten, and then replacing the mitten on the puppet) indicate that, for most infants, the onset of the ability to recall the past occurs:
Select one:

A. between 3 and 6 months of age.
B. between 6 and 12 months of age.
C. between 12 and 18 months of age.
D. between 18 and 24 months of age.

A

There is physiological and behavioral evidence that the neural mechanisms required for long-term memory recall undergo significant development during the second half of the first year of life.

a. Incorrect See explanation for response b.
b. CORRECT In one study using the procedure described in this question, 75% of infants 6, 9, or 11 months of age imitated at least one action (e.g., removing the mitten) after a delay of 24 hours (R. Barr et al., Developmental changes in deferred imitation by 6- to 24-month old infants, Infant Behavior and Development, 19, 159-170, 1996).
c. Incorrect See explanation for response b.
d. Incorrect See explanation for response b.

The correct answer is: between 6 and 12 months of age.

138
Q

Studies investigating self-recognition in young children have found that by _______ months of age, over 60% of infants recognize their own image in a mirror.
Select one:

A. 21
B. 18
C. 15
D. 12

A

Researchers have used a number of techniques to investigate self-recognition in babies including mirrors, photos, and use of the words “I” and “me.”

a. CORRECT In a frequently cited study investigating the emergence of self-recognition, J. Brooks-Gunn and M. Lewis had mothers wipe the noses of their babies, ages 9 to 24 months, with a red dye, and then place the babies in front of a mirror. Results indicated that about 30% of the 18-month-old babies recognized themselves in the mirror (i.e., they rubbed the red spot on their own noses when they saw their reflection), while 60% of the 21-month-old babies did so (Social cognition and the acquisition of self, New York, Plenum, 1979).
b. Incorrect See explanation for response a.
c. Incorrect See explanation for response a.
d. Incorrect See explanation for response a.

The correct answer is: 21

139
Q

Gerald Patterson and his colleagues have developed a model to explain the origins of adolescent delinquency. Their approach is most consistent with which of the following?
Select one:

A. object relations theory
B. social learning theory
C. cognitive developmental theory
D. social comparison theory

A

According to Patterson and his colleagues, adolescent delinquency is traceable to unskilled and inefficient childrearing by the child’s parents, high levels of punishment, and a tendency to react to a child’s negative actions in kind.

a. Incorrect See explanation for response b.
b. CORRECT Patterson’s model emphasizes the impact of modeling/imitation and reinforcement and, therefore, is consistent with a social learning approach. Additional information about Patterson’s model is provided in the Lifespan Development chapter in the written study materials.
c. Incorrect See explanation for response b.
d. Incorrect See explanation for response b.

The correct answer is: social learning theory

140
Q

Solitary (non-social) play:
Select one:

A. is the predominant form of play for children between one and three years of age.
B. is useful as an indicator of cognitive development during the preschool years.
C. may be indicative of developmental delays if it constitutes a significant portion of the child’s play after age three.
D. is not necessarily less mature than group play and, in some circumstances, may be associated with a high level of competence.

A

Research on children’s play suggests that it is the quality, not the quantity, of solitary play that determines its significance.

a. Incorrect See explanation for response d.
b. Incorrect See explanation for response d.
c. Incorrect See explanation for response d.
d. CORRECT The studies have generally shown that nonsocial play is not necessarily more immature than group play. In fact, because it often consists of goal-directed or educational activities or activities involving large-motor movements, it is often associated with high levels of competence.

The correct answer is: is not necessarily less mature than group play and, in some circumstances, may be associated with a high level of competence.

141
Q

Based on her study of preschoolers and their parents, Diana Baumrind (1967, 1971) concluded that children of __________ parents tend to be aggressive, mistrusting, dependent, and “conflicted-irritable.”
Select one:

A. instrumental
B. permissive
C. authoritative
D. authoritarian

A

Baumrind originally categorized parents as authoritarian, authoritative, permissive or rejecting-neglecting.

a. Incorrect This is not one of the categories identified by Baumrind.
b. Incorrect Baumrind found that children of permissive parents tend to be “impulsive-aggressive.”
c. Incorrect Baumrind found that children of authoritative parents are most often “energetic-friendly.”
d. CORRECT Baumrind’s research linked authoritarian parenting with children who are “conflicted-irritable” - i.e., fearful, moody, easily annoyed, and aimless.

The correct answer is: authoritarian

142
Q

Studies examining the consequences of divorce and maternal remarriage on children has revealed that boys and girls have difficulty adjusting to a stepfather during the first two years after remarriage and that:
Select one:

A. both adapt thereafter and, in many cases, show better adjustment than peers whose mothers have not remarried.
B. boys (but not girls) adapt thereafter and, in many cases, show better adjustment than male peers whose mothers have not remarried.
C. girls (but not boys) adapt thereafter and, in many cases, show better adjustment than female peers whose mothers have not remarried.
D. subsequently continue to be less well-adjusted than peers whose mothers have not remarried.

A

Questions on the effects of divorce and stepparenting are difficult because the research is not entirely consistent. However, there is some fairly consistent evidence that having a stepfather has fewer benefits for girls than for boys.

a. Incorrect See explanation for response b.
b. CORRECT Boys adapt to stepfathers better than girls and often eventually show better adjustment than male peers whose mothers did not remarry (see, e.g., E. M. Hetherington, Coping with family transitions: Winners, losers, and survivors, Child Development, 60, 60-114, 1989).
c. Incorrect See explanation for response b.
d. Incorrect See explanation for response b.

The correct answer is: boys (but not girls) adapt thereafter and, in many cases, show better adjustment than male peers whose mothers have not remarried.

143
Q

According to Erikson’s theory of psychosocial development, middle-aged adults are in which stage of development?
Select one:

A. ego integrity versus despair
B. initiative versus guilt
C. generativity versus stagnation
D. industry versus inferiority

A

Erikson’s theory of psychosocial development includes eight stages. Middle-aged adults are in the second to last stage. For additional information on Erikson’s stages, see the Lifespan Development chapter of the written study materials.

a. Incorrect Ego integrity versus despair is the final psychosocial conflict.
b. Incorrect Initiative versus guilt is the conflict for children aged three through six.
c. CORRECT Generativity versus stagnation is the psychosocial conflict for middle-aged adults.
d. Incorrect Industry versus inferiority is the conflict for those aged six to twelve.

The correct answer is: generativity versus stagnation

144
Q

As defined by Piaget, a __________ circular reaction occurs when a baby’s action gets a pleasurable or interesting response from an object or other person, which then leads the baby to repeat the action.
Select one:

A. reflexive
B. primary
C. secondary
D. tertiary

A

According to Piaget, much of the cognitive development that occurs during the sensorimotor stage is the result of circular reactions, in which the child learns to reproduce a pleasurable or interesting experience that originally occurred by chance. Piaget distinguished between three types of circular reactions - primary, secondary, and tertiary.

a. Incorrect Piaget considered reflexes to be the “building blocks” of cognitive development from birth to 1 month.
b. Incorrect Primary circular reactions predominate from ages 1 to 4 months. They involve simple motor habits that center around the baby’s own body (e.g., thumb sucking).
c. CORRECT Secondary circular reactions predominate from ages 4 to 8 months and are actions involving other people or objects. For example, at this stage, an infant may learn that grabbing a rattle with his/her hand and shaking it produces an interesting noise.
d. Incorrect Tertiary circular reactions predominate from ages 12 to 18 months and involve varying an original action on an external object to see what happens.

The correct answer is: secondary

145
Q

Longitudinal research by Tizard and colleagues (e.g., Hodges and Tizard, 1989) indicates that infants initially raised in institutions are subsequently able to develop a close bond with their adoptive parents:
Select one:

A. only if they are adopted by 12 months of age.
B. only if they are adopted by 24 months of age.
C. even if they are not adopted until 4 to 6 years of age.
D. only if they had established a strong bond with a biological parent prior to being institutionalized.

A

Tizard et al.’s research indicates that “late adoptees” (children placed in adoptive homes after age 4) are able to form strong bonds when the adoption occurs by six years of age.

a. Incorrect See explanation for response c.
b. Incorrect See explanation for response c.
c. CORRECT Although these researchers found that children adopted between the ages of 4 and 6 did develop strong bonds with their adoptive parents, their data also revealed that these children had higher-than-normal rates of emotional and social problems in childhood and adolescence, including an excessive need for adult attention and “overfriendliness” toward unfamiliar adults [J. Hodges and B. Tizard, Social and family relationships of ex-institutionalized adolescents, Journal of Child Psychology and Psychiatry, 30, 77-97, 1989].
d. Incorrect See explanation for response c.

The correct answer is: even if they are not adopted until 4 to 6 years of age.

146
Q

Freud’s psychosexual theory of development is based on the premise that, in each stage:
Select one:

A. efforts to obtain pleasure center on a different part of the body.
B. a different psychological need is “prepotent.”
C. the source of anxiety is related to a different psychosocial conflict.
D. the source of anxiety is related to a different psychic need.

A

Freud’s psychosexual theory emphasizes the impact of the sex drive (libido) on development.

a. CORRECT In each stage of development, a different part of the body is most sensitive to erotic stimulation.
b. Incorrect See explanation above.
c. Incorrect See explanation above.
d. Incorrect See explanation above.

The correct answer is: efforts to obtain pleasure center on a different part of the body.

147
Q

Egan and Perry’s (2001) multidimensional model of gender identity consists of five components - membership knowledge, gender typicality, gender contentedness, felt pressure, and:
Select one:

A. intergroup bias.
B. gender stability.
C. integrative awareness.
D. gender constancy.

A

This may have been one of those “distant galaxy questions” for you. If so, a good approach would have been to see if you could eliminate any answers because you know they refer to a different theory. Doing so would increase your chance of choosing the correct answer.

a. CORRECT This answer lists the fifth component identified by S. K. Egan and D. G. Perry (Gender identity: A multidimensional analysis with implications for psychosocial adjustment. Developmental Psychology, 37, 451-463, 2001). As defined by these authors, membership knowledge refers to the individual’s knowledge about his/her own gender; gender typicality is the degree to which the individual perceives his/her characteristics to be similar to those of individuals of the same gender; gender contentedness is the extent to which the individual is satisfied with his/her own gender; felt pressure is the degree to which the individual feels pressure to conform to gender group norms; and intergroup bias refers to the belief that one’s same-sex group is superior to the other sex group.
b. Incorrect Gender stability is the second stage in Kohlberg’s model of gender identity development.
c. Incorrect Integrative awareness is the final stage of Atkinson, Morten, and Sue’s (1993) Racial/Cultural Identity Development Model.
d. Incorrect Gender constancy is the third stage in Kohlberg’s model of gender identity development.

The correct answer is: intergroup bias.

148
Q

Freud proposed that, during the ________ stage of psychosexual development, an individual experiences guilt as a result of the conflict between his or her unacceptable desires and a fear of being punished for having those desires.
Select one:

A. anal
B. phallic
C. latency
D. genital

A

For the licensing exam, you want to be familiar with the basic characteristics of each of Freud’s stages of psychosexual development - oral, anal, phallic, latency, and genital.

a. Incorrect During the anal stage, libido (sexual energy) is centered in the anus and the primary task is control of bodily wastes.
b. CORRECT During the phallic stage, libido is centered in the genitals and the primary task is resolution of the Oedipal or Electra conflict, which is marked by a desire for the opposite-sex parent and a view of the same-sex parent as a rival. Feelings of guilt arise during this stage as the superego emerges and conscience develops.
c. Incorrect During the latency stage, libidinal energy is diffuse rather than focused on any one area of the body, and the emphasis is on developing social skills rather than on achieving sexual gratification.
d. Incorrect During the genital stage, libido is again centered in the genitals but the successful outcome at this stage occurs when sexual desire is blended with affection to produce mature sexual relationships.

The correct answer is: phallic

149
Q

The effects of teratogens on the developing organism vary with the type of teratogen and the organ system. However, in general, these agents are most likely to cause severe structural damage:
Select one:

A. during the first eight weeks of development.
B. during the 9th through 18th weeks of development.
C. during the second trimester.
D. during the third trimester.

A

The effects of teratogens on fetal development depend on several factors including the type and amount of the substance, the duration of exposure, the time of exposure, and the mother’s physiological condition.

a. CORRECT Teratogens can have a negative effect on fetal development during the entire pregnancy, but their consequences are much more severe during critical periods of development when the organ system is growing most rapidly. The critical period varies from organ to organ, but generally occurs within the first eight weeks of development. (An exception is the central nervous system, which has a critical period that extends into the 16th week of development.)
b. Incorrect See explanation for response a.
c. Incorrect See explanation for response a.
d. Incorrect See explanation for response a.

The correct answer is: during the first eight weeks of development.

150
Q

Which of the developmental changes is most likely to occur in males between the ages of 40-45?
Select one:

A. a decrease in work satisfaction
B. significant personality change and emotional disturbance due to the “mid-life” crisis
C. a shift in perspective from “time-since-birth” to “time-left-to-live”
D. peaceful acceptance of the impending black doom of the grim reaper

A

According to Levinson, the mid-life transition, or the transition between early and middle adulthood, occurs between the ages of 40-45.

a. Incorrect Research suggests that middle aged men (and women) are more satisfied with their work than younger adults.
b. Incorrect Contrary to popular belief, research suggests that mid-life is not a time of crisis for most people.
c. CORRECT As a result of an increasing awareness of mortality, the mid-life transition is marked by a shift in awareness from “time-since-birth” to “time-left-to-live.”
d. Incorrect Patently absurd and potentially offensive. responses such as this one are not likely to be correct.

The correct answer is: a shift in perspective from “time-since-birth” to “time-left-to-live”

151
Q

Taddio and colleagues (1997, 2005) examined pain perception in newborns and found that, when newborn boys were not given anesthesia during circumcision:
Select one:

A. they showed no adverse reactions, suggesting that they did not experience pain.
B. they showed an adverse reaction but exhibited no subsequent effects of this experience.
C. they showed an adverse reaction followed by increased sensitivity to pain in subsequent months.
D. they showed an adverse reaction followed by decreased sensitivity to pain in subsequent months.

A

Contrary to what was previously believed, research has confirmed that newborns are sensitive to pain.

a. Incorrect See explanation for response c.
b. Incorrect See explanation for response c.
c. CORRECT Taddio et al. found that newborns who were not given anesthesia during circumcision displayed a more adverse reaction to a vaccination several months later than did those who were given anesthesia. Other research has confirmed that exposure to painful medical procedures during early infancy can affect later pain perception, although the exact effects may depend on whether the infant was full-term or premature.
d. Incorrect See explanation for response c.

The correct answer is: they showed an adverse reaction followed by increased sensitivity to pain in subsequent months.

152
Q

For children of divorced parents, increased frequency of contact with the noncustodial father:
Select one:

A. is consistently associated with more behavior problems and lower academic achievement.
B. is consistently associated with fewer behavior problems and better academic achievement.
C. is associated with fewer behavior problems and better academic achievement only when the father is supportive and authoritative.
D. is unrelated to severity of behavior problems or level of academic achievement regardless of the father’s supportiveness and parenting style.

A

The long-term impact of contact with the noncustodial father on child outcomes was investigated by E. M. Hetherington [An overview of the Virginia Longitudinal Study of divorce and remarriage with a focus on early adolescence, Journal of Family Psychology, 7(1), 39-56, 1993].

a. Incorrect See explanation for response c.
b. Incorrect See explanation for response c.
c. CORRECT Hetherington found that frequency of contact with the noncustodial father itself was not predictive of child outcomes. However, a greater frequency of contact was associated with better outcomes – especially for boys – when the father was supportive, authoritative in terms of parenting style, and did not frequently engage in open conflict with his ex-spouse in front of the child.
d. Incorrect See explanation for response c.

The correct answer is: is associated with fewer behavior problems and better academic achievement only when the father is supportive and authoritative.

153
Q

From the perspective of Vygotsky’s sociocultural theory, “private speech” is:
Select one:

A. a manifestation of a young child’s egocentrism.
B. self-directed speech that guides the child’s behavior.
C. evidence that language acquisition is a creative (rather than imitative) process.
D. unrelated to cognitive development.

A

Private speech refers to a preschool child’s talking aloud to him/herself while engaging in daily activities.

a. Incorrect This is Piaget’s interpretation of private speech.
b. CORRECT Vygotsky believed that private (self-directed) speech is essential for cognitive development and is a source of self-guidance for young children.
c. Incorrect This describes figurative speech (the deliberate use of words in a figurative, rather than literal, way).
d. Incorrect See explanation for response b.

The correct answer is: self-directed speech that guides the child’s behavior.

154
Q

Klinefelter’s syndrome is a genetic disorder that:
Select one:

A. affects males and is caused by the presence of an extra X chromosome.
B. affects males and is caused by the presence of an extra Y chromosome.
C. affects females and is caused by the presence of an extra X chromosome.
D. affects females and is caused by the presence of an extra Y chromosome.

A

Klinefelter’s syndrome is one of the most common genetic abnormalities.

a. CORRECT A male with Klinefelter’s syndrome has two or more X (female) chromosomes in addition to a single Y chromosome. Symptoms include small testicles, infertility, enlarged breast tissue, scant facial/body hair, and a decreased libido.
b. Incorrect See explanation above.
c. Incorrect See explanation above.
d. Incorrect See explanation above.

The correct answer is: affects males and is caused by the presence of an extra X chromosome.

155
Q

The “storm and stress theory” of adolescent mental health proposes that most adolescents experience extensive physical, social, and psychological turmoil. Research conducted in the past three or four decades on this topic:
Select one:

A. confirms that most adolescents do experience substantial “storm and stress.”
B. demonstrates that “storm and stress” is characteristic of adolescents in westernized cultures only.
C. indicates that, for the large majority of individuals, adolescence is not a time of significant “storm and stress.”
D. shows that “storm and stress” is largely due to increases in family conflicts during the adolescent period.

A

Research conducted in the past few decades has largely refuted Hall’s (1904) proposition that adolescence is a time of substantial “storm and stress.”

a. Incorrect See explanation for response c.
b. Incorrect See explanation for response c.
c. CORRECT Studies suggest that, while adolescents may exhibit greater emotional lability, only about 10 to 20% of adolescents exhibit some type of severe emotional turmoil, which is approximately the same percent as found in the adult population.
d. Incorrect In fact, contrary to the popular view of adolescence, the research suggests that family conflicts do not actually increase substantially during this period.

The correct answer is: indicates that, for the large majority of individuals, adolescence is not a time of significant “storm and stress.”

156
Q

Research on attachment has shown that young victims of child abuse are most likely to exhibit which of the following attachment patterns?
Select one:

A. disengaged
B. avoidant
C. ambivalent
D. disorganized

A

Ainsworth distinguished between three attachment patterns (secure, insecure/ambivalent, and insecure/avoidant), and Main subsequently identified the disorganized/disoriented pattern.

a. Incorrect See explanation for response d.
b. Incorrect See explanation for response d.
c. Incorrect See explanation for response d.
d. CORRECT Mary Main found that about 80% of maltreated children exhibit a disorganized/disoriented pattern of attachment.

The correct answer is: disorganized

157
Q

Kochanska and Knaack (2003) conclude that effortful (inhibitory) control is an important contributor to the early development of conscience. Their research found that effortful control becomes a stable, coherent trait by ________ months of age.
Select one:

A. 8 to 12
B. 12 to 18
C. 18 to 30
D. 33 to 45

A

Research on the development of conscience indicates that its precursors are apparent during the second and third years of life.

a. Incorrect See explanation for response d.
b. Incorrect See explanation for response d.
c. Incorrect See explanation for response d.
d. CORRECT G. Kochanska and A. Knaack found that effortful control is modestly coherent across tasks at age 22 months but does not become highly coherent (trait-like) until 33 months of age. They also found that children with higher effortful control at 22 to 45 months had stronger consciences at 56 months of age [Effortful control as a personality characteristic of young children: Antecedents, correlates, and consequences, Journal of Personality, 71(6), 1087-1112, 2003].

The correct answer is: 33 to 45

158
Q

The age of onset of separation anxiety varies from child to child but is usually between six and eight months of age. It then peaks in intensity at about ______ months and thereafter declines.
Select one:

A. 10 to 14
B. 14 to 18
C. 18 to 24
D. 24 to 28

A

The age at which developmental milestones are reached vary considerably from child to child, so it is difficult to make generalizations. Most (but, unfortunately, not all) authorities list it as beginning at about 6 to 8 months of age.

B. CORRECT - The intensity of separation anxiety continues to increase during the first half of the second year of age, and subsequently declines.

The correct answer is: 14 to 18

159
Q

Research on infantile amnesia has shown that:
Select one:

A. children do not develop memories for autobiographical events until age 3 or 4.
B. the loss of early memories is universal and is unrelated to gender, culture, or type of event.
C. memories for autobiographical events that occurred prior to age 3 or 4 remain stable over the lifespan.
D. the ability to recall memories for autobiographical events that occurred prior to age 3 or 4 varies over the lifespan.

A

Infantile amnesia refers to a person’s inability to recall events that occurred prior to about age 3 or 4. While early research suggested that the loss of early memories is absolute, subsequent studies indicate that many older children, adolescents, and adults are able to recall some events that took place prior to age 3 or 4.

a. Incorrect Children under the age of 3 do develop memories for autobiographical events, but many of these memories are “lost” in subsequent years.
b. Incorrect There is evidence that the loss of memory is, in fact, related to gender, cultural background, type of event, and other factors.
c. Incorrect See explanation for response d.
d. CORRECT The studies on infantile amnesia have found that memories for events that occurred prior to age 3 or 4 are limited but not completely absent. In addition, the recall of early memories varies across the lifespan. J. M. Rybash and K. L. Hrubi, for example, found that older adults typically have later-occurring first memories than younger adults do (Psychometric and psychodynamic correlates of first memories in younger and older adults, Gerontologist, 37, 581-587, 1997]; and J. Kilstrom and J. Harackiewicz found that, in adolescence, reported first memories vary over time (The earliest recollections: A new survey, Journal of Personality, 50, 134-148, 1982).

The correct answer is: the ability to recall memories for autobiographical events that occurred prior to age 3 or 4 varies over the lifespan.

160
Q

Classically conditioned reflexes are an example of implicit memory and are mediated primarily by the:
Select one:

A. suprachiasmatic nucleus
B. hippocampus
C. arcuate fasciculus
D. cerebellum

A

Implicit memories are recalled without conscious awareness. Skills and habits, emotional associations, and classically conditioned reflexes are examples of implicit memories.

a. Incorrect The suprachiasmatic nucleus controls circadian rhythms.
b. Incorrect The hippocampus is essential for the formation of explicit memories.
c. Incorrect The arcuate fasciculus is a bundle of fibers that connects Broca’s area with Wernicke’s area.
d. CORRECT Conditioned reflexes rely primarily on the cerebellum. For example, the cerebellum played a key role in Pavlov’s research in which dogs were conditioned to respond to a bell with salivation. Other areas of the brain involved in implicit memory include the amygdala (which mediates emotional associations) and the striatum (which is responsible for memories of skills and habits).

The correct answer is: cerebellum

161
Q

Which of the following brain structures is responsible for interhemispheric communication?
Select one:

A. precentral gyrus
B. arcuate fasciculus
C. cingulate gyrus
D. corpus callosum

A

Interhemispheric communication refers to communication between the right and left hemispheres of the brain.

a. Incorrect The precentral gyrus is located in the frontal lobe and contains the motor cortex.
b. Incorrect The arcuate fasciculus connects Wernicke’s area to Broca’s area.
c. Incorrect The cingulate gyrus is part of the limbic system and is involved in the processing and formation of emotions.
d. CORRECT The corpus callosum is the primary bundle of nerve fibers connecting the left and right hemispheres. Its role in transferring information between the hemispheres was identified in the 1950s by Myers and Sperry.

The correct answer is: corpus callosum

162
Q

Taking _______ to treat the symptoms of the flu, chickenpox, or other viral illness can produce Reyes syndrome.
Select one:

A. ibuprofen
B. aspirin
C. an antihistamine
D. an antibiotic

A

Reyes syndrome is a rare disorder that is most common in children and adolescents, usually affects the brain and liver, and can be fatal.

a. Incorrect See explanation for response b.
b. CORRECT Although the cause of Reyes syndrome is still unknown, it has been linked to the use of aspirin and other salicylate-containing medications during or shortly after a viral, fever-causing illness. Early symptoms include persistent vomiting, listlessness, and drowsiness.
c. Incorrect See explanation for response b.
d. Incorrect See explanation for response b.

The correct answer is: aspirin

163
Q

The James-Lange theory of emotions emphasizes the role of:
Select one:

A. visceral and muscular reactions.
B. thalamic stimulation of the cortex.
C. limbic system structures.
D. past experiences with emotion-arousing stimuli.

A

The James-Lange theory is classified as a peripheralist theory of emotion.

a. CORRECT According to the James-Lange theory, “we are afraid because we run.” In other words, emotions are perceptions of bodily reactions.
b. Incorrect This sounds more like the Cannon-Bard theory which emphasizes the brain mechanisms that mediate emotion (especially the thalamus and cerebral cortex).
c. Incorrect See explanation above.
d. Incorrect See explanation above.

The correct answer is: visceral and muscular reactions.

164
Q

The “rebound effect” associated with benzodiazepine use refers to:
Select one:

A. the persistence of symptoms despite an increase in the dose of the drug.
B. the re-emergence of symptoms after long-term use of the drug.
C. an initial paradoxical increase in symptoms.
D. a temporary increase in severity of symptoms when the drug is discontinued.

A

Discontinuation of a benzodiazepine may result in a variety of undesirable symptoms including a “rebound effect.”

a. Incorrect See explanation for response d.
b. Incorrect See explanation for response d.
c. Incorrect See explanation for response d.
d. CORRECT A person taking a benzodiazepine for anxiety, for instance, may experience “rebound anxiety” when he/she stops taking the drug. Rebound anxiety occurs within hours to days after the drug is stopped and is more severe than the original symptoms.

The correct answer is: a temporary increase in severity of symptoms when the drug is discontinued.

164
Q

If an adult takes phenobarbital every night for six weeks and then abruptly stops taking the drug, what will be the likely result:
Select one:

A. she will not experience any change in REM sleep.
B. she will experience an increase in REM sleep.
C. she will experience an inability to sleep.
D. she will experience an increased need for sleep.

A

Answer C is correct: Phenobarbital, a barbiturate, was originally used as a sleeping aid and an anticonvulsant. Due to the recognition of their untoward side effects and the development of safer drugs, barbiturates are now rarely prescribed. Barbiturate use causes a decrease in REM sleep and abrupt cessation of use can cause a “REM rebound,” as well as an inability to sleep, poor quality sleep, and nightmares.

Answer C is a more comprehensive answer and thus the “best” answer of those provided. Because barbiturate use causes a decrease in REM overall, stopping barbituates can have a temporary increase in REM as a response (“REM rebound”). But REM rebound is one aspect of the side effects. Other side effects include poor quality sleep, inability to sleep at all, etc. So an inability to sleep is more characteristic of barbiturate withdrawal than increased REM sleep.

The correct answer is: she will experience an inability to sleep.

165
Q

Bilateral lesions in which of the following areas of the brain is most likely to result in a loss of the fear response without loss of other emotional responses?
Select one:

A. amygdala
B. hippocampus
C. suprachiasmatic nucleus
D. thalamus

A

For the exam, you want to be familiar with the major functions of the areas of the brain listed in the answers to this question.

a. CORRECT The case of “patient SM” has confirmed that the amygdala is responsible for the experience of fear in response to frightening stimuli. SM has focal bilateral amygdala lesions as the result of a rare disease. While she is able to experience other emotions, she does not experience fear. See J. S. Feinstein et al., The human amygdala and the induction and experience of fear, Current biology, 21(1), 34-38, 2010.
b. Incorrect The hippocampus plays a role in learning and memory, including converting short-term declarative memories to long-term memories.
c. Incorrect The suprachiasmatic nucleus mediates the sleep-wake cycle and other circadian rhythms.
d. Incorrect The thalamus is involved in motor activity, language, and memory and acts as a “relay station” for all of the senses except olfaction.
The correct answer is: amygdala

166
Q

REM (rapid eye movement) sleep accounts for about ___ of a newborn’s total sleep period and ___ of an adult’s total sleep period.
Select one:

A. 75%; 10%
B. 50%; 20%
C. 30%; 40%
D. 10%; 50%

A

Sleep patterns vary over the lifespan, and the pattern of an infant is quite different from that of an adult.

a. Incorrect See explanation for response b.
b. CORRECT Infants not only begin a sleep period with REM sleep but also spend more time in REM sleep, with REM sleep representing about 50% of their total sleep period. In contrast, adults begin a sleep period with non-REM sleep and spend less time (about 20%) in REM sleep.
c. Incorrect See explanation for response b.
d. Incorrect See explanation for response b.

The correct answer is: 50%; 20%

167
Q

Melzack and Wall’s (1965) gate-control theory is useful for understanding how:
Select one:

A. non-pain stimuli can block or modify the sensation of pain.
B. signals are transferred from one neuron to an adjacent neuron.
C. newly acquired information interferes with the ability to recall previously acquired information.
D. information from short-term memory is transferred to long-term m

A

As long as you have gate-control theory associated with response to pain, you would have been able to quickly identify the correct answer to this question.

a. CORRECT Gate-control theory explains why rubbing your elbow after accidentally hitting it against a hard surface helps reduce the sensation of pain.
b. Incorrect This answer describes synaptic transmission.
c. Incorrect This describes the interference theory of forgetting.
d. Incorrect This describes elaborative rehearsal and other effective memory strategies.

The correct answer is: non-pain stimuli can block or modify the sensation of pain.

168
Q

Prosopagnosia is believed to be due to lesions in the:
Select one:

A. junction of the occipital, temporal, and parietal lobes.
B. junction of the frontal and occipital lobes.
C. precentral gyrus.
D. central sulcus.

A

Knowing that prosopagnosia is a type of visual agnosia and that the occipital lobe mediates visual processing would have helped you identify the correct answer to this question.

a. CORRECT The occipital lobe is involved in visual processing, while the temporal lobe mediates long-term memory; and damage to these areas plus the parietal lobe may produce an inability to recognize familiar faces.
b. Incorrect See explanation above.
c. Incorrect The precentral gyrus contains the primary motor cortex and is not involved in visual processing.
d. Incorrect The central sulcus separates the frontal and parietal lobes.

The correct answer is: junction of the occipital, temporal, and parietal lobes.

169
Q

Agranulocytosis is a potential side effect of clozapine, carbamazepine, and a number of other psychiatric drugs. Early symptoms of this disorder include:
Select one:

A. fever, sore throat, mouth ulcers, and lethargy.
B. nausea, diarrhea, vomiting, and a metallic taste in the mouth.
C. sweating, palpitations, headache, tremulousness, and cardiac arrhythmia.
D. constricted pupils, decreased visual acuity, sweating, constipation, and nausea.

A

Agranulocytosis is caused by a failure of the bone marrow to produce a sufficient number of certain white blood cells, which increases the body’s susceptibility to infection.

a. CORRECT Fever, sore throat, chills, mouth ulcers, lethargy, and weakness are early signs of agranulocytosis.
b. Incorrect These are potential gastrointestinal side effects of lithium.
c. Incorrect These symptoms may occur when a beta-blocker is abruptly discontinued.
d. Incorrect These are signs of narcotic-analgesic use.

The correct answer is: fever, sore throat, mouth ulcers, and lethargy.

170
Q

Brain imaging techniques have linked ADHD to smaller-than-normal globus pallidus, caudate nucleus, and:
Select one:

A. temporal lobe.
B. parietal lobe.
C. prefrontal cortex.
D. entorhinal cortex.

A

Functional and structural brain imaging techniques have linked ADHD to abnormalities in several areas of the brain.

a. Incorrect See explanation for response c.
b. Incorrect See explanation for response c.
c. CORRECT The right frontal lobe (especially the prefrontal cortex) and the caudate nucleus and globus pallidus (structures of the basal ganglia) tend to be smaller in individuals with ADHD and to have lower-than-normal levels of metabolic activity.
d. Incorrect See explanation for response c.

The correct answer is: prefrontal cortex.

171
Q

Which of the following drugs would be most useful for reducing neuropathic pain?
Select one:

A. amitriptyline
B. fluoxetine
C. clozapine
D. tacrine hydrochloride

A

Neuropathic pain is chronic pain that is due to a nervous system injury or dysfunction. The first-line treatments for neuropathic pain are analgesics, which include certain antidepressants, anticonvulsants, opioids, and local anesthetics.

a. CORRECT Antidepressant drugs that increase levels of both serotonin and norepinephrine not only reduce the depression that often accompanies neuropathic pain but also have analgesic properties. The tricyclic amitriptyline is one of the oldest and most widely used tricyclic drug for neuropathic pain.
b. Incorrect Although SSRIs are useful for alleviating the depression associated with neuropathic pain, they do not have analgesic properties.
c. Incorrect The neuroleptics have not been found useful for alleviating neuropathic pain.
d. Incorrect Tacrine hydrochloride is a cognitive-enhancer that is used to reduce cognitive deficits associated with Alzheimer’s dementia.

The correct answer is: amitriptyline

172
Q

Parkinson’s disease is associated primarily with a degeneration of cells in the:
Select one:

A. reticular formation.
B. substantia nigra.
C. amygdala.
D. temporal and occipital lobes.

A

Parkinson’s disease is a progressive neurological condition characterized by tremor, rigidity, bradykinesia, and postural instability.

a. Incorrect See explanation for response b.
b. CORRECT The substantia nigra is part of the basal ganglia and is the primary site of neuronal loss in Parkinson’s disease. Additional information on Parksinson’s disease is provided in the Physiological Psychology and Psychopharmacology chapter of the written study materials.
c. Incorrect See explanation for response b.
d. Incorrect See explanation for response b.

The correct answer is: substantia nigra.

173
Q

Which of the following drugs are most effective for individuals who have received a diagnosis of Major Depressive Disorder with Atypical Features?
Select one:

A. MAOIs or SSRIs
B. MAOIs or tricyclics
C. SSRIs or tricyclics
D. tricyclics or neuroleptics

A

The choice of the best antidepressant for a particular individual depends on several factors including the nature of his/her symptoms.

a. CORRECT Atypical depression includes reversed vegetative symptoms (increase sleep and appetite), marked mood reactivity, phobic symptoms, and/or a sense of severe fatigue. The MAOIs and SSRIs have been found most effective for these symptoms.
b. Incorrect The response rate for tricyclics is 35 to 50%, while the response rate for MAOIs and SSRIs is 55 to 75%.
c. Incorrect See explanation above.
d. Incorrect See explanation above.

The correct answer is: MAOIs or SSRIs

174
Q

As the result of a traumatic head injury sustained in a motor vehicle accident, a middle-aged man has mild impairments in judgment, insight, and planning and reduced sexual interest. Damage to which of the following areas of the brain is most likely responsible for these deficits?
Select one:

A. dorsolateral prefrontal region
B. orbitofrontal region
C. medial temporal region
D. posterior parietal region

A

Although traumatic brain injury can produce diffuse brain injury, the frontal and temporal lobes are the areas of the cortex that are usually most adversely affected.

a. CORRECT The dorsolateral prefrontal region mediates executive cognitive functions. Damage to this area is associated with deficits in attention, planning, problem-solving, and other higher-order cognitive abilities as well as reduced sexual interest and apathy.
b. Incorrect The orbitofrontal region is involved in inhibition. Damage to this area may produce disinhibition, impaired social insight, and inappropriate affect.
c. Incorrect The temporal lobes are involved in learning and memory, and damage to the medial temporal lobes along with certain adjacent structures can produce amnesia.
d. Incorrect The posterior parietal region is a major sensorimotor area of the brain, and damage can produce apraxia or contralateral neglect.

The correct answer is: dorsolateral prefrontal region

175
Q

A menopausal woman who is considering hormone replacement therapy (HRT) should be advised that this treatment may do all of the following except:
Select one:

A. reduce the risk for osteoporosis.
B. reduce or eliminate “hot flashes.”
C. increase sex drive.
D. reduce mood symptoms.

A

HRT continues to be controversial because it is not only associated with a number of benefits but also with some negative side effects.

a. Incorrect This is one of the major advantages of HRT.
b. Incorrect HRT is effective for alleviating hot flashes.
c. CORRECT The results of research investigating the effects of HRT on libido are inconsistent, and the best conclusion that can be drawn at this time is that it does not increase sex drive.
d. Incorrect HRT does alleviate mood swings.

The correct answer is: increase sex drive.

176
Q

A researcher for a major drug company is going to test a new anti-depressant for market viability, why would the researcher choose a Practical Clinical Trial (PCT) over another method?
Select one:

A. A PCT is intended to answer a single, clearly formulated research question by concentrating on a major outcome of direct clinical significance.
B. A PCT provides multiple answers which gives it a complex depth for which a researcher may analyze for more effective results.
C. A PCT tends to provide a clear, formulated answer that the researcher can utilize to demonstrate cause and effect between the three major variables being studied.
D. A PCT is a rich and complex system of analysis for which a researcher may utilize to test multiple interventions.

A

The correct answer is A.

PCTs tend to be large, simple trials with many hundreds, if not thousands, of patients. PCTs frequently provide relevant information on common conditions, such as depression or anxiety, or the availability of a variety of treatments, both pharmacological and psychosocial. Answers B, C, and D are incorrect because PCTs tend to be simple, not complex; there is not a set number of variables being tested; and is practical and therefore very useful in the testing for various psychological interventions and treatments.

The correct answer is: A PCT is intended to answer a single, clearly formulated research question by concentrating on a major outcome of direct clinical significance.

177
Q

Which of the following systems has/have been linked to Social Phobias?
Select one:

A. serotonergic only
B. serotonergic and dopaminergic
C. cholinergic only
D. cholinergic and glutamatergic

A

Several neurotransmitter abnormalities have been linked to the anxiety disorders, but serotonin and dopamine have been most closely associated with Social Phobia.

a. Incorrect See explanation for response b.
b. CORRECT Theories of Social Phobia focus on the role of serotonergic and dopaminergic pathways in the brain. See e.g., J. A. Gray, The neuropsychology of anxiety, Oxford, Oxford University Press, 1982.
c. Incorrect See explanation for response b.
d. Incorrect See explanation for response b.

The correct answer is: serotonergic and dopaminergic

178
Q

The venom of the black widow spider is believed to cause violent and uncontrollable muscle contractions by affecting the activity of which of the following neurotransmitters?
Select one:

A. acetylcholine
B. norepinephrine
C. enkephalin
D. serotonin

A

To answer this question, you must know which of the neurotransmitters listed is involved in the control of the skeletal muscles. Alternatively, you might be able to choose the right answer through a process of elimination.

a. CORRECT Acetylcholine is found in the peripheral nervous system, the spinal cord, and certain regions of the brain. In the peripheral nervous system, it activates both muscles and glands. The venom of the black widow spider is believed to work by causing a continuous flow of acetylcholine into the neuromuscular junction, resulting in uncontrollable muscle contractions.
b. Incorrect Norepinephrine, a catecholamine, is most associated with personality, mood, and drive states.
c. Incorrect Enkephalin, one of the endogenous morphine substances found in the brain, is believed to be involved in the mediation of pain.
d. Incorrect Serotonin is believed to be involved in the suppression of the ARAS, the regulation of temperature, hunger, and aggression, and the affective disorders and schizophrenia.

The correct answer is: acetylcholine

179
Q

Research on the role of hormones on sexual arousal suggests that:
Select one:

A. estrogen is responsible for arousal in females, while androgen is responsible for arousal in males.
B. estrogen is responsible for arousal in males, while androgen is responsible for arousal in females.
C. estrogen is responsible for arousal in both males and females.
D. androgen is responsible for arousal in both females and males.

A

Although research on the role of hormones in sexual behavior remains somewhat inconclusive, there is sufficient evidence to conclude that certain hormones do play a role in human sexual arousal.

a. Incorrect See explanation for response d.
b. Incorrect See explanation for response d.
c. Incorrect See explanation for response d,
d. CORRECT The research suggests that estrogen plays an insignificant role in female sexual motivation and arousal and that androgen, which is produced by the adrenal cortex, plays an important role in the sexual functioning of both males and females.

The correct answer is: androgen is responsible for arousal in both females and males.

180
Q

Precocious puberty, or the development of secondary sex characteristics prior to age 8 in girls and 8.5 in boys, has been linked to premature awakening of the:
Select one:

A. hypothalamic-pituitary axis.
B. hypothalamic-pituitary-adrenal axis.
C. hippocampal-anterior thalamic axis.
D. mesocorticolimbic-dopaminergic axis.

A

You would have been able to choose the correct response for this question as long as you knew that the hypothalamus is involved in the release of the sex hormones and that the adrenal glands are associated with stress (but not sex) hormones (response b).

a. CORRECT The hypothalamic-pituitary axis is also known as the hypothalamic-pituitary-gonadal axis and is involved in sexual maturation.
b. Incorrect The hypothalamic-pituitary-adrenal axis is involved in stress and anxiety.
c. Incorrect The hippocampal-anterior thalamic axis is involved in memory (especially episodic memory).
d. Incorrect The mesocorticolimbic-dopaminergic system is the brain’s “reward pathway” and mediates the reinforcing effects of drugs.

The correct answer is: hypothalamic-pituitary axis.

181
Q

Patients who have recently started taking naltrexone (ReVia) as a treatment for alcohol dependence are most likely to experience which of the following drug side effects?
Select one:

A. dizziness, ataxia, visual disturbances, nausea, and rash
B. blurred vision, sexual dysfunction, weight gain, edema, and tremor
C. shortness of breath, increased dreaming, nausea, diarrhea, and bradycardia
D. abdominal cramping, nausea, insomnia, nervousness, and headache

A

As a treatment for alcohol dependence, naltrexone exerts its therapeutic effects by blocking the craving for and reinforcing effects of alcohol.

a. Incorrect These side effects are characteristic of carbamazepine, an anti-seizure drug that is also used to treat bipolar disorder.
b. Incorrect These are possible side effects of the MAOI inhibitors.
c. Incorrect These are possible side effects of the beta blocker propranolol.
d. CORRECT These are potential side effects of naltrexone. Note, however, that most people experience few side effects when taking naltrexone as a treatment for alcohol dependence.

The correct answer is: abdominal cramping, nausea, insomnia, nervousness, and headache

182
Q

A person with transcortical aphasia will:
Select one:

A. be able to communicate verbally but will have deficits in comprehension.
B. have intact comprehension but exhibit a number of grammatical errors and labored speech.
C. have significant difficulties in both producing and comprehending speech.
D. exhibit minor peculiarities in speech and a total lack of verbal comprehension.

A

Transcortical aphasia results when the major speech areas in the left (dominant) hemisphere become isolated from other cortical areas due to a loss of blood supply.

a. Incorrect See explanation for response c.
b. Incorrect See explanation for response c.
c. CORRECT A person with transcortical aphasia is unable to verbally describe what he or she sees or desires and also has deficits in language comprehension. However, the person may be able to generate automatic-like responses (e.g., singing) and be able to repeat statements made by others. When the damage is limited to certain areas of the brain, the individual may exhibit a more limited type of transcortical aphasia - i.e., transcortical motor aphasia (intact comprehension with difficulties in producing speech) or transcortical sensory aphasia (difficulties in comprehension only).
d. Incorrect See explanation for response c.

The correct answer is: have significant difficulties in both producing and comprehending speech.

183
Q

A patient who has recently been prescribed Prozac (fluoxetine) as a treatment for depression:
Select one:

A. should be warned that she may temporarily experience some mild cognitive impairment.
B. should be advised that she will temporarily experience dry mouth, constipation, urinary retention, and blurred vision.
C. should be warned that sleep and anxiety problems may temporarily increase.
D. must avoid tyramine-rich foods.

A

Prozac and the other SSRIs are associated with fewer side effects than the tricyclics.

a. Incorrect Cognitive impairment is more likely with a tricyclic.
b. Incorrect These are side effects of the tricyclics.
c. CORRECT One disadvantage of the SSRI’s is that they may temporarily exacerbate sleep and anxiety problems.
d. Incorrect This is true about the MAOIs, not the SSRIs.

The correct answer is: should be warned that sleep and anxiety problems may temporarily increase.

184
Q

On the MMPI-2, an attempt to “fake good” is suggested by which of the following?
Select one:

A. elevated F scale score with low L and K scale scores.
B. elevated F and L scale scores with a low K scale score.
C. low F scale score with elevated L and K scale scores.
D. low K scale score with a moderately high L scale score and a very high F scale score.

A

To identify the correct answer to this question, you need to be familiar with the interpretations of scores on the MMPI-2’s F, L, and K scales.

a. Incorrect A high F scale score with low L and K scale scores suggests an attempt to “fake bad.”
b. Incorrect This pattern is not indicative of an attempt to “fake good.”
c. CORRECT A low score on the F (infrequency) scale indicates an absence of psychopathology, social conformity, or an attempt to “fake good”; a high score on the L (lie) scale suggests an attempt to “fake good,” conformity, or denial; and a high score on the K (correction) scale suggests an attempt to “fake good,” denial, or a lack of insight.
d. Incorrect Although an elevated L scale score suggests an attempt to “fake good,” a high F scale score and a low K scale score may indicate an attempt to “fake bad.”

The correct answer is: low F scale score with elevated L and K scale scores.

185
Q

Which of these professionals would use a psychophysiological measure in an appropriate research setting and approach?
Select one:

A. Psychologist who is researching emotional states and using surveys to gather the data
B. Psychiatrist who is researching the effect of anger on heart rate in a laboratory setting
C. Psychologist who is measuring the long-term effects of anti-psychotic medication usage in young adults via longitudinal case study
D. Industrial and Organizational Psychologist who is measuring attitudes in the workplace as it relates to satisfaction and happiness with each employees role

A

The correct answer is B. This approach measures physiological functions such as: heart rate, skin perspiration, facial muscles, which can often describe the emotional state of an individual. The physiological reactions are monitored and measured with sensors attached to the participant. The main strength of this approach is the ability to investigate and analyze momentary experiences without intervening in the interaction occurring. Answers A, C and D are incorrect as the research methods do not necessarily correlate with psychophysiological research, but rather case studies, interview techniques and survey reporting.

The correct answer is: Psychiatrist who is researching the effect of anger on heart rate in a laboratory setting

186
Q

When scoring the Mini-Mental State Exam (MMSE), it’s important to keep in mind that the use of the standard cutoff score (below 24) as an indication of cognitive impairment may produce a spuriously high rate of false positives for:
Select one:

A. adults with a college education.
B. adults with Alzheimer’s disease.
C. White adults.
D. African American and Hispanic adults.

A

The MMSE is a widely used screening tool for Major Neurocognitive Disorder, and a score of 23 or less is ordinarily considered to provide preliminary evidence of this disorder.

a. Incorrect The opposite is true. The MMSE may fail to detect Major Neurocognitive Disorder in highly educated older adults but may produce a high rate of false positives for individuals with less than a high school education.
b. Incorrect The MMSE is considered to be an accurate screening device for Alzheimer’s disease.
c. Incorrect See explanation for answer d.
d. CORRECT Several studies have confirmed that the use of the standard cutoff score (below 24) as an indication of cognitive impairment may produce a significantly higher rate of false positives for African American and Hispanic American adults than for White adults, even when scores are adjusted for educational level. See, e.g., M. Bohnstedt, P. J. Fox, and N. D. Kohatsu, Correlates of mini-mental status examination among elderly demented patients: The influence of race-ethnicity, Journal of Clinical Epidemiology, 47, 1381-1387, 1994.

The correct answer is: African American and Hispanic adults.

187
Q

The routing subtests for the Stanford-Binet Intelligence Scales, Fifth Edition (SB5) are which of the following?
Select one:

A. Quantitative Reasoning and Memory for Sentences
B. Delayed Response and Block Span
C. Object Series/Matrices and Vocabulary
D. Procedural Knowledge and Verbal Analogies

A

Administration of the SB5 begins with the routing subtests, which are used to determine the beginning level of administration for the remaining subtests.

a. Incorrect See explanation for response c.
b. Incorrect See explanation for response c.
c. CORRECT The Object Series/Matrices and Vocabulary subtests are the routing subtests for the SB5.
d. Incorrect See explanation for response c.

The correct answer is: Object Series/Matrices and Vocabulary

188
Q

Which of the following MMPI-2 two-point codes is not accurately matched with its interpretation?
Select one:

A. 78/87: bipolar symptoms with substance abuse
B. 49/94: acting out behaviors with low frustration tolerance
C. 23/32: depressed with mild physical symptoms
D. 34/43: passive-aggressive and angry with poor insight

A

Knowing the names of the MMPI-2 scales listed in the answers may have helped you identify the correct response. Note that this question is asking which answer does not correctly match a two-point code with its interpretation.

a. CORRECT Scale 7 is the psychasthenia scale, and Scale 8 is the schizophrenia scale. Highest scores on these two scales is associated with significant emotional turmoil, depression, pessimism, OCD, and psychosis. (Bipolar disorder and possible substance abuse are suggested by a two-point code of 29/92.)
b. Incorrect Scale 4 is the psychopathic deviate scale, and Scale 9 is the hypomania scale. Highest scores on Scales 4 and 9 is associated with acting out behaviors, impulsivity, low frustration tolerance, and shallow, superficial relationships.
c. Incorrect Scale 2 is the depression scale, and Scale 3 is the hysteria scale. Highest scores on these two scales is indicative of mild to moderate depression with mild physical symptoms but without severe anxiety.
d. Incorrect Highest scores on Scale 3 (hysteria) and Scale 4 (psychopathic deviate) is associated with passive-aggressive, anger and hostility, a tendency to blame others, and poor insight.

The correct answer is: 78/87: bipolar symptoms with substance abuse.

189
Q

According to _______________, successful intelligence is composed of three components – analytical, creative, and practical.
Select one:

A. Sternberg’s triarchic model
B. Gardner’s multiple intelligences model
C. the Cattell-Horn-Carroll theory
D. Carroll’s three-stratum theory

A

For the exam, you want to be familiar with the four theories of intelligence listed in the answers to this question. These are described in the Psychological Assessment chapter of the written study materials.

a. CORRECT Sterberg’s triarchic theory describes successful intelligence as consisting of the three components listed in this question.
b. Incorrect See explanation for response a.
c. Incorrect See explanation for response a.
d. Incorrect See explanation for response a.

The correct answer is: Sternberg’s triarchic model

190
Q

Researchers relied primarily on which of the following techniques to derive the “Big Five” personality traits?
Select one:

A. meta-analysis
B. LISREL
C. discriminant analysis
D. factor analysis

A

Development of the five-factor model of personality was based on an atheoretical “lexical” approach.

a. Incorrect See explanation for response d.
b. Incorrect See explanation for response d.
c. Incorrect See explanation for response d.
d. CORRECT The lexical approach assumes that all socially-relevant personality characteristics are encoded in language, and the original identification of the “Big Five” traits began with an list of 18,000 traits derived by Gordon Allport from the 1925 edition of Webster’s Dictionary. Factor analysis of these traits by a number of investigators produced the same five core factors.

The correct answer is: factor analysis

191
Q

Schaie’s (1996) Seattle Longitudinal Study examined the relationship between cognitive abilities and:
Select one:

A. socioeconomic status.
B. speed of processing.
C. age.
D. gender.

A

This should have been an easy question as long as you know that a primary purpose of longitudinal studies is to investigate the effects of increasing age on the target characteristic(s).

a. Incorrect See explanation for response c.
b. Incorrect See explanation for response c.
c. CORRECT Schaie’s Seattle Longitudinal Study investigated age-related changes in cognitive abilities. In contrast to earlier cross-sectional studies which indicated that increasing age in adulthood is associated with declines in many cognitive abilities, Schaie’s study found that IQ remains relatively stable during adulthood and that substantial age-related declines prior to age 70 are apparent for only two abilities - perceptual speed and numerical ability.
d. Incorrect See explanation for response c.

The correct answer is: age.

192
Q

To assess the general intelligence of a six-year old child who is deaf, you would use which of the following tests?
Select one:

A. Halstead-Reitan
B. Kaufman Assessment Battery for Children
C. Fagan Test
D. Hiskey-Nebraska

A

A number of tests have been developed as alternatives to the Stanford-Binet and the Wechsler tests for assessing the general intellectual ability of individuals with disabilities.

a. Incorrect The Halstead-Reitan Neuropsychological Battery is a screening test for brain damage that assesses sensorimotor, perceptual, and language functioning.
b. Incorrect The Kaufman Assessment Battery for Children (KABC-II) was designed to be a culturally fair measure of cognitive ability for children ages 3 to 18.
c. Incorrect The Fagan Test of Infant Intelligence is for infants from 3 to 12 months of age.
d. CORRECT The Hiskey-Nebraska Test of Learning Aptitude was developed for use with deaf and hard-of-hearing children aged 3 to 17.

The correct answer is: Hiskey-Nebraska

193
Q

A person is exhibiting the actor-observer effect when he/she tends to:
Select one:

A. view the behavior of others as due to dispositional factors and his/her own behaviors as due to situational factors.
B. view the behavior of others as due to situational factors and his/her own behaviors as due to dispositional factors.
C. overestimates his/her own ability to succeed and underestimates the ability of others to succeed.
D. underestimates his/her own ability to succeed and overestimates the ability of others to succeed.

A

Knowing that the actor-observer effect refers to the types of attributions that people make (i.e., dispositional versus situational attributions) would have allowed you to eliminate answers c and d.

a. CORRECT Research on attribution of cause has found that we tend to attribute the behaviors of others to dispositional factors (which is referred to as the fundamental attribution bias). However, when making attributions about our own behaviors, we tend to attribute them to situational factors. The difference when making attributions about our own behaviors and the behaviors of others is referred to as the actor-observer effect.
b. Incorrect See explanation for response a.
c. Incorrect See explanation for response a.
d. Incorrect See explanation for response a.

The correct answer is: view the behavior of others as due to dispositional factors and his/her own behaviors as due to situational factors.

194
Q

When males view erotic films that depict aggressive acts toward females:
Select one:

A. they become more accepting of violence toward women.
B. they become more willing to support women’s rights.
C. they become sexually aroused and less aggressive, even when provoked.
D. they are less likely to act aggressively than males who view non-aggressive erotic films.

A

The research has found that, at least under certain conditions, viewing violence tends to increase violence.

a. CORRECT A number of studies have found that exposure to aggressive erotic films increases aggression toward females as well as acceptance of violence toward women.
b. Incorrect This is the opposite of what has been found.
c. Incorrect Studies have found that exposure to an aggressive erotic film can increase aggressiveness and that this effect is enhanced when the individual is provoked following viewing of the film.
d. Incorrect Exposure to mild non-aggressive erotica has been found to reduce aggression.

The correct answer is: they become more accepting of violence toward women.

195
Q

In a study examining affiliation, experimental subjects (introductory psychology students) are told that they will be receiving a series of electric shocks. Subjects are then given the opportunity to either wait alone, wait with other students who will be participating in the same experiment, or wait with other students who are merely waiting to see their faculty advisors. Results of the study will most likely confirm the hypothesis that:
Select one:

A. misery loves company.
B. misery loves miserable company.
C. misery loves unmiserable company.
D. misery loves no company.

A

Research on affiliation has found that highly anxious subjects tend to prefer to wait for an experiment to begin with other highly anxious subjects.

a. Incorrect Although subjects in studies similar to the one described in this question prefer to wait with others, they usually prefer to wait with other anxious (“miserable”) subjects.
b. CORRECT Schachter (1959) found that high anxiety subjects not only prefer to affiliate, but prefer to affiliate with others in similar circumstances in other words, “misery loves miserable company.”
c. Incorrect “Miserable” subjects like being in the company of others, especially others who are also miserable.
d. Incorrect Highly anxious subjects would rather be with others than wait alone.

The correct answer is: misery loves miserable company.

196
Q

Social comparison theory’s predictions about behavior are particularly applicable to:
Select one:

A. situations involving uncertainty.
B. inequitable situations.
C. people who are working on a difficult task.
D. people who belong to a cohesive group.

A

According to social comparison theory, people evaluate their own attitudes, abilities, and emotions by comparing themselves with similar others.

a. CORRECT Social comparison theory applies to a variety of situations but, as noted in the Social Psychology chapter of the written study materials, seems to be particularly applicable to situations that lack objective standards (i.e., to uncertain situations).
b. Incorrect See explanation above.
c. Incorrect See explanation above.
d. Incorrect See explanation above

The correct answer is: situations involving uncertainty.

197
Q

Which of the following is NOT true about the effects of crowding?
Select one:

A. Men are more negatively affected than women by crowded conditions.
B. Increasing age is associated with an increasing negative impact of crowding.
C. People are more willing to discuss intimate details in crowded (versus uncrowded) conditions.
D. Crowded (versus uncrowded) conditions are associated with a higher risk for health-related problems.

A

The outcomes of crowding have been linked to a number of individual and environmental characteristics.

a. Incorrect Men do tend to be more adversely affected by crowding.
b. Incorrect Children are less affected by crowding than adults.
c. CORRECT This is the opposite of what is true – people are less willing to discuss intimate topics and to seek social support in crowded situations. See, e.g., G. W. Evans et al., Residential density and psychological health: The mediation effects of social support, Journal of Personality and Social Psychology, 57, 994-999, 1989.
d. Incorrect Living and working in crowded conditions is associated with more health problems.

The correct answer is: People are more willing to discuss intimate details in crowded (versus uncrowded) conditions.

198
Q

According to Berscheid and Walster’s (1974) two-factor theory of love, love is the result of:
Select one:

A. attraction and arousal.
B. passion and intimacy.
C. mutual attraction and reciprocal reinforcement.
D. physiological arousal and a label for it.

A

Berscheid and Walster conceptualize love as being similar to other emotions.

a. Incorrect See explanation for response d.
b. Incorrect See explanation for response d.
c. Incorrect See explanation for response d.
d. CORRECT Support for the two-factor theory is provided, for example, by a study showing that couples who participate in an exciting, arousing game together subsequently report stronger positive feelings toward one another than couples who participate in a more mundane activity.

The correct answer is: physiological arousal and a label for it.

199
Q

A psychology intern considers her supervisor to be likeable, admirable, and accepting. As a consequence, the intern is open and responsive to her supervisor’s comments and recommendations. In this situation, the intern is responding to her supervisor’s:
Select one:

A. referent power.
B. reward power.
C. expert power.
D. legitimate power.

A

French and Raven’s (1959) six bases of social power are described in the Social Psychology chapter of the written study materials.

a. CORRECT In this situation, the supervisor has become a “significant reference person” for the intern – i.e., the supervisor’s influence stems from the intern’s desire to identify with (or “be like”) the influential person.
b. Incorrect A person has reward power when he/she controls desirable rewards.
c. Incorrect A person has expert power when he/she has superior knowledge or expertise.
d. Incorrect A person has legitimate power when he/she has authority, status, or social position.

The correct answer is: referent power.

200
Q

Fritz Heider’s (1958) balance theory proposes that the relationship between three entities can be either balanced or unbalanced. These entities are:
Select one:

A. a person, another person, and an attitude object.
B. the intrapersonal self, the interpersonal self, and the agent self.
C. attitudes, cognitions, and behaviors.
D. the person’s past, present, and future goals.

A

Answer A is correct. Balance theory is a cognitive consistency theory that predicts that a person is motivated to change an attitude when he/she experiences inconsistencies in attitudes. Balance theory is also known as P-O-X theory, where p is the person, O is another person, and X is the attitude object.

The correct answer is: a person, another person, and an attitude object.

201
Q

The Parent-Teacher Association (PTA) votes to keep a certain book out of the high school library because of its controversial nature. When students find out about the PTA’s decision, many of them immediately buy the book at local bookstores. This is an illustration of:
Select one:

A. deindividuation.
B. door-in-the-face.
C. psychological reactance.
D. reactivity.

A

In this situation, the students are reacting to a restriction placed on their access to a particular book.

a. Incorrect See explanation for response c.
b. Incorrect See explanation for response c.
c. CORRECT The tendency to act in ways opposite of what is required or requested (especially when one feels that one’s freedom is being threatened) is referred to as psychological reactance.
d. Incorrect See explanation for response c.

The correct answer is: psychological reactance.

202
Q

You are developing an educational program to discourage high school students from using drugs. You are considering using “fear arousal” as one method for persuading the students but should keep in mind that the research has shown that:
Select one:

A. lower levels of fear arousal are associated with the greatest amount of opinion change.
B. the greater the amount of fear aroused, the greater the opinion change.
C. high levels of fear arousal are associated with the greatest amount of opinion change as long as certain conditions are met.
D. fear arousal is not a good method for inducing opinion change.

A

Like many relationships, the relationship between fear arousal and attitude change is a complex one.

a. Incorrect See explanation for response c.
b. Incorrect See explanation for response c.
c. CORRECT Although some early research suggested that high fear arousal results in less attitude change than low fear arousal, subsequent studies have demonstrated that high levels of fear arousal are the most effective for inducing attitude change as long as the fear-arousing message is accompanied by information about what actions to take to reduce or avert dangerous consequences.
d. Incorrect See explanation for response c.

The correct answer is: high levels of fear arousal are associated with the greatest amount of opinion change as long as certain conditions are met.

203
Q

Which of the following is used to determine the amount of variability in one variable that is predictable from another variable?
Select one:

A. coefficient of determination
B. Cronbachs alpha
C. coefficient of concordance
D. eta

A

For the exam, you want to be familiar with the coefficients listed in the answers to this question.

a. CORRECT The coefficient of determination is calculated by squaring the correlation coefficient and provides a measure of shared variability, or the amount of variability in one variable that can be explained by variability in another variable (or, put another way, the amount of variability in one variable that is predictable from another variable).
b. Incorrect Cronbach’s alpha provides an estimate of a measure’s internal consistency reliability.
c. Incorrect The coefficient of concordance provides an estimate of inter-rater reliability for ranked data and three or more raters.
d. Incorrect Eta is a correlation coefficient that is used to estimate the variability of two continuous variables that have a nonlinear relationship.

The correct answer is: coefficient of determination

204
Q

Which of the following techniques would be appropriate for using scores on measures of self-esteem, hopelessness, and perceived social support to classify adolescents as either non-depressed, vulnerable to depression, or depressed?
Select one:

A. linear regression analysis
B. multiple regression analysis
C. multiple discriminant function analysis
D. principal components analysis

A

For the EPPP, you’ll want to be familiar with the appropriate use of each of the techniques listed in the answers to this question. Additional information about these techniques is provided in the Statistics and Research Design chapter of the written study materials.

a. Incorrect Linear regression is appropriate when a single predictor will be used to predict or estimate a score on a single criterion.
b. Incorrect Multiple regression is appropriate when multiple predictors will be used to predict a score on a single criterion.
c. CORRECT Multiple discriminant function analysis is an extension of discriminant function analysis when multiple predictors will be used to classify individuals into one of three or more criterion groups.
d. Incorrect Principle components analysis is similar to factor analysis and is used to identify the set of variables (components) that accounts for the total variance in a set of test scores.

The correct answer is: multiple discriminant function analysis

205
Q

To evaluate an intervention designed to increase the time a hyperactive child stays on-task when doing schoolwork, the best technique would be:
Select one:

A. situational sampling.
B. event recording.
C. interval recording.
D. sequential analysis.

A

There are several methods for sampling and recording behavioral observations. In this situation, the behavior may have no clear beginning or end, and the purpose of the evaluation is to determine how long the child exhibits the behavior.

a. Incorrect Situational sampling is used when an experimenter is interested in observing a behavior in different contexts.
b. Incorrect Event recording is used primarily to investigate rare events (behaviors) and usually involves counting the number of times the behavior occurs during an event, which wouldn’t be appropriate in this case.
c. CORRECT Interval recording involves dividing a period of time into equal intervals and recording whether or not the target behavior occurred during each interval. This technique could be easily used to assess time-on-task.
d. Incorrect Sequential analysis is used to study complex behavioral sequences (e.g., social interactions).

The correct answer is: interval recording.

206
Q

You conduct a research study to assess the effects of T.V. violence on aggressive behavior. You plan to observe sixteen children during recess for three days and calculate their average number of aggressive acts. You will then have all children observe aggressive T.V. programs for three hours and again observe the children during recess for three days and calculate their average number of aggressive acts. To analyze the data you collect, you will use which of the following?
Select one:

A. t-test for a single sample
B. t-test for correlated samples
C. one-way ANOVA
D. chi-square for a single sample

A

This study has one dependent variable that is measured on a ratio scale (number of aggressive acts) and one independent variable that has two levels (before film and after film).

a. Incorrect The t-test for a single sample is used to compare an obtained sample mean to a known population mean.
b. CORRECT The analysis will involve comparing two means obtained from the same group of participants. Therefore, the means are related and the t-test for related (correlated) samples is the appropriate test.
c. Incorrect The one-way ANOVA is used to compare two or more independent means.
d. Incorrect The chi-square is used to compare nominal (frequency) data – i.e., the number of observations in each category.

The correct answer is: t-test for correlated samples

207
Q

Obtaining research participants for a study on the sexual practices of single young adults from computer dating services, “singles’ clubs,” and activities for singles sponsored by a variety of organizations would help increase:
Select one:

A. statistical power.
B. statistical significance.
C. internal validity.
D. external validity.

A

In this situation, the researcher is obtaining participants from a variety of “natural” settings.

a. Incorrect See explanation for response d.
b. Incorrect See explanation for response d.
c. Incorrect See explanation for response d.
d. CORRECT External validity refers to the generalizability of research results. When participants are selected from natural settings (i.e., from settings where singles are likely to go to meet other singles), this will increase the generalizability of the results.

The correct answer is: external validity.

208
Q

To assess the relationship between the linear combinations of two or more X variables and two or more Y variables, you would use which of the following?
Select one:

A. multivariate analysis of variance
B. analysis of covariance
C. discriminant function analysis
D. canonical correlation

A

Answer D is correct: To identify the answer to this question, you need to determine (1) whether you are interested in the relationship (correlation) between variables or the effects of one or more variables on another variable or variables; and (2) how many variables there are. If you were just guessing, this would have been the best guess (and the right answer) since it is the only one that contains the word “correlation” and the question states that you are interested in the RELATIONSHIP between variables. Additional information about these techniques is provided in the Statistics and Research Design chapter of the written study materials.

Answer A: MANOVA is used to assess the effects of one or more IVs on two or more DVs.

Answer B: ANCOVA is used to assess the effects of an IV on a DV when the effects of an extraneous variable have been statistically removed.

Answer C: Discriminant function analysis is used to assess the relationship between variables when there are multiple X variables and one Y variable that is discrete (categorical).

The correct answer is: canonical correlation

209
Q

During an experiment, the investigator asks participants about their beliefs regarding the study’s purpose and how they were expected to perform. When analyzing the data, she finds that participants’ actual performance is consistent with their beliefs and expectations. This suggests that the study’s results may be confounded by:
Select one:

A. carryover effects.
B. the halo bias.
C. the Hawthorne effect.
D. demand characteristics.

A

In this situation, the participants may have acted in ways consistent with their expectations rather than simply in response to the experimental manipulation.

a. Incorrect Carryover effects occur in repeated measures designs when the effects of one treatment have an impact on the effects of subsequent treatments.
b. Incorrect The halo bias is a type of rating error and occurs when a rater’s rating of a person on one dimension of performance affects how the rater rates that person on unrelated dimensions of performance.
c. Incorrect The Hawthorne effect occurs when research participants act differently because of the novelty of the situation and the special attention they receive as research participants.
d. CORRECT Demand characteristics are unintentional cues in the experimental environment or manipulation that affect participants’ beliefs or expectations and thereby may account for the results of the study.

The correct answer is: demand characteristics.

210
Q

A research study examined the relationship between television viewing habits and academic achievement test scores. The results showed that children who watched more television generally had lower test scores. The Pearson-product moment correlation coefficient was -.81. What is the proportion of variance of achievement test scores that is accounted for by television viewing?
Select one:

A. 0.9
B. 0.66
C. -0.81
D. -0.9

A

The value of the Pearson-product moment correlation coefficient ranges from -1.0 to +1.0, and the corresponding proportion of variance (which is referred to as the coefficient of determination) is computed by squaring the correlation coefficient.

a. Incorrect See explanation for response b.
b. CORRECT The correlation coefficient for television viewing and achievement scores is -.81. The proportion of variance is calculated by squaring -.81 (i.e., -.812), which is approximately .66.
c. Incorrect See explanation for response b.
d. Incorrect See explanation for response b.

The correct answer is: 0.66

211
Q

Dr. Muzak wants to determine if background music (music versus no music) affects performance on three different cognitive tasks. Subjects will receive all combinations of all levels of music and task, but they will be administered in a different order to different subgroups of subjects. This is an example of which research design?
Select one:

A. split-plot
B. randomized block
C. Solomon
D. counterbalanced

A

Answer D is correct: This question describes a type of within-subjects design. The counterbalanced design is a type of within-subjects design in which the order of the different levels of the independent variable(s) are applied to different subjects or subgroups of subjects in a different order.

Answer A: When using a split-plot (mixed) design, the study has at least one between-groups variable and one within-subjects variable.

Answer B: In a randomized block design, at least one of the IVs is a blocking (extraneous) variable, and subjects within each block are randomly assigned to levels of another IV.

Answer C: The Solomon four-group design is used to measure the effects of pretesting on a study’s internal and external validity.

The correct answer is: counterbalanced

212
Q
Latent class analysis (LCA) is most similar to which of the following techniques?
Select one:

A. multitrait-multimethod matrix
B. cluster sampling
C. latent trait analysis
D. ANCOVA

A

LCA is used to identify the underlying latent structure of a set of observed data.

a. Incorrect The multitrait-multimethod matrix is used to assess construct (convergent and divergent) validity.
b. Incorrect LCA is similar to cluster analysis - not cluster sampling, which is a sampling technique that entails selecting clusters (groups) of participants rather than individual participants.
c. CORRECT Latent trait analysis (LTA) is also used to identify the underlying latent structure of a set of observed data. A primary difference between the two techniques is that, in LCA, the latent variable that determines the structure is nominal; while, in LTA, the latent variable is continuous.
d. Incorrect The ANCOVA (analysis of covariance) is a type of analysis of variance that is used to statistically remove the effects of an extraneous variable (the covariate) on the dependent variable so that it is easier to detect the effects of the independent variable on the dependent variable.

The correct answer is: latent trait analysis

213
Q

You conduct a research study to compare the effects of four weight-loss programs. To compare the average weight lost by participants in the four groups, you could conduct either a series of t-tests or a one-way ANOVA. You decide to use the one-way ANOVA because its use:
Select one:

A. will decrease the likelihood that you will make a Type I error.
B. will decrease the likelihood that you will make a Type II error.
C. will allow you to control the effects of an extraneous variable.
D. will allow you to assess both main and interaction effects.

A

If you conduct separate t-tests in this study, you will have to make many comparisons Treatment 1 vs. Treatment 2, Treatment 1 vs. Treatment 3, Treatment 1 versus Treatment 4, Treatment 2 vs. Treatment 3, etc. As the number of comparisons in a study increases, so too does the probability of making a Type I error.

a. CORRECT An advantage of the one-way ANOVA (or any ANOVA) is that it helps control the “experimentwise error rate” (i.e., the probability of making a Type I error). If alpha is set at .05 for this study, for instance, the probability of making a Type I error would be help at 5%. In contrast, if the individual t-tests were conducted, each at the .05 level, the probability of making a Type I error would be much higher.
b. Incorrect The opposite is actually true because of the inverse relationship between Type I and Type II errors.
c. Incorrect The analysis of covariance is useful for this purpose.
d. Incorrect A study must include at least two independent variables to assess interaction effects. This study seems to have only one independent variable.

The correct answer is: will decrease the likelihood that you will make a Type I error.

214
Q

A psychologist working at an alcohol rehabilitation clinic is conducting a study to determine if treatment length is related to relapse. She adds four weeks to the standard 12-week program and randomly assigns one-half of the current clinic clients to the extended 16-week treatment and the other half to the standard 12-week treatment. Relapse is measured as the number of times an individual drinks during the 3 months following the end of his or her treatment program. The appropriate correlation coefficient for the data collected in this study is:
Select one:

A. Pearson r.
B. tetrachoric.
C. point biserial.
D. Spearman rho.

A

In this situation, there is one continuous variable (relapse rate) and one dichotomous variable (treatment length – 12 weeks or 16 weeks).

a. Incorrect This would be the appropriate correlation coefficient if treatment length was also a continuous variable (e.g., the possible values were 1 week, 2 weeks, 3 weeks, and so on).
b. Incorrect The tetrachoric coefficient is used when both variables are continuous but have been artificially dichotomized.
c. CORRECT The point biserial coefficient is used when one variable is continuous and the other is a true dichotomy. (In this case, treatment length is a true dichotomy because participants receive either a 12-week or a 16-week program.)
d. Incorrect The Spearman rho (rank-ordered) correlation coefficient is used when data on both variables are ranks.

The correct answer is: point biserial.

215
Q

A test has a test-retest reliability coefficient of .70. This means that:
Select one:

A.
70% of variability in test scores is true score variability.

B.
49% of variability in test scores is true score variability.

C.
70% of variability in test scores is attributable to measurement error.

D.
49% of variability in test scores is attributable to measurement error.

A

As discussed in the Test Construction chapter of the written study materials, a reliability coefficient is always interpreted directly.

a. CORRECT A reliability coefficient of .70 indicates that 70% of variability in test scores is due to true score variability, while the remaining 30% is due to measurement error.
b. Incorrect See explanation above.
c. Incorrect See explanation above.
d. Incorrect See explanation above.

The correct answer is: 70% of variability in test scores is true score variability.

216
Q

Bobby B. obtains a test score of 110. The test has a mean of 120 and a standard deviation of 10 and test scores have a range of 100 and are normally distributed. If Bobby’s teacher converts all of the students’ test scores to T-scores or z-scores, Bobby’s score will be which of the following?
Select one:

A.
T = 60; z = +1.0

B.
T = 90; z = -1.0

C.
T = 40; z = -2.0

D.
T = 40; z = -1.0

A

T-scores and z-scores are standard scores, which means they report an examinee’s score in terms of standard deviations from the mean.

a. Incorrect See explanation for response d.
b. Incorrect See explanation for response d.
c. Incorrect See explanation for response d.
d. CORRECT Knowing that the T-score distribution has a mean of 50 and standard deviation of 10 and that the z-score distribution has a mean of 0 and standard deviation of 1 would have helped you identify the correct answer to this question. Since Bobby’s raw score is one standard deviation below the mean, his T-score would be 40 and his z-score would be -1.0. (In the T-score distribution, a score of 40 is one standard deviation below the mean and, in the z-score distribution, a z-score of -1.0 is one standard deviation below the mean.)
The correct answer is: T = 40; z = -1.0

217
Q

When discussing a new screening test that you’ve developed with several psychologists, one of the psychologists says, “Let’s say that I know a particular patient does not have the disorder that is assessed by the screening test. What is the chance that the test results will show that my patient does not have the disorder?” This psychologist is interested in your test’s:
Select one:

A.
sensitivity.

B.
specificity.

C.
positive predictive value.

D.
negative predictive value.

A

The terms listed in the answers to this question are used to describe the accuracy or usefulness of diagnostic tests. Additional information about these terms is provided in the Test Construction chapter of the written study materials.

a. Incorrect A test’s sensitivity refers to the proportion of individuals with the disorder who are correctly identified by the test as having the disorder.
b. CORRECT A test’s specificity refers to the proportion of individuals without the disorder who are correctly identified by the test as not having the disorder.
c. Incorrect A test’s positive predictive value refers to the proportion of individuals who test positive for the disorder and actually have the disorder.
d. Incorrect A test’s negative predictive value refers to the proportion of individuals who test negative for the disorder and actually do not have the disorder.

The correct answer is: specificity.

218
Q

When conducting a factor analysis, a researcher would choose an orthogonal (rather than oblique) rotation when:
Select one:

A.
the variables included in the analysis are correlated.

B.
the variables included in the analysis are uncorrelated.

C.
the effects of one or more variables have been removed from X only.

D.
the effects of one or more variables have been removed from X and Y.

A

When conducting a factor analysis, the rotation of factors can be either orthogonal or oblique.

a. Incorrect An oblique rotation is used when the variables included in the analysis are believed to be correlated. For example, if you conduct a factor analysis on three tests that are designed to measure verbal ability and three tests that are designed to measure nonverbal ability and there’s evidence that verbal and nonverbal ability are correlated, you would perform an oblique rotation.
b. CORRECT An orthogonal rotation is used when the variables included in the analysis are believed to be uncorrelated. For example, if you conduct a factor analysis on 50 questionnaire items designed to measure a leader’s task- or person-orientation and you believe that these two orientations are independent (uncorrelated), you would perform an orthogonal rotation.
c. Incorrect This answer describes semi-partial correlation.
d. Incorrect This answer describes partial correlation.

The correct answer is: the variables included in the analysis are uncorrelated.

219
Q

When conducting a principal components analysis, the components are extracted so that:
Select one:

A.
each component accounts for the same amount of variability.

B.
the first component accounts for the largest amount of variability.

C.
the last component accounts for the largest amount of variability.

D.
the middle component accounts for the largest amount of variability.

A

Principal components analysis is similar to factor analysis and is used to identify a set of variables that accounts for all (or nearly all) of the total variance in a set of test scores.

a. Incorrect See explanation for response b.
b. CORRECT Principal components are extracted so that the first component accounts for the largest amount of variability in test scores; the second component accounts for the second largest amount of variability, etc.
c. Incorrect See explanation for response b.
d. Incorrect See explanation for response b.

The correct answer is: the first component accounts for the largest amount of variability.

220
Q

“Common method bias” refers to the fact that there is likely to be some degree of correlation among similar methods of measurement even if they are designed to assess different characteristics. Which of the following techniques was designed to control the effects of common method bias?
Select one:

A.
analysis of covariance

B.
LISREL

C.
principal components analysis

D.
multitrait-multimethod matrix

A

As long as you weren’t overly distracted by the term “common method bias,” you may have been able to figure this out by the rest of the language in the question – i.e., “similar methods of measurement” and “measure different characteristics.”

a. Incorrect See explanation for response d.
b. Incorrect See explanation for response d.
c. Incorrect See explanation for response d.
d. CORRECT As its name implies, the multitrait-multimethod matrix involves using multiple methods to measure multiple traits.

The correct answer is: multitrait-multimethod matrix

221
Q

The internal consistency of test items can be assessed with which of the following?
Select one:

A.
kappa statistic

B.
split-half reliability

C.
alternate forms reliability

D.
coefficient of stability

A

Internal consistency (homogeneity of test items) is used as an index of reliability.

a. Incorrect The kappa statistic is used to measure inter-rater reliability.
b. CORRECT Split-half reliability is a type of internal consistency reliability and provides information on the consistency of items in two halves of the test.
c. Incorrect Alternative forms reliability provides information on the equivalence of items contained in two different forms of the test.
d. Incorrect The coefficient of stability (test-retest reliability coefficient) provides information of the consistency of a test over time.

The correct answer is: split-half reliability

222
Q

A psychologist constructs a new test of cynicism. She correlates the scores on her new test with scores on another test of cynicism and obtains a correlation of .22. Then, she correlates scores on her test with scores on a test of repressed anger and obtains a correlation of .60. The psychologist has obtained evidence that her test has:
Select one:

A.
good construct validity.

B.
inadequate construct validity.

C.
good criterion-related validity.

D.
inadequate criterion-related validity.

A

The psychologist has obtained a low correlation (.22) between her test and another test that measures the same theoretical construct (cynicism). In contrast, she obtained a relatively high correlation (.60) between her test and a test that measures a different theoretical construct (repressed anger).

a. Incorrect A test with good construct validity would have a high correlation with another test of the same trait (high convergent validity) and a low correlation with another test of a different trait (high divergent validity).
b. CORRECT This test has low convergent validity since the correlation between the test and another test of the same construct is low. It also has low divergent validity, as evidenced by the high correlation between the test and a test of a different trait. Convergent and divergent validity provide evidence of construct validity, or the usefulness of a test for measuring a theoretical trait or construct.
c. Incorrect A test has high criterion-related validity when scores on the test correlate highly with scores on a criterion measure.
d. Incorrect A test has low criterion-related validity when scores on the test do not correlate highly with scores on a criterion.

The correct answer is: inadequate construct validity.

223
Q

In the context of item response theory, item difficulty is defined:
Select one:

A.
by the number of examinees in the tryout sample who answered the item correctly.

B.
by the ability of the item to discriminate between examinees who fail or succeed on an external criterion.

C.
by the probability that an examinee with a given level of the ability measured by the test will answer the item correctly.

D.
by the probability that an examinee will pass the item given his/her predicted score on the test.

A

Item difficulty is one of the characteristics indicated by the item characteristic curve (ICC). See the Test Construction chapter for additional information about the ICC.

a. Incorrect This is how item difficulty in the context of classical test theory is determined.
b. Incorrect This is not how item difficulty is determined.
c. CORRECT An item is difficult when only examinees with very high levels of the attribute measured by the test can be expected to answer the item correctly at a high probability.
d. This is not how item difficulty is determined.

The correct answer is: by the probability that an examinee with a given level of the ability measured by the test will answer the item correctly.

224
Q

The optimal item difficulty (p) for a true-false test is:
Select one:

A.
.25.

B.
.50.

C.
.75.

D.
.95.

A

As described in the Test Construction chapter of the written study materials, the optimal item difficulty depends on several factors, including the likelihood that an examinee can choose the correct answer by guessing.

C is correct - The optimal item difficulty is halfway between 100% of examinees answering the item correctly and the likelihood of answering the item correctly by chance alone. For true-false items, the probability of choosing the correct answer by chance is .50, so the optimal difficulty level is .75.

The correct answer is: .75.